Download as pdf or txt
Download as pdf or txt
You are on page 1of 421

BioMedical Admissions Test 4500/12

November 2020 30 minutes

SECTION 2 Scientific Knowledge and Applications


INSTRUCTIONS TO CANDIDATES
*7334022998*

Please read this page carefully, but do not open the question paper until you are told that
you may do so.

This paper is Section 2 of 3. Your supervisor will collect this question paper and answer sheet
before giving out Section 3.

A separate answer sheet is provided for this section. Please check you have one. You also
require a soft pencil and an eraser.

Please complete the answer sheet with your:

• BMAT candidate number


• Centre number
• Date of birth
• Name

Speed as well as accuracy is important in this section. Work quickly, or you might not finish
the paper. There are no penalties for incorrect responses, only marks for correct answers, so
you should attempt all 27 questions. Each question is worth one mark.

Answer on the sheet provided. Questions ask you to show your choice between options by
shading one circle. If you make a mistake, erase thoroughly and try again.

You must complete the answer sheet within the time limit.

You can use the question paper for rough working or notes, but no extra paper is allowed.

Calculators are NOT permitted.

Please wait to be told you may begin before turning this page.

This question paper consists of 22 printed pages and 2 blank pages.


Copyright © UCLES 2020 PV4
BLANK PAGE

2
1 Which two of the following statements about genetic variation are correct?

1 Asexual reproduction always produces genetic variation.

2 Gamete production is the only possible source of genetic variation in sexual


reproduction.

3 The environment can cause genetic variation.

4 Mutations can produce genetic variation.

A 1 and 2 only

B 1 and 3 only

C 1 and 4 only

D 2 and 3 only

E 2 and 4 only

F 3 and 4 only

2 A sample of an element consists of two isotopes. The relative abundance of each isotope is
shown in the table.

number of number of number of relative


protons neutrons electrons abundance
isotope 1 31 38 31 60%
isotope 2 31 40 31 40%

What is the relative atomic mass of the element in this sample?

A 31.0

B 38.8

C 39.0

D 62.0

E 69.8

F 70.0

3
3 A supermarket has a large open-topped deep freezer to keep products frozen but still visible to
customers.

Which statement about the air in this freezer explains why the products remain frozen, even
though it is open-topped?

A The temperature difference between the air inside and outside the freezer is too large for
heat to enter the freezer.
B The temperature difference between the air inside and outside the freezer is too small for
heat to enter the freezer.
C The warm air above the freezer is denser than cold air inside the freezer.

D The cold air inside the freezer is denser than the hot air above the freezer.

E The products inside the freezer trap the cold air so it cannot escape.

4 Which one of the following is a simplification of

2x + 1
2– 2
4x + 4x + 1

4x + 1
A
2x + 1

4x + 3
B
2x + 1

4x + 9
C
2x + 5
2
8x + 4x + 1
D 2
4x + 2x + 1
2
8x + 4x + 1
E 2
4x + 2x + 2

F 2

4
5 Scientists are using human stem cells to develop treatments for a wide variety of health
conditions.

Two types of stem cell that are available for this work are:

• stem cells collected from an early embryo


• bone marrow stem cells collected from an adult

Which of the following statements is/are correct?

(Assume that no mutations occur.)

1 Both of these types of stem cell can divide producing daughter cells. Each daughter
cell will contain one haploid copy of the donor’s genome.
2 The stem cells collected from an embryo are able to differentiate into a wider variety
of specialised cells than the adult bone marrow stem cells.
3 The use of stem cell therapy to treat a medical condition can increase the risk of a
person developing cancer.

A none of them

B 1 only

C 2 only

D 3 only

E 1 and 2 only

F 1 and 3 only

G 2 and 3 only

H 1, 2 and 3

5
6 Hot, concentrated aqueous sodium hydroxide and chlorine react as shown in the equation:

6NaOH + 3Cl 2 → 5NaCl + NaClO3 + 3H2O

Which of the following statements is/are correct?

1 Chlorine has oxidation state + 5 in NaClO3.

2 This is an example of a disproportionation reaction.

3 Some of the oxygen in the hydroxide ions is oxidised.

A none of them

B 1 only

C 2 only

D 3 only

E 1 and 2 only

F 1 and 3 only

G 2 and 3 only

H 1, 2 and 3

6
7 An electric fan heater contains a heating element and a motor that drives the fan. The circuit
diagram is shown.

240 V
dc power supply

motor
M

switch heating element

At first, the switch is open and the current in the motor is 0.40 A.

The switch is then closed.

Which of the following statements is/are correct after the switch is closed?

1 The current in the heating element added to the current in the motor is 0.40 A.

2 The voltage across the heating element is 240 V.

3 The resistance of the circuit is smaller than it is with the switch open.

A none of them

B 1 only

C 2 only

D 3 only

E 1 and 2 only

F 1 and 3 only

G 2 and 3 only

H 1, 2 and 3

7
8 In a sale, the normal price of a camera is reduced by 20%.

The sale price of the camera is £180.

Which expression gives the normal price of the camera?

A 0.8 × £180

B 1.02 × £180

C 1.2 × £180

£180
D
0 .8
£180
E
0 .2
£180
F
0.08

9 Some rabbits have a genetic condition. The dominant allele codes for this condition.

A homozygous dominant rabbit mated with a rabbit that did not have the condition.

They had three offspring.

One of the offspring then mated with a rabbit that did not have the condition and they also
produced three offspring. Two of the offspring had the condition and one did not.

One body cell that is in early interphase is taken from each of the rabbits in these three
generations.

What is the total number of copies of the allele for the condition in this collection of cells?

A 6

B 7

C 8

D 11

E 12

F 14

8
10 The reaction between hydrogen and nitrogen in the presence of an iron catalyst is known as
the Haber process. It is a reversible reaction.

The energy profile for this reaction is shown.

What is the energy change when one mole of ammonia is decomposed into its component
elements?

A 46 kJ is absorbed

B 46 kJ is released

C 57 kJ is absorbed

D 57 kJ is released

E 92 kJ is absorbed

F 92 kJ is released

G 149 kJ is absorbed

H 149 kJ is released

9
11 A ‘gold medal’ used in events such as the Olympic Games is made from a silver–nickel alloy
with a coating of a gold–silver alloy.

The mass of the medal is 256 g, of which 24.0 g is the mass of the coating.

The density of the silver–nickel alloy is 10.0 g cm–3 and the density of the gold–silver alloy is
16.0 g cm–3.

What is the volume of the medal?

A 16.0 cm 3

B 16.9 cm 3

C 23.2 cm 3

D 24.7 cm 3

E 25.6 cm 3

F 27.1 cm 3

10
12 Triangle PQR is equilateral with sides of length 10 cm.

P, Q, and R are points on the circumference of a circle with centre O.

P [diagram not to scale]

10 cm 10 cm

Q R
10 cm

Which one of the following is an expression, in centimetres, for the radius of the circle?

A 5 cos 60°

5
B
tan 60°

C 5 sin 60°

5
D
sin 60°

E 5 tan 60°

5
F
cos 60°

11
13 Several processes can change the proportion of gases in the atmosphere by adding or
removing carbon dioxide or oxygen.

Which row is correct?

adds removes
adds oxygen removes oxygen
carbon dioxide carbon dioxide

aerobic respiration in activity of


A combustion photosynthesis
animals decomposers
anaerobic
activity of aerobic respiration in
B respiration in combustion
decomposers plants
animals
anaerobic
activity of aerobic respiration in
C photosynthesis respiration in
decomposers animals
animals
aerobic respiration in activity of
D photosynthesis photosynthesis
plants decomposers

activity of aerobic respiration in


E photosynthesis combustion
decomposers plants
anaerobic
aerobic respiration in aerobic respiration in aerobic respiration in
F respiration in
animals plants animals
animals

12
14 Four metals, labelled R, T, X and Z, have the following characteristics:

• R fizzes when added to dilute hydrochloric acid.


• T is precipitated when R reacts with an aqueous solution of a compound of T.
• X is the only one of these metals that occurs naturally on Earth uncombined with other
elements.
• Z is the only one of these metals manufactured commercially by electrolysis of one of its
molten compounds, whereas R and T can be manufactured by heating their oxides with
carbon.

What is the order of reactivity of these metals, from most to least reactive?

A R, Z, T, X

B R, T, X, Z

C T, X, R, Z

D X, R, T, Z

E X, T, R, Z

F Z, R, T, X

G Z, T, R, X

H Z, R, X, T

13
232
15 Thorium-232 ( 90 Th ) is an unstable nuclide that decays through a sequence of radioactive
emissions to form a stable nuclide of lead.

All of the emissions during this sequence are either alpha or beta (β–) particles.

One of the intermediate nuclides, reached after four alpha and two beta decays, is a nuclide of
an element labelled X.

What is the symbol for this nuclide of X?

214
A 80 X

216
B 80 X

216
C 84 X

216
D 88 X

224
E 84 X

224
F 88 X

224
G 90 X

228
H 88 X

16 The graphs of the following functions are drawn:

1 y = 2x − 1

2 y = 1− x 2

y = (1 − x )
2
3

4 y = 2− x

Which two graphs do not intersect?

A 1 and 2

B 1 and 3

C 1 and 4

D 2 and 3

E 2 and 4

F 3 and 4

14
17 Six test tubes were set up. Two different solutions were added to each test tube as shown in
the table.

The temperature, pH and all concentrations were optimal.

Which three mixtures would lead to the presence of amino acids in the test tube?

test tube mixture of solutions


1 lipase and lipid
2 lipase and boiled protein
3 protease and lipase
4 boiled protease and protein
5 protease and boiled protein
6 protease and protein

A 1, 2 and 6 only

B 1, 4 and 6 only

C 2, 3 and 4 only

D 2, 4 and 5 only

E 3, 4 and 6 only

F 3, 5 and 6 only

G 4, 5 and 6 only

15
18 Propene is an alkene. The structural formula of propene is shown.

H H
C C
H C H
H
H

A polymer can be made from propene.

What is the correct structural formula of this polymer?

CH3 H

D C C
H H
n

16
19 A diver of mass 45 kg jumps from a diving board and reaches a maximum height of 10 m above
the surface of the water in a swimming pool.

Assume Earth’s gravitational field strength is 10 N kg–1, that air resistance is negligible, and that
the diver is treated as a point particle.

What is the diver’s kinetic energy as she hits the water, and at what speed does she hit the
water?

kinetic energy / J speed / m s–1

A 450 10

B 450 2 5

C 450 10

D 4500 10

E 4500 10 2

F 4500 100

17
20 y
R (4, 5)
Q

P (0, 1)

PQRS is a kite. Points P, Q and R are plotted on the diagram.

P is the point (0, 1) and R is the point (4, 5).

angle PQR = 90°

The point Q lies on the y-axis.

The line PS is a segment of the line 2y + x = 2

The coordinates of S are ( l, m).

What is 2l + m ?

A –3

B 1

C 2

D 10

E 12

F 13

18
21 The table below gives some information about the number and biomass of organisms in the
food chain:

rose bush → aphid → ladybird

biomass at each
number of
organism level
organisms
(arbitrary units)
rose bush 1 800
aphid 4000 200
ladybird 50 10

What percentage of the biomass in the producer is transferred into the biomass of each
individual secondary consumer, in this food chain?

A 0.025

B 0.0625

C 0.1

D 1.25

E 5.0

F 25.0

22 The compounds BrCl and BrI melt at – 66 °C and 42 °C, respectively.

What is the best explanation for this difference in melting points?

A The Br − I covalent bond is stronger than the Br − Cl covalent bond.

B Chlorine is more reactive than iodine so chlorine breaks away from bromine more readily
than iodine.
C The forces between BrI molecules are stronger than the forces between BrCl molecules.

D Iodide ions are larger than chloride ions so bromide ions form stronger ionic bonds with
iodide ions than with chloride ions.
E Iodine atoms are larger than chlorine atoms so the BrI giant covalent lattice is more
difficult to break up than the BrCl giant covalent lattice.

19
23 A submarine uses sound of frequency 6000 Hz to locate obstructions ahead of it in the sea.

A pulse of this sound reflects off an object that is 750 m in front of the submarine. The pulse
returns to the submarine 1.00 s after it is transmitted.

What is the wavelength of this sound in the sea water?

A 12.5 cm

B 25.0 cm

C 4.00 m

D 8.00 m

E 4.50 km

F 9.00 km

20
24 The diameter XY of a circle has length d cm.

O d cm P

O is a point on the circumference of this circle such that a new circle with centre O passes
through points X and Y.

P is a point on the new circle and the arc XPY is shown in the diagram.

Which of the following is an expression for the perimeter, in cm, of the shaded region?

d (2 + π )
A
2
d (2 + 2π )
B
2
d (4 + π)
C
4
d (4 + 2π )
D
4
d (8 + π )
E
8
d (8 + 2π )
F
8

21
25 The diagram shows part of a DNA double helix being copied.

Key
X = unnamed bases
XX = unnamed base pairs

XX
XX
XX strand 2
strand 1
30% adenine (A), 28% thymine (T)
and 25% guanine (G) XX
XX
XX
XX

XX
X X
XX XX
pX rX
XX XX
XX XX

forming new strands

Which of the following statements, considered independently, is/are correct?

1 The probability of any base in strand 2 being guanine (G) is 0.17.

2 If p is guanine and r cytosine, a mutation must have taken place.

3 The copying process shown is taking place during mitosis.

A none of them

B 1 only

C 2 only

D 3 only

E 1 and 2 only

F 1 and 3 only

G 2 and 3 only

H 1, 2 and 3

22
26 Diammonium hydrogen phosphate, (NH4)2HPO4, can be used as a fertiliser.

The following equation shows how it can be synthesised:

2NH3(g) + H3PO4(aq) → (NH4)2HPO4(s)

What is the mass of diammonium hydrogen phosphate that is produced when 3.40 kg of
ammonia is reacted with phosphoric acid (H3PO4), in excess, with a yield of 80%?

(Mr values: NH3 = 17.0; H3PO4 = 98.0; (NH4)2HPO4 = 132)

A 5.28 kg

B 10.6 kg

C 13.2 kg

D 15.8 kg

E 21.1 kg

F 26.4 kg

27 A bar magnet is rotating at a constant rate of 120 revolutions per minute inside a circular coil of
wire, causing an induced voltage across the coil.

Which of the following statements is/are correct?

1 Increasing the number of turns of wire on the coil would increase both the maximum
value and the frequency of the induced voltage.

2 The direction of the induced voltage in this coil reverses every 0.25 seconds.

3 There is always an induced current in a conductor which is experiencing a change in


magnetic field.

A none of them

B 1 only

C 2 only

D 3 only

E 1 and 2 only

F 1 and 3 only

G 2 and 3 only

H 1, 2 and 3

END OF TEST

23
BLANK PAGE

This document was initially designed for print and as such does not reach accessibility standard
WCAG 2.1 in a number of ways including missing text alternatives and missing document structure.

If you need this document in a different format please email


admissionstesting@cambridgeassessment.org.uk telling us your name, email address and
requirements and we will respond within 15 working days.

24
BioMedical Admissions Test (BMAT) 2020

Section 2 answer key Section 2 score conversion

Question Answer BMAT


Total mark
1 F score
2 E 0 1
3 D 1 1
4 A 2 1
5 G 3 1.6
6 E 4 2.1
7 G 5 2.6
8 D 6 3
9 B 7 3.3
10 A 8 3.7
11 D 9 4
12 D 10 4.3
13 D 11 4.6
14 F 12 4.8
15 C 13 5.1
16 E 14 5.4
17 F 15 5.7
18 D 16 6
19 E 17 6.3
20 F 18 6.6
21 A 19 6.9
22 C 20 7.3
23 B 21 7.7
24 D 22 8.1
25 E 23 8.6
26 B 24 9
27 C 25 9
26 9
27 9
BioMedical Admissions Test 4500/12

Wednesday 30 October 2019 30 minutes

SECTION 2 Scientific Knowledge and Applications


INSTRUCTIONS TO CANDIDATES
*1452307714*

Please read this page carefully, but do not open the question paper until you are told that
you may do so.

This paper is Section 2 of 3. Your supervisor will collect this question paper and answer sheet
before giving out Section 3.

A separate answer sheet is provided for this section. Please check you have one. You also
require a soft pencil and an eraser.

Please complete the answer sheet with your:

• BMAT candidate number


• Centre number
• Date of birth
• Name

Speed as well as accuracy is important in this section. Work quickly, or you might not finish
the paper. There are no penalties for incorrect responses, only marks for correct answers, so
you should attempt all 27 questions. Each question is worth one mark.

Answer on the sheet provided. Questions ask you to show your choice between options by
shading one circle. If you make a mistake, erase thoroughly and try again.

You must complete the answer sheet within the time limit.

You can use the question paper for rough working or notes, but no extra paper is allowed.

Calculators are NOT permitted.

Please wait to be told you may begin before turning this page.

This question paper consists of 24 printed pages and 4 blank pages.

All questions in this paper assume knowledge that is


currently on the BMAT specification.
Copyright © UCLES 2019 PV3
BLANK PAGE

2
1 The diagram shows some stages involved in genetic engineering.

X Y

Which row correctly identifies W, X, Y and Z?

W X Y Z
A chromosome restriction enzyme plasmid ligase

B chromosome ligase bacterium restriction enzyme

C chromosome restriction enzyme bacterium ligase

D chromosome ligase plasmid restriction enzyme

E gene restriction enzyme plasmid ligase

F gene ligase bacterium restriction enzyme

G gene restriction enzyme bacterium ligase

H gene ligase plasmid restriction enzyme

3
2 A cleaning solution is used to remove limescale, CaCO3, from bathroom surfaces. When the
solution is sprayed onto the limescale, effervescence (fizzing) occurs and the solid limescale
begins to disappear. This chemical reaction is needed because the limescale cannot be
removed with water alone.

Which of the following statements about this chemical reaction are correct?

1 The cleaning solution is acidic.

2 The effervescence (fizzing) is caused by the release of hydrogen gas.

3 The pH of the reacting solution will go down as the reaction proceeds.

4 The salt produced in the reaction is more soluble than CaCO3.

A 1 and 2 only

B 1 and 3 only

C 1 and 4 only

D 2 and 3 only

E 2 and 4 only

F 3 and 4 only

G 1, 2 and 3 only

H 2, 3 and 4 only

4
3 The diagram shows the forces acting on a cyclist as she accelerates at 4.0 m s–2 on a straight,
horizontal section of road. The constant resistive forces are air resistance and a 300 N force
due to friction.

resistive forces acting against the cyclist


and bicycle =
300 N + air resistance forward force produced by the cyclist
and the bicycle = 600 N

The combined mass of the cyclist and bicycle is 50 kg.

What is the value of the air resistance?

A 50 N

B 100 N

C 200 N

D 300 N

E 450 N

4 The non-zero numbers p and q are such that p + q = 3 ( p – q )

pq
What is the value of
p + q2
2

1
A
3
2
B
5
1
C
2
3
D
5
2
E
3

5
5 The labels 1, 2 and 3 in the following diagram represent cellular processes.

The shaded areas represent processes that can happen without the use of energy provided by
cellular respiration.

1
2

Which processes could labels 1, 2 and 3 represent?

1 2 3
A active transport diffusion osmosis

B active transport diffusion aerobic respiration

C diffusion osmosis active transport

D diffusion aerobic respiration osmosis

E osmosis active transport aerobic respiration

F aerobic respiration active transport diffusion

G aerobic respiration osmosis active transport

6
6 A chemist used electrolysis to electroplate a pure copper rod with a layer of silver. The
concentration of metal ions in the electrolyte was kept constant.

Which row in the table identifies appropriate substances used as the anode (positive
electrode), the cathode (negative electrode) and the electrolyte?

anode cathode
electrolyte
(positive electrode) (negative electrode)
A pure silver rod pure copper rod copper nitrate solution

B pure copper rod pure silver rod copper nitrate solution

C pure graphite rod pure silver rod copper nitrate solution

D pure silver rod pure copper rod silver nitrate solution

E pure copper rod pure silver rod silver nitrate solution

F pure graphite rod pure copper rod silver nitrate solution

7
7 A measuring cylinder resting on an electronic balance contains a liquid as shown in diagram 1.

A small solid object X is gently lowered into the liquid and no liquid splashes out of the cylinder.

The result is shown in diagram 2.

cm3 cm3
500 500
450 450
400 400
350 350
300 300
250 250
200 200
150 150
100 object X 100
50 50

750.00
750.00 gg 950.00 g
diagram 1 diagram 2

What is the density of the material from which object X is made?

A 0.40 g cm–3

B 1.33 g cm–3

C 1.60 g cm–3

D 1.90 g cm–3

E 5.00 g cm–3

F 6.00 g cm–3

G 6.33 g cm–3

H 7.60 g cm–3

8
8 Find the value of

2 1
6 × 10 + 4 × 10
−2 −3
1.2 × 10 + 4 × 10

A 2

B 40

C 20

D 10 40

E 200

F 2000

9
9 A lizard was placed in a chamber with a constant flow of air. The temperature of the chamber
was varied between 25 °C and 10 °C. Apart from temperature and time, all other variables were
kept constant.

The carbon dioxide concentrations in the air leaving the chamber are shown in the graph.
carbon dioxide concentration
/ arbitrary units

400

350
25 °C
300

250 20 °C

200
15 °C
150
10 °C
100

50

0
0 50 100 150 200 250 300
time / minutes

Which of the following statements is/are correct?

1 Between 25 °C and 10 °C the respiration rate of the lizard is inversely proportional to


the environmental temperature.
2 The data demonstrates that the lizard is only respiring aerobically.

3 The carbon dioxide could be detected by passing the air leaving the chamber
through limewater.

A none of them

B 1 only

C 2 only

D 3 only

E 1 and 2 only

F 1 and 3 only

G 2 and 3 only

H 1, 2 and 3

10
10 Consider the non-radioactive elements in Group 1.

Which of the following statements is/are correct about the element with the largest atomic
number?

1 It has the most vigorous reaction with cold water.

2 Its atom has the same number of electrons in its highest occupied energy level as
other members of the group.

3 It is the most readily oxidised.

A none of them

B 1 only

C 2 only

D 3 only

E 1 and 2 only

F 1 and 3 only

G 2 and 3 only

H 1, 2 and 3

11
11 A large metal sphere mounted on an insulating stand is negatively charged. A smaller,
uncharged metal sphere mounted on an insulating stand is placed near to the negatively
charged sphere.

The smaller sphere is connected to the Earth by a conducting wire. Electrons flow from the
smaller sphere to Earth because they are repelled by the larger sphere. The smaller sphere is
then disconnected from the Earth.

Which row must be correct about the particles in the larger sphere and the final charge of the
smaller sphere?

particles in the larger sphere final charge of the smaller sphere


A there are fewer protons than electrons negative

B there are fewer protons than electrons neutral

C there are fewer protons than electrons positive

D there are more protons than electrons negative

E there are more protons than electrons neutral

F there are more protons than electrons positive

12
12 Point M has coordinates (6, 3p −1) and point N has coordinates (1− p, 2).

The gradient of the straight line joining M and N is − 3 and it crosses the y-axis at (0, r ).

What is the value of r ?

A − 11

B −9

C −5

D −1

E 1

F 5

G 9

H 11

13 Which of the following is/are included in the carbon cycle?

1 amino acids in an animal

2 cellulose in a plant cell wall

3 lipids in a bacterial cell membrane

A none of them

B 1 only

C 2 only

D 3 only

E 1 and 2 only

F 1 and 3 only

G 2 and 3 only

H 1, 2 and 3

13
14 The decomposition of hydrogen peroxide in the presence of a manganese(IV) oxide catalyst
produces water and oxygen gas.

2H2O2(aq) → 2H2O(l) + O2(g)

0.2 g of manganese(IV) oxide granules are added to 50 cm3 of 0.1 mol dm–3 hydrogen peroxide
at 20 °C. The volume of gas collected is shown on the graph as curve X.

A second experiment is carried out at 20 °C using the same mass of manganese(IV) oxide.
The volume of gas collected is shown as curve Y.

total volume
of gas

0
0 time

Which of the following conditions could result in curve Y?

volume of concentration of
manganese(IV) oxide
hydrogen peroxide hydrogen peroxide
particle size
/ cm3 / mol dm–3
A powder 20 0.2

B powder 25 0.2

C powder 50 0.1

D granules 20 0.1

E granules 25 0.2

14
15 Which statement about waves is correct?

A All longitudinal waves travel at the same speed.

B Longitudinal waves cause vibrations perpendicular to the direction of energy transfer.

C Sound waves are transverse.

D Transverse waves always travel faster than longitudinal waves.

E Transverse waves are all electromagnetic.

F Ultrasonic waves are longitudinal.

G Ultraviolet waves consist of compressions and rarefactions.

H X-rays cannot travel in a vacuum.

16 ( )
One side of a rectangle is 7 − 5 cm.

The rectangle has an area of 66 cm2.

What is the perimeter, in cm, of the rectangle?

35 − 5 5
A
2

B 35 − 5 5

35 + 5
C
2

D 35 + 5

105 − 15 5
E
2

F 105 − 15 5

105 + 7 5
G
2

H 105 + 7 5

15
17 A pea plant is heterozygous for seed colour and homozygous for seed shape.

Yellow seed colour (Y) is dominant to green seed colour (y). Round seed shape (R) is
dominant to wrinkled seed shape (r).

Using the information provided, which of the following statements, taken independently,
could be correct?

1 This plant contains the alleles Y, y, R and r.

2 This plant contains only the alleles Y, y and R.

3 This plant contains only the alleles Y, R and r.

4 The genotype of this plant for seed colour is Yy.

5 The genotype of this plant for seed colour is YY.

6 The genotype of this plant for seed shape is Rr.

7 The genotype of this plant for seed shape is rr.

A 1, 4 and 7 only

B 1, 5 and 6 only

C 2, 4 and 6 only

D 2, 4 and 7 only

E 2, 5 and 7 only

F 3, 4 and 7 only

G 3, 5 and 6 only

H 3, 5 and 7 only

16
18 The concentration of Fe2+ ions in a solution can be determined by titration with acidifed
potassium manganate(VII) solution.

The ionic equation for the reaction is:

5Fe2+ + MnO4– + 8H+ → 5Fe3+ + Mn2+ + 4H2O

A 25.0 cm3 sample of solution containing Fe2+ ions reacted exactly with 10.0 cm3 of
0.0500 mol dm–3 potassium manganate(VII) solution.

What is the concentration of Fe2+, in mol dm–3 ?

A 1.00 × 10–4 mol dm–3

B 4.00 × 10–3 mol dm–3

C 1.25 × 10–2 mol dm–3

D 2.00 × 10–2 mol dm–3

E 1.00 × 10–1 mol dm–3

F 1.00 × 101 mol dm–3

17
19 Cosmologists have deduced that soon after the Universe was formed it was filled with
electromagnetic radiation in the form of intense gamma-rays. They have also deduced that the
Universe has been expanding since that time. One effect of this expansion is that the
electromagnetic radiation that fills the Universe is now in the microwave region of the spectrum.

Here are three statements about the effects of the Universe expanding on the electromagnetic
radiation that filled the early Universe:

1 The expansion has caused an increase in the frequency of the radiation.

2 The expansion has caused a decrease in the wavelength of the radiation.

3 The expansion has caused frequency to change in direct proportion to wavelength.

Which of the statements is/are correct?

A none of them

B 1 only

C 2 only

D 3 only

E 1 and 2 only

F 1 and 3 only

G 2 and 3 only

H 1, 2 and 3

18
20 To get to work, Sylvie first catches a bus and then catches a train.

The probability that the bus is on time is 0.6

The probability that the bus is late is 0.4

If the bus is on time, then the probability that she will catch the train is 0.8

If the bus is late, then the probability that she will catch the train is 0.6

Given that Sylvie catches the train, what is the probability that the bus was on time?

1
A
3
12
B
25
2
C
5
3
D
5
2
E
3
18
F
25
6
G
7

19
21 Which of the following statements is/are correct for a healthy human?

1 A protein section coded for by part of a gene consisting of 500 base pairs will have a
maximum of 166 amino acids.
2 A liver cell and a mature red blood cell from the same person will both have a
chromosome number of 46.
3 The number of adenine bases in an allele must be the same as the number of
guanine bases.

A none of them

B 1 only

C 2 only

D 3 only

E 1 and 2 only

F 1 and 3 only

G 2 and 3 only

H 1, 2 and 3

22 Complete combustion of 35 cm3 of a straight-chain alkane vapour gave 105 cm3 of carbon
dioxide gas. Both gas volumes were measured at the same temperature and pressure.

Which of the following is the molecular formula of the alkane?

A C2H4

B C2H6

C C3H6

D C3H8

E C4H10

20
23 A water jet pack has lifted a person vertically upwards, as shown in the sketch. He is now
stationary at a constant height.

person

nozzle
ejected water
tube
water

Water rises through the tube and is ejected at a speed of 15 m s–1 through two nozzles.

In a time of 12 s, each of the nozzles ejects 400 kg of water vertically downwards.

What is the momentum of the water ejected by each nozzle in 12 s, and what is the upward
force due to the water ejected by the jet pack?

momentum / kg m s–1 force / N

A 6000 1000

B 6000 2000

C 12 000 1000

D 12 000 2000

E 45 000 3750

F 45 000 7500

G 90 000 7500

H 90 000 15 000

21
24 A cross-country running track is in the shape of a regular pentagon.

second leg
third leg

first leg

fourth leg

start & finish


fifth leg

[diagram not to scale]

Competitors run clockwise around the track.

When on the third leg of the course they run on a bearing of 110°.

What bearing do they run on for the first leg?

A 034°

B 038°

C 106°

D 178°

E 182°

F 244°

G 322°

H 326°

22
25 A student carried out tests to find out which molecules were present in a sample of urine from a
human male. The student recorded the presence of large protein molecules in the urine, and
suggested that this indicated that the kidney was not functioning correctly.

Which of the following could have resulted in a high concentration of protein in the urine
sample?

1 damage to cell membranes between the blood vessels and the Bowman’s capsule

2 a reduced rate of active transport and selective re-absorption from the nephron

3 Cells in the collecting duct do not have receptors to bind to antidiuretic hormone
(ADH) and so are unable to respond to the hormone.

A none of them

B 1 only

C 2 only

D 3 only

E 1 and 2 only

F 1 and 3 only

G 2 and 3 only

H 1, 2 and 3

23
26 Diborane has the formula B2H6.

10 11
Assume that boron consists of two isotopes, containing 20% 5B atoms and 80% 5B atoms,
1
and that all hydrogen atoms are 1H .

Molecules of diborane will therefore have relative masses of 26, 27 or 28.

In what relative proportion will molecules of diborane with masses of 26, 27 and 28 occur?

A 1:2:8

B 1 : 2 : 16

C 1:4:8

D 1 : 4 : 16

E 1 : 8 : 16

F 1 : 8 : 64

G 1 : 16 : 64

24
27 A 100% efficient ideal transformer has 400 turns on its primary coil and 100 turns on its
secondary coil. The input voltage across the primary coil is 240 V. An output current of 2.0 A
flows in the secondary coil.

primary coil: secondary coil:


400 turns 100 turns

input voltage output current


240 V 2.0 A

What are the output voltage across the secondary coil, the input current in the primary coil, and
the output power of the transformer?

output voltage / V input current / A output power / W


A 60 0.50 30

B 60 0.50 120

C 60 8.0 120

D 60 8.0 480

E 960 0.50 480

F 960 0.50 1920

G 960 8.0 1920

H 960 8.0 7680

END OF TEST

25
BLANK PAGE

26
BLANK PAGE

27
BLANK PAGE

28
BioMedical Admissions Test – BMAT October 2019
SECTION 2
Answer key

Question Answer
1 E
2 C
3 B
4 B
5 A
6 D
7 C
8 E
9 D
10 H
11 C
12 H
13 H
14 A
15 F
16 D
17 D
18 E
19 A
20 E
21 B
22 D
23 A
24 H
25 B
26 E
27 B
BioMedical Admissions Test – BMAT October 2019
SECTION 2
Score conversion

Total mark BMAT score


0 1.0
1 1.0
2 1.0
3 1.6
4 2.1
5 2.5
6 2.8
7 3.1
8 3.4
9 3.7
10 3.9
11 4.1
12 4.4
13 4.6
14 4.8
15 5.1
16 5.3
17 5.5
18 5.8
19 6.0
20 6.3
21 6.6
22 6.9
23 7.3
24 7.7
25 8.3
26 9.0
27 9.0
BioMedical Admissions Test 4500/12

Wednesday 31 October 2018 30 minutes


*3273846895*
*9058458629*

SECTION 2 Scientific Knowledge and Applications

INSTRUCTIONS TO CANDIDATES

Please read this page carefully, but do not open the question paper until you are told that
you may do so.

This paper is Section 2 of 3. Your supervisor will collect this question paper and answer sheet
before giving out Section 3.

A separate answer sheet is provided for this section. Please check you have one. You also
require a soft pencil and an eraser.

Please complete the answer sheet with your:

• BMAT candidate number


• Centre number
• Date of birth
• Name

Speed as well as accuracy is important in this section. Work quickly, or you may not finish
the paper. There are no penalties for incorrect responses, only marks for correct answers, so
you should attempt all 27 questions. Each question is worth one mark.

Answer on the sheet provided. Questions ask you to show your choice between options by
shading one circle. If you make a mistake, erase thoroughly and try again.

You must complete the answer sheet within the time limit.

You can use the question paper for rough working or notes, but no extra paper is allowed.

Calculators are NOT permitted.

Please wait to be told you may begin before turning this page.

This paper consists of 19 printed pages and 5 blank pages.


This paper consists of 20 printed pages and 4 blank pages.

The question in this paper marked with an asterisk (* Q5) assumes


knowledge that is not currently on the BMAT specification.
PV4 Copyright © UCLES 2018
BLANK PAGE

2
1 People with cystic fibrosis may be unable to digest their food fully because of thick sticky
mucus. This prevents the secretion of pancreatic juices. Doctors sometimes prescribe enzyme
supplement medication to improve digestion.

Which of the following enzymes should be included in the medication?

1 amylase

2 lipase

3 protease

A none of them

B 1 only

C 2 only

D 3 only

E 1 and 2 only

F 1 and 3 only

G 2 and 3 only

H 1, 2 and 3

3
2 Elements Q and Z are shown in the Periodic Table as:

What is the empirical formula and bonding type of the compound formed between these
elements?

empirical formula bonding type


A QZ ionic

B QZ covalent

C QZ2 ionic

D QZ2 covalent

E Q2Z ionic

F Q2Z covalent

3 Here are four statements about light:

1 In a vacuum, green light travels faster than blue light.

2 In a vacuum, the wavelength of red light is greater than the wavelength of green
light.
3 In water, red light travels faster than it travels in a vacuum.

4 In water, the frequency of blue light is greater than the frequency of red light.

Which pair of statements is correct?

A 1 and 2

B 1 and 3

C 1 and 4

D 2 and 3

E 2 and 4

F 3 and 4

4
5
48 m p
4 Which one of the following is a simplification of 2 3
?
40 m p
5 1
6 2 3
A m p
5

6 3 −2
B m p
5
6 3 −3
C m p
5
5 1

D 8m 2 p 3

3 −2
E 8m p

3 −3
F 8m p

* 5 The diagram shows part of the nitrogen cycle.

nitrogen gas 2
1
ammonium ions
5 6
nitrates
nitrites
7
4
3

protein

Which of the numbered arrows represent(s) denitrification?

A 1 only

B 2 only

C 3 only

D 5 only

E 1 and 4 only

F 2 and 5 only

G 3 and 6 only

H 6 and 7 only

5
6 The diagram below shows the apparatus that can be used in fractional distillation.

Which of the following statements is/are correct about the process shown in the diagram?

1 Covalent bonds between atoms are broken at position 1 and have reformed at
position 3.
2 The particles are closer together at position 3 than at position 2.

3 The physical process at position 1 involves an exothermic change.

A none of them

B 1 only

C 2 only

D 3 only

E 1 and 2 only

F 1 and 3 only

G 2 and 3 only

H 1, 2 and 3

6
7 A wave travelling through a material has a frequency of 50 Hz and a wavelength of 0.40 m.

How long does it take the wave to travel 100 m through this material?

A 0.20 s

B 0.80 s

C 1.25 s

D 2.00 s

E 5.00 s

F 20.0 s

8 My wallet contains two 50p coins and five 20p coins only.

I need 70p to buy an item.

If I pick two coins at random from my wallet, what is the probability that I have picked the exact
money to buy the item?

1
A
2
5
B
21
10
C
21
20
D
49
25
E
441
50
F
441

7
9 A sample of healthy human cells contains 4 cells. After they were allowed to divide 5 times
using the same type of cell division, there were 128 cells in the sample.

Assuming that no mutations occur, which two of the following statements are correct?

1 All the cells would have the same number of chromosomes.

2 The cells were gametes.

3 The cells would be clones of the original cells.

4 This type of cell division is called meiosis.

A 1 and 2 only

B 1 and 3 only

C 1 and 4 only

D 2 and 3 only

E 2 and 4 only

F 3 and 4 only

10 Iron is extracted from iron ore (which contains Fe2O3) using carbon monoxide.

This overall reaction is slow and so must be heated.

The chemical equation for this reaction under certain conditions is shown below:

Fe2O3 + 3CO → 2Fe + 3CO2 ∆H = − 30 kJ / mol

Which one of the following statements about this process is correct?

A The iron ions in Fe2O3 lose electrons to become Fe.

B The oxide ions in Fe2O3 each have a charge of − 6.

C The Fe2O3 acts as an oxidising agent in this reaction.

D The reaction is an endothermic process.

E The reaction is an example of electrolysis.

8
11 Quantities associated with a moving vehicle include its velocity, speed, momentum and kinetic
energy.

Which of these quantities will always change if the vehicle changes direction?

A speed and kinetic energy only

B speed and momentum only

C speed, kinetic energy and momentum only

D velocity and kinetic energy only

E velocity and momentum only

F velocity, kinetic energy and momentum only

9
12 All four corners of a rectangle are on the circumference of a circle.

The rectangle has a perimeter of 24 cm.

The ratio of length : width for the rectangle is 3 : 1

[diagram not to scale]

What is the area of the shaded region in cm2 ?

9
A π − 27
4
B 9 π − 27

C 16 π − 32

D 20 π − 32

81π
E − 27
4
45 π
F − 27
2
G 64 π − 32

H 80 π − 32

10
13 The diagram shows three different types of animal cell.

1 epithelial cell

2 mature red blood cell

3 muscle cell

Which of these cells is/are found as part of a tissue?

A none of them

B 1 only

C 2 only

D 3 only

E 1 and 2 only

F 1 and 3 only

G 2 and 3 only

H 1, 2 and 3

11
14 The masses of atoms and molecules can be determined using a mass spectrometer. The
masses can be shown as a series of peaks.

Element X exists as a diatomic molecule. The mass spectrum will show the mass of ions from
individual atoms of X and from X2 molecules.

The mass spectrum for element X is shown below:


relative abundance

0 5 10 15 20 25 30 35 40 45 50 55 60 65 70 75 80
mass
charge

How many different isotopes of X can be determined from this spectrum?

A 1

B 2

C 3

D 4

E 5

12
15 A champion weightlifter raises a 200 kg set of weights from the floor to above his head in one
movement.

The bar is lifted through a vertical distance of 1.8 m.

When the lift is completed the weightlifter holds the weights stationary for 2.0 seconds and then
drops them to the floor.

1.8 m

At what speed do the weights hit the floor?

(gravitational field strength = 10 N / kg)

A 0.90 m / s

B 1.1 m / s

C 3.0 m / s

D 3.6 m / s

E 6.0 m / s

F 9.0 m / s

G 18 m / s

H 36 m / s

13
16 The first four triangular numbers are:

1 3 6 10

The difference between the first and second triangular numbers is 2.

The difference between the second and third triangular numbers is 3.

The difference between the third and fourth triangular numbers is 4.

All the triangular numbers follow this pattern.

The difference between the r th triangular number and the (r + 3 ) triangular number is 126.
th

What is the value of r ?

A 39

B 40

C 41

D 42

E 43

14
17 The diagram shows a simple piece of apparatus that can be used to demonstrate some of the
events involved in human ventilation.

Y tube
bung (stopper)

glass container balloon

rubber
diaphragm

Which events occurring during ventilation will be demonstrated by pulling the rubber diaphragm
downwards, as shown by the arrow?

1 diaphragm relaxes

2 diaphragm contracts

3 volume of thorax increases

4 volume of thorax decreases

5 pressure in thorax increases

6 pressure in thorax decreases

A 1, 3, 5

B 1, 3, 6

C 1, 4, 5

D 1, 4, 6

E 2, 3, 5

F 2, 3, 6

G 2, 4, 5

H 2, 4, 6

15
18 Which of the following shows the correct structure of part of the polymer formed by the
polymerisation of CH3 – CH = CH – CH3 ?

H H H H H H

A C C C C C C

H H H H H H

19 A circuit consists of a 10 Ω resistor and a variable resistor connected in series with a 6.0 V
battery. The variable resistor has a minimum resistance of 2.0 Ω and a maximum resistance of
20 Ω.

What is the difference between the largest and smallest currents possible in this circuit?

A 0.20 A

B 0.30 A

C 0.40 A

D 0.50 A

E 0.60 A

F 0.70 A

16
20 The mean mass of a sweet in a bag of 20 sweets must be greater than 10 grams but not
greater than 10.5 grams.

A bag is being filled with sweets. The mean mass of the first 16 sweets is exactly 9.5 grams.

Four more sweets, each of mass x grams, are added to the bag to bring the mean mass of the
20 sweets into the correct range.

What is the complete range of possible values of x ?

A 12 < x ≤ 14.5

B 12 < x < 14.5

C 12 ≤ x < 14.5

D 10.125 < x ≤ 10.725

E 10.125 < x < 10.725

F 10.125 ≤ x < 10.725

21 An animal cell is surrounded by a very dilute glucose solution which has a lower concentration
of glucose than the glucose solution in the cytoplasm of the cell. There is net movement of
glucose molecules and water molecules into the cell.

A second, identical cell is treated for a short time with a chemical which inhibits respiration. The
cell is then surrounded by the same glucose solution.

Which row in the table shows the effect of this chemical on the movement of glucose molecules
into the cell and the movement of water molecules across the cell surface membrane
immediately after it is surrounded by the solution?

net movement of glucose molecules net movement of water molecules

A do not move into the cell move into the cell

B move into the cell move into the cell

C do not move into the cell no net movement into or out of the cell

D move into the cell no net movement into or out of the cell

E do not move into the cell move out of the cell

F move into the cell move out of the cell

17
22 A compound of oxygen and fluorine has a relative molecular mass which is twice that of its
empirical formula mass. 105 g of the compound contains 57 g of fluorine.

What is the molecular formula of the compound?

(Ar values: O = 16; F = 19)

A OF

B OF2

C O2F2

D O3F3

E O6F6

23 The radioactive isotope plutonium-244 becomes radon-220 after a succession of decays. The
atomic number of plutonium is 94 and the atomic number of radon is 86.

How many alpha particles and how many beta particles are emitted altogether during the decay
of one nucleus of plutonium-244 to radon-220?

alpha particles beta particles


A 6 2

B 6 4

C 8 8

D 12 4

E 12 16

18
2
x x +3
24 Which one of the following is a simplification of − ?
x −1 x2 + 2x − 3

1
A
x +1
3
B
x+3
3( x − 1)
C
( x + 1)( x − 3 )
3( x + 1)
D
( x − 1)( x + 3 )
2
x −3
E
(
( x − 1) x 2 + 2 x − 3 )

25 In a type of fruit fly the brown body colour allele is dominant to the black body colour allele. A
male fruit fly heterozygous for body colour and a female fruit fly heterozygous for body colour
were allowed to mate, producing offspring.

What is the ratio, in its simplest form, of the maximum number of genotypes in the offspring to
the maximum number of phenotypes in the offspring?

A 4:3

B 3:4

C 3:2

D 3:1

E 2:3

F 1:2:1

19
26 Xenon gas reacts with fluorine gas to form gaseous xenon hexafluoride at high temperature
and pressure. The overall energy change is − 330 kJ / mol.

Under these conditions, the F – F bond energy of the fluorine gas is 158 kJ / mol.

What is the Xe – F bond energy?

A 81.3 kJ / mol

B − 81.3 kJ / mol

C 134 kJ / mol

D − 134 kJ / mol

E 213 kJ / mol

F − 213 kJ / mol

G 804 kJ / mol

H − 804 kJ / mol

27 The radioactive isotope carbon-14 is found in living material in small quantities. There are
approximately 1000 carbon-14 atoms for every 1015 carbon-12 atoms. Whilst the material is still
living this ratio remains constant, because even though the carbon-14 is decaying, it is being
constantly replenished. When the material dies the carbon-14 decays and is not replaced. The
half-life of carbon-14 is about 6000 years.

In a bone the ratio of carbon-14 to carbon-12 atoms is found to be 100 : 1015 .

Which of the following is the closest estimate of the age of the bone?

A 60 years

B 600 years

C 1000 years

D 10 000 years

E 20 000 years

F 30 000 years

G 50 000 years

H 60 000 years

END OF TEST

20
BLANK PAGE

21
BLANK PAGE

22
BLANK PAGE

23
BLANK PAGE

24
BioMedical Admissions Test – BMAT October 2018
SECTION 2
Answer key

Question Answer
1 H
2 C
3 E
4 B
5 A
6 C
7 E
8 C
9 B
10 C
11 E
12 F
13 H
14 B
15 E
16 B
17 F
18 E
19 B
20 A
21 A
22 C
23 B
24 B
25 C
26 C
27 E
BioMedical Admissions Test – BMAT October 2018
SECTION 2
Score conversion

Total Mark BMAT score


0 1.0
1 1.0
2 1.3
3 1.8
4 2.3
5 2.6
6 2.9
7 3.2
8 3.4
9 3.6
10 3.8
11 4.0
12 4.2
13 4.4
14 4.6
15 4.8
16 5.0
17 5.2
18 5.4
19 5.7
20 5.9
21 6.2
22 6.5
23 6.8
24 7.2
25 7.7
26 8.6
27 9.0
BioMedical Admissions Test 4500/12

Thursday 2 November 2017 30 minutes


  

SECTION 2 Scientific Knowledge and Applications

INSTRUCTIONS TO CANDIDATES

Please read this page carefully, but do not open the question paper until you are told that you
may do so.

This paper is Section 2 of 3. Your supervisor will collect this question paper and answer sheet
before giving out Section 3.

A separate answer sheet is provided for this section. Please check you have one. You also require a
soft pencil and an eraser.

Please complete the answer sheet with your:

• BMAT candidate number


• Centre number
• Date of birth
• Name

Speed as well as accuracy is important in this section. Work quickly, or you may not finish the
paper. There are no penalties for incorrect responses, only marks for correct answers, so you
should attempt all 27 questions. Each question is worth one mark.

Answer on the sheet provided. Questions ask you to show your choice between options by shading
one circle. If you make a mistake, erase thoroughly and try again.

You must complete the answer sheet within the time limit.

You can use the question paper for rough working or notes, but no extra paper is allowed.

Calculators are NOT permitted.

Please wait to be told you may begin before turning this page.

This paper consists of 23 printed pages and 5 blank pages.

The question in this paper marked with an asterisk (* Q7) assumes


knowledge that is not currently on the BMAT specification.

PV3 © UCLES 2017


2

BLANK PAGE

© UCLES 2017
3

1 The diagram shows a section through part of a healthy human.

P
Q

Which row in the table shows the correct secretions from P, Q and R?

secretion(s) from P secretions from Q secretion(s) from R

A bile protease, high hydrogen insulin, protease, lipase,


ion concentration amylase
B bile, high hydrogen ion protease, low hydrogen protease, lipase,
concentration ion concentration amylase
C insulin, protease, lipase protease, high hydrogen bile
ion concentration
D protease, lipase, protease, low hydrogen bile, high hydrogen ion
amylase ion concentration concentration
E insulin, protease, lipase, protease, high hydrogen bile
amylase ion concentration
F protease, lipase, protease, low hydrogen insulin
amylase ion concentration

© UCLES 2017 [Turn over


4

2 The diagram shows an electrolysis experiment.

+ – power supply

ammeter
A

+ –
beaker

cathode (copper)

anode (copper)
copper(II) sulfate solution

Which reaction occurs at the anode and which reaction occurs at the cathode for this
experiment?

anode cathode
A SO42– → SO2 + O2 + 2e– Cu2+ + 2e– → Cu

B Cu2+ + 2e– → Cu SO42– → SO2 + O2 + 2e–

C 2H+ + 2e– → H2 Cu → Cu2+ + 2e2–

D Cu2+ + 2e– → Cu Cu → Cu2+ + 2e–

E Cu → Cu2+ + 2e– Cu2+ + 2e– → Cu

© UCLES 2017
5

3 Below are four statements about electromagnetic waves.

1 Microwaves have a smaller wavelength than visible light.

2 The speed of visible light in a vacuum is higher than the speed of other types of
electromagnetic wave.
3 Gamma rays have the largest wavelength of any electromagnetic wave.

4 Radio waves are used in hospital radiography to look for broken bones.

Which of the statements are correct?

A none of them

B 1 and 2 only

C 1 and 3 only

D 1 and 4 only

E 2 and 3 only

F 2 and 4 only

G 3 and 4 only

H 1, 2, 3 and 4

4 Which one of the following is equivalent to ( 5 −2 ) 2


?

A 1

B 9

C 21

D 29

E 1− 4 5

F 9−4 5

G 21− 4 5

H 29 − 4 5

© UCLES 2017 [Turn over


6

5 SCID is an inherited condition in humans. People with some types of SCID are unable to make
a functional enzyme necessary for the production of healthy white blood cells.

A scientist studying these types of SCID compared features of the DNA and the structure of the
enzyme in people who have this condition and people without SCID.

Which of the following features may have been different?

1 the allele of the gene coding for the enzyme

2 the order of amino acids in the enzyme

3 the order of bases in the gene coding for the enzyme

4 the shape of the active site of the enzyme

A 1 only

B 4 only

C 1 and 3 only

D 2 and 3 only

E 2 and 4 only

F 1, 2 and 4 only

G 1, 2, 3 and 4

© UCLES 2017
7

6 Which of the following atoms and ions contain(s) 20 neutrons and 18 electrons?

34 2 –
1 16S

37 –
2 17Cl

40
3 18 Ar

39 +
4 19K

40
5 20Ca

A 3 only

B 1 and 2 only

C 1 and 3 only

D 2 and 4 only

E 2 and 5 only

F 4 and 5 only

G 2, 4 and 5 only

H 1, 2, 3 and 4 only

© UCLES 2017 [Turn over


8

* 7 A puddle is left on a road after a rain shower. The water in the puddle slowly disappears by
evaporation.

Three statements about the effect of changing different conditions on this process are given
below.

1 The rate of evaporation is greater at higher temperatures.

2 The rate of evaporation is greater when the air above the puddle is still.

3 The rate of evaporation is greater when the puddle has a larger surface area.

Which of the statements is/are correct?

(Assume for each statement that all other conditions remain the same.)

A none of them

B 1 only

C 2 only

D 3 only

E 1 and 2 only

F 1 and 3 only

G 2 and 3 only

H 1, 2 and 3

© UCLES 2017
9

8 In a group of 20 patients at a medical centre, 5 suffered from migraines.

Two patients from the group of 20 were picked at random for a survey on the use of painkillers.

What is the probability that both of the patients picked suffered from migraines?

1
A
19
1
B
16
35
C
76
1
D
2
21
E
38
9
F
16

© UCLES 2017 [Turn over


10

9 The diagram shows four experiments used to investigate movement of substances across
dialysis tubing. This tubing is a partially permeable membrane which allows both glucose and
water to pass through it.

experiment 1 experiment 2 experiment 3 experiment 4

10%
glucose
solution
distilled 10%
water glucose
solution
distilled
water
dialysis
tubing

Which row in the table shows the experiment(s) where there will be movement of glucose
through the partially permeable membrane and the experiment(s) where there will be
movement of water through the partially permeable membrane?

experiment(s) with movement of experiment(s) with movement of


glucose through partially water through partially
permeable membrane permeable membrane
A 4 only 3 only

B 3 only 4 only

C 2 and 4 only 1 and 3 only

D 2, 3 and 4 only 1, 3 and 4 only

E 2, 3 and 4 only 1, 2, 3 and 4

© UCLES 2017
11

10 A piece of magnesium ribbon is dropped into a beaker of dilute aqueous hydrochloric acid at
room temperature. After a while the production of bubbles of gas slows down.

Which of the following statements can correctly explain this observation?

1 The particles have less energy.

2 The concentration of hydrochloric acid decreases.

3 The activation energy for the reaction increases.

A none of them

B 1 only

C 2 only

D 3 only

E 1 and 2 only

F 1 and 3 only

G 2 and 3 only

H 1, 2 and 3

© UCLES 2017 [Turn over


12

11 Two resistors R1 and R2 are connected in series with a cell.

Resistor R1 has twice the resistance of resistor R2.

R1 R2

Six statements about this circuit are given below.

1 The voltage across each resistor is the same.

2 The voltage across R1 is twice that across R2.

3 The voltage across R2 is twice that across R1.

4 The current is the same in both resistors.

5 The current in R1 is twice the current in R2.

6 The current in R2 is twice the current in R1.

Which pair of statements is correct?

A 1 and 4 only

B 1 and 6 only

C 2 and 4 only

D 2 and 5 only

E 2 and 6 only

F 3 and 4 only

G 3 and 5 only

H 3 and 6 only

© UCLES 2017
13

12 In triangle PRS, line QT is parallel to side RS.

RS = 1.5 cm

TS = 1.8 cm

QT = 0.3 cm

[diagram not to scale]


R

1.5 cm
Q

0.3 cm

P T S
1.8 cm

What is the length of PS?

A 0.25 cm

B 0.36 cm

C 0.45 cm

D 2.05 cm

E 2.16 cm

F 2.25 cm

© UCLES 2017 [Turn over


14

13 Dolly the sheep was born in 1996. She was unusual because she had no biological father.

Sheep have a diploid number of 54 chromosomes in their body cells.

The diagram shows the process of how she was produced.


1. mammary gland 2. egg cell (gamete)
(udder) cell donor donor

nucleus removed
cultured from gamete
mammary gland
cells
3. cells fused
nucleus from
mammary gland cell
4. grown in culture

early embryo

5. implanted
in uterus
surrogate mother
of a third
sheep

6. development lamb (‛Dolly’)

Which of the following statements about this process is/are correct?

1 The gamete cell nucleus contained 27 chromosomes.

2 The cells produced in step 4 had the same properties as stem cells.

3 None of the cells involved in the process were produced by meiosis.

A none of them

B 1 only

C 2 only

D 3 only

E 1 and 2 only

F 1 and 3 only

G 2 and 3 only

H 1, 2 and 3

© UCLES 2017
15

14 Disproportionation is the simultaneous oxidation and reduction of the same species in a


reaction.

In which of the following chemical equations does disproportionation occur?

1 Fe + CuCl2 → FeCl2 + Cu

2 Cu2O → Cu + CuO

3 Cl2 + H2O → HCl + HClO

4 BaCl2 + Na2SO4 → BaSO4 + 2NaCl

5 Hg2Cl2 → Hg + HgCl2

A 2 only

B 2 and 3 only

C 3 and 4 only

D 1, 4 and 5 only

E 2, 3 and 5 only

15 Uranium-238 is a naturally-occurring alpha emitter. It can be used in the manufacture of the


isotope plutonium-239, during which it is bombarded by neutrons.

The process of converting a nucleus of uranium-238 to a nucleus of plutonium-239 is a


three-stage sequence of nuclear reactions.

What is this sequence?

(Atomic numbers: uranium = 92; plutonium = 94)

1st stage 2nd stage 3rd stage

A emission of an α particle emission of a β particle absorption of a neutron

B emission of an α particle absorption of a neutron emission of an α particle

C emission of a β particle emission of an α particle absorption of a neutron

D emission of a β particle absorption of a neutron emission of a β particle

E absorption of a neutron emission of an α particle emission of an α particle

F absorption of a neutron emission of a β particle emission of a β particle

© UCLES 2017 [Turn over


16

GM
16 The acceleration due to gravity at the surface of a planet is given by g = 2
, where M and R
R
are the mass and radius of the planet respectively, and G is the gravitational constant.

It is given that

g = 10 N / kg

G = 7 × 10–11 N m2 / kg2

R = 6 × 106 m

What is M, correct to one significant figure?

A 5 × 1024 kg

B 2 × 1024 kg

C 5 × 1020 kg

D 2 × 1020 kg

E 5 × 1018 kg

F 2 × 1018 kg

© UCLES 2017
17

17 The diagram shows an external view of the human heart.

part of an artery
present on the
surface of the heart

Which of the following statements about the artery shown on the diagram is/are correct?

1 It is the site of diffusion of glucose and oxygen into the muscle cells of the heart.

2 It transports blood at high pressure.

3 It contains muscle cells.

A none of them

B 1 only

C 2 only

D 3 only

E 1 and 2 only

F 1 and 3 only

G 2 and 3 only

H 1, 2 and 3

© UCLES 2017 [Turn over


18

18 Propanoic acid is a monoprotic acid.

Magnesium is a Group 2 metal.

Which of the following chemical equations is correct for the reaction between magnesium
carbonate and propanoic acid?

A 2CH3CH2COOH + MgCO3 → Mg(C3H5O2)2 + H2O

B C3H6O2 + MgCO3 → MgC3H5O2 + CO2 + H2O

C 2C3H6O2 + MgCO3 → Mg(C3H5O2)2 + CO2 + H2O

D 2CH3COOH + MgCO3 → Mg(CH3COO)2 + CO2 + H2O

E C3H6O2 + 3MgCO3 → Mg3C3O2 + 3CO2 + 3H2O

19 A swimming pool is 10 m wide. A loud sound is made in the water 2.0 m from one wall and the
reflected sounds are detected with a microphone placed next to the sound source. The
reflection from the wall 8.0 m away arrives 0.010 s after the reflection from the wall 2.0 m away.

What is the speed of sound in water?

A 270 m / s

B 330 m / s

C 530 m / s

D 600 m / s

E 1200 m / s

© UCLES 2017
19

1 1 1
20 Express + − as a single algebraic fraction.
2x x −1 x

1
A
2x −1
1
B
2( x − 1)
2x −1
C
x ( x − 1)
x −3
D
2 x ( x − 1)
x +1
E
2 x ( x − 1)
1
F
x ( x − 1)
−1
G
x ( x − 1)

© UCLES 2017 [Turn over


20

21 The family tree shows the inheritance of freckles.

Key
1 2
male with freckles

female with freckles


3 4 5 6
male without freckles

female without freckles


7 8

Which row in the table shows the probability that the next child produced by parents 1 and 2,
and the probability that the next child produced by parents 5 and 6 will have freckles?

probability that the next child produced probability that the next child
by parents 1 and 2 will have freckles produced by parents 5 and 6 will
have freckles
A 1 1

B 0.5 1

C 1 0.75

D 0.5 0.75

E 1 0.5

F 0.5 0.5

G 1 0.25

H 0.5 0.25

© UCLES 2017
21

22 Hydrated copper(II) sulfate has the formula CuSO4·5H2O.

100 cm3 of a solution contained 10 g of hydrated copper(II) sulfate. What is the concentration,
in mol / dm3, of this solution?

(Ar values: H = 1.0; O = 16; S = 32; Cu = 64)

A 0.100 mol / dm3

B 0.400 mol / dm3

C 0.495 mol / dm3

D 0.532 mol / dm3

E 0.562 mol / dm3

F 0.625 mol / dm3

23 A book rests on a table, which in turn rests on the floor. The floor exerts a force P on the table.

Force P and one other force constitute a Newton’s Third Law interaction pair of forces.

What is the other force?

A the force that the book exerts on the table

B the force that the book exerts on the Earth

C the force that the Earth exerts on the book

D the force that the Earth exerts on the table

E the force that the floor exerts on the Earth

F the force that the table exerts on the floor

© UCLES 2017 [Turn over


22

24 The diagram shows a shape made from a quarter circle of radius 6 cm and a right-angled
triangle with a hypotenuse of length 9 cm.

[diagram not to scale]

9 cm
6 cm

Which of the following expressions gives the area, in square centimetres, of the shape?

A 3π + 9 5

B 3 π + 9 13

C 9π + 9 5

D 9 π + 9 13

E 9 π + 18 5

F 9 π + 27 5

G 36 π + 9 5

© UCLES 2017
23

25 The picture shows a Siamese cat. This type of cat has a gene which is sensitive to
temperature. When this gene is inactive in the cells, the coat colour is pale. When the gene is
active, it produces an enzyme which causes the coat colour to be darker. If this cat had grown
up in a warmer environment its ears, front of face, paws and tail would be paler than those
shown in the picture below.

A student wrote the following statements using this information.

1 The enzyme is denatured at the internal body temperature of the cat so the coat
colour is pale.
2 The temperature of the ears, front of face, paws and tail was cooler than the body
temperature so they are darker.
3 The coat colour in Siamese cats depends on both genes and the environment.

Which of these statements is/are correct?

A none of them

B 1 only

C 2 only

D 3 only

E 1 and 2 only

F 1 and 3 only

G 2 and 3 only

H 1, 2 and 3

© UCLES 2017 [Turn over


24

26 What is the volume of hydrogen gas formed (measured at room temperature and pressure)
when 0.23 g of pure sodium reacts completely with an excess of water?

(Ar values: H = 1.0; Na = 23.


Assume that the molar volume of gas at room temperature and pressure is 24 dm3.)

A 0.10 dm3

B 0.12 dm3

C 0.20 dm3

D 0.24 dm3

E 0.48 dm3

27 A graph of kinetic energy, in joules ( y - axis) against the square of the speed in (m / s)2 (x - axis)
is plotted for an object of mass 2.5 kg travelling along the surface of the Earth. The result is a
straight line.

What is the numerical value of the gradient of this line?

A 0.40

B 0.80

C 1.25

D 2.50

E 5.00

F 6.25

END OF TEST

© UCLES 2017
25

BLANK PAGE

© UCLES 2017
26

BLANK PAGE

© UCLES 2017
27

BLANK PAGE

© UCLES 2017
28

BLANK PAGE

© UCLES 2017
BioMedical Admissions Test – BMAT November 2017
SECTION 2
Answer key

Question Answer
1 A
2 E
3 A
4 F
5 G
6 D
7 F
8 A
9 E
10 C
11 C
12 F
13 E
14 E
15 F
16 A
17 G
18 C
19 E
20 E
21 D
22 B
23 F
24 C
25 G
26 B
27 C
BioMedical Admissions Test – BMAT November 2017
SECTION 2
Score conversion

Total Mark BMAT score


0 1.0
1 1.0
2 1.0
3 1.3
4 1.8
5 2.2
6 2.6
7 2.9
8 3.2
9 3.5
10 3.7
11 4
12 4.2
13 4.5
14 4.7
15 4.9
16 5.2
17 5.4
18 5.6
19 5.9
20 6.2
21 6.5
22 6.8
23 7.2
24 7.7
25 8.3
26 9
27 9
BioMedical Admissions Test 4500/12

Wednesday 2 November 2016 30 minutes


*5389754902*

SECTION 2 Scientific Knowledge and Applications

Instructions to Candidates

Please read this page carefully, but do not open the question paper until you are told that you
may do so.

This paper is Section 2 of 3. Your supervisor will collect this question paper and answer sheet
before giving out Section 3.

A separate answer sheet is provided for this section. Please check you have one. You also require a
soft pencil and an eraser.

Please complete the answer sheet with your:

• BMAT candidate number


• Centre number
• Date of birth
• Name

Speed as well as accuracy is important in this section. Work quickly, or you may not finish the
paper. There are no penalties for incorrect responses, only marks for correct answers, so you
should attempt all 27 questions. Each question is worth one mark.

Answer on the sheet provided. Questions ask you to show your choice between options by shading
a circle. If you make a mistake, erase thoroughly and try again.

Any rough work should be done on this question paper.

Calculators are NOT permitted.

Please wait to be told you may begin before turning this page.

This paper consists of 20 printed pages and 4 blank pages.

All questions in this paper assume knowledge that is


currently on the BMAT specification.
PV4 © UCLES 2016
2

BLANK PAGE

© UCLES 2016
3

1 The diagram shows a kidney and its associated vessels from a healthy individual.

2
1

3
[not to scale]

Which row correctly identifies the vessels along with the concentration of urea they contain?

lowest concentration of urea highest concentration of urea


A 1 is the aorta 2 is the vena cava

B 1 is the vena cava 2 is the aorta

C 3 is the renal artery 5 is the urethra

D 3 is the renal vein 5 is the ureter

E 4 is the renal vein 5 is the ureter

F 4 is the renal artery 5 is the urethra

© UCLES 2016 [Turn over


4

2 Element X has the electronic structure 2, 8, 3.

Which of the following statements about this element are correct?

1 The element is in Group 12, Period 3 of the Periodic Table.

2 The element reacts with oxygen to form a compound with the formula X2O3.

3 The element reacts with bromine to form a compound with the formula XBr3.

4 The atomic number of the element is 13.

5 The element is an alkali metal.

A 1 and 5 only

B 2 and 3 only

C 2 and 5 only

D 3 and 4 only

E 1, 4 and 5 only

F 2, 3 and 4 only

© UCLES 2016
5

3 A student carries out an experiment to determine the density of the material from which two
identical solid objects are made. She uses a balance and a measuring cylinder containing a
fixed volume of liquid. The diagrams show different stages of her experiment, with some of the
readings on the balance and some on the measuring cylinder.

350 cm3

250 cm3

20 g 170g 470g

Which calculation should be used to determine the density of the material from which the
objects are made?

280
A g / cm3
50
280
B g / cm3
300
280
C g / cm3
350
300
D g / cm3
50
300
E g / cm3
100
600
F g / cm3
350
750
G g / cm3
350
770
H g / cm3
350

© UCLES 2016 [Turn over


6

4 A straight line passes through the points P ( − 3, 3) and Q (6, 9).

Which one of the following is an equation of a straight line which is parallel to PQ?

A y = 2x − 3
3

B y = 3x − 8
2

C y= x−1
4 2

D y = x −2
2

E y = 2 x − 11

F y = 4 x − 23

© UCLES 2016
7

5 The diagram shows some of the stages of how a length of DNA can be removed from one
organism and introduced into another organism.

cut out by X
W

cut by Y inserted using Z

[not to scale]

Which row is correct?

W is a X is a Y is a Z is a
A chromosome restriction enzyme restriction enzyme restriction enzyme

B chromosome restriction enzyme restriction enzyme ligase

C chromosome ligase ligase ligase

D gene ligase restriction enzyme ligase

E gene ligase ligase restriction enzyme

F gene restriction enzyme ligase restriction enzyme

6 Which one of the following mixtures could not be separated using the technique given?

mixture separating technique


A calcium carbonate and water evaporation

B pentane and octane fractional distillation

C silicon dioxide and water filtration

D sodium chloride and water distillation

E ethanol and water separating funnel

© UCLES 2016 [Turn over


8

7 Nickel has an atomic number of 28. The mass numbers of four of its isotopes are 58, 60, 61
and 62.

Below are three statements about these isotopes of nickel.

1 All of them have the same chemical properties.

2 All of them have nuclei containing 28 protons.

3 One of them has a nucleus that contains 62 neutrons.

Which statement(s) is/are correct?

A 1 only

B 2 only

C 3 only

D 1 and 2 only

E 1 and 3 only

F 2 and 3 only

G 1, 2 and 3

H none of them

8 The mean mass of a group of N people is 75 kg.

Jim, Karen and Leroy join this group, without anyone leaving; the new mean mass is 78 kg.

The mean mass of Jim, Karen and Leroy is 90 kg.

What is the value of N?

A 4

B 12

C 15

D 30

E 48

F 90

© UCLES 2016
9

9 The following statements are features of an enzyme from a healthy human.

1 It works at an optimum pH below 4.

2 It digests a substrate into amino acids.

3 It works at an optimum temperature of approximately 37°C.

Which enzyme has these features?

A amylase in the mouth

B amylase from the pancreas

C lipase in the mouth

D lipase from the pancreas

E protease from the small intestine

F protease from the stomach

10 Mohr’s Salt is a common laboratory reagent. Use the information of the most abundant
isotopes below to calculate the formula mass, Mr, of the hydrated salt.

Formula of Mohr’s Salt: (NH4)2Fe(SO4)2.6H2O

Most abundant isotopes: 11H, 147 N, 168 O, 32 56


16 S, 26 Fe

A 144

B 204

C 284

D 360

E 374

F 392

© UCLES 2016 [Turn over


10

11 An AC voltage is induced in a coil of wire when it is rotated in a magnetic field.

This voltage can be displayed on the screen of an instrument called an oscilloscope, which
shows the variation of the induced voltage on the y-axis plotted against time on the x-axis.

The oscilloscope trace below is the result of rotating a coil at a constant speed in a uniform
magnetic field.
y

Which one of the following traces would result from rotating the coil at a faster constant speed,
whilst keeping all other conditions unchanged? (The scales on the axes do not change.)

A y B y C
y

x x x

D y
E
y

x x

© UCLES 2016
11

12 The diagram represents the circular cross-section of an artery with external diameter d of
1.6 cm. The thickness t of the artery wall is 1 mm.

t = 1 mm

[not to scale]

d = 1.6 cm
What is the internal cross-sectional area of the artery (shaded in the diagram)?

A 0.36π mm2

B 14π mm2

C 15π mm2

D 49π mm2

E 225π mm2

13 Which of the following molecules are involved in both aerobic and anaerobic respiration in a
healthy human?

1 carbon dioxide

2 glucose

3 lactic acid

A 1 only

B 2 only

C 1 and 2 only

D 1 and 3 only

E 2 and 3 only

F 1, 2 and 3

© UCLES 2016 [Turn over


12

14 Identify the correct products of electrolysis of the following electrolytes.

electrolyte being product at positive inert product at negative inert


electrolysed electrode (anode) electrode (cathode)
A aqueous calcium bromide bromine calcium

B aqueous copper nitrate nitrogen copper

C aqueous potassium sulfate oxygen hydrogen

D molten aluminium oxide aluminium oxygen

E molten sodium chloride chlorine hydrogen

© UCLES 2016
13

15 The diagram represents a satellite communication link between two points on Earth. The
distances between the transmitting station, the satellite and the receiving station are shown.

The frequency of the waves used for the link can be taken as 1.5 × 1010 Hz, and the speed of
light as 3.0 × 108 m / s.

satellite [not to scale]

45 000 km
45 000 km

transmitting
station receiving
station
Earth

What type of wave is used in such a link, and what is the time delay between a signal being
transmitted and then being received at the receiving station?

type of wave time delay (s)


A microwave 0.0015

B microwave 0.0030

C microwave 0.15

D microwave 0.30

E ultraviolet 0.0015

F ultraviolet 0.0030

G ultraviolet 0.15

H ultraviolet 0.30

© UCLES 2016 [Turn over


14

16 The diagram shows a quadrilateral PQRS.

Q 5 cm R
[not to scale]

θ
P S
11 cm

Given that tan θ = 4 , what is the area of the quadrilateral PQRS?


3

A 34 cm2

B 36 cm2

C 64 cm2

D 88 cm2

E 112 cm2

17 Which of the following statements could describe the result of a single mutation of a gene
coding for a protease enzyme in a fertilised human egg cell?

1 A new allele is formed coding for a protease enzyme that works more efficiently.

2 A new allele is formed coding for a protease enzyme that works less efficiently.

3 A new allele is formed coding for a non-functional protein that has no effect on the
cell.
4 A new allele is formed coding for a non-functional protein that has a negative effect
on the cell.

A 1 only

B 2 only

C 1 and 2 only

D 3 and 4 only

E 1, 2 and 3 only

F 1, 2 and 4 only

G 1, 2, 3 and 4

© UCLES 2016
15

18 What volume of a 0.10 mol dm–3 solution of NaOH is needed to neutralise 30 cm3 of a
0.20 mol dm–3 aqueous solution of a diprotic acid?

A 7.5 cm3

B 15 cm3

C 30 cm3

D 60 cm3

E 120 cm3

19 A defibrillator provides an electric shock to a heart to restore its normal rhythm. The energy to
do this is 125 J, at a steady voltage of 500 V, for a time of 10 milliseconds.

What is the current during this process?

A 2.5 mA

B 25 mA

C 2.5 A

D 25 A

E 6250 A

F 6250 kA

20 Rearrange the formula a = c + e to make f the subject.


b d f

A f = bce − ade
ac

B f = bc + be − ad
a

f = bde
C
ad − bc

D f = bcd
ad − be

E f = bc + be
ad

© UCLES 2016 [Turn over


16

21 Which statements describe a role of mitosis?

1 asexual reproduction

2 growth of a cell

3 repair of cells

4 stem cell division

A 1 and 4 only

B 1, 2 and 3 only

C 1, 2 and 4 only

D 1, 3 and 4 only

E 2, 3 and 4 only

F 1, 2, 3 and 4

© UCLES 2016
17

22 Calcium carbonate reacts with hydrochloric acid. The reaction gives off carbon dioxide gas.

Line X on the graph shows the volume of carbon dioxide formed against time when 100 cm3 of
1.0 mol dm–3 of hydrochloric acid reacts with calcium carbonate chips at 20°C. There was an
excess of calcium carbonate chips.

CaCO3 + 2HCl → CaCl2 + CO2 + H2O(l)

Which line best represents the volume of carbon dioxide formed against time when the reaction
is repeated with 50 cm3 of 2.0 mol dm–3 of hydrochloric acid reacting with excess calcium
carbonate chips at 20 °C?

A
volume of CO2

B
C
X

D
E

time

A line A

B line B

C line C

D line D

E line E

© UCLES 2016 [Turn over


18

23 An object that has a weight of 15 N on Earth is taken to a planet where it has a weight of 3.0 N.
The planet has no atmosphere.

Which line in the table correctly shows the mass of the object on the planet, and the kinetic
energy it gains after falling from rest near the surface of the planet through a vertical distance
of 10 m?

(Take the gravitational field strength g on Earth to be 10 N / kg.)

mass of object on kinetic energy after falling


planet (kg) 10 m on planet (J)
A 0.30 0.60

B 0.30 6.0

C 0.30 30

D 1.5 15

E 1.5 30

F 1.5 150

© UCLES 2016
19

24 Some blood is found at a crime scene. The police know that it belongs to the one criminal
involved.

A person’s red blood cells can have type A antigens, type B antigens, both types or neither
type.

In a population:

45% of people have type A antigens but not type B

9% of people have type B antigens but not type A

43% of people have neither type of antigen

3% of people have both types of antigen

An antibody test shows that there are type B antigens present in the red blood cells at the
crime scene.

What is the probability that the criminal’s red blood cells have both type A and type B
antigens?

3
A
100
1
B
16
3
C
25
1
D
4
3
E
4

© UCLES 2016 [Turn over


20

25 An experiment was carried out to investigate a gene for coat colour in mice.

The diagram shows the results of three crosses between different mice, producing three
different families, P, Q and R.

female male female male


X
X X
X XX

family Q

family P family R

female

female
female
female

What is the maximum possible number of heterozygous mice shown in the diagram?

A 2

B 3

C 4

D 5

E 6

F 7

G 8

H 12

© UCLES 2016
21

26 The gases X and Y react with each other to produce gas Z according to the equation:

2X(g) + Y(g) → 2Z(g)

100 cm3 of X was mixed with 10 cm3 of Y, in a freely moving gas syringe sealed with a rubber
cap. The reaction went to completion. All volumes were measured at the same temperature
and pressure.

gas syringe

rubber cap

What is the final volume of gas in the syringe?

A 20 cm3

B 55 cm3

C 80 cm3

D 100 cm3

E 110 cm3

F 120 cm3

2
27 Light travels through glass at a speed which is of its speed through air.
3

3
Light travels through water at a speed which is of its speed through air.
4

A ray of light has a wavelength of 360 nm when travelling through water.

What is the wavelength of this ray of light when travelling through glass?

A 180 nm

B 240 nm

C 320 nm

D 330 nm

E 390 nm

F 540 nm

G 720 nm
END OF TEST

© UCLES 2016
22

BLANK PAGE

© UCLES 2016
23

BLANK PAGE

© UCLES 2016
24

BLANK PAGE

© UCLES 2016
BioMedical Admissions Test – BMAT November 2016
SECTION 2
Answer key

Question Answer
1 D
2 F
3 D
4 A
5 B
6 E
7 D
8 B
9 F
10 F
11 B
12 D
13 B
14 C
15 D
16 C
17 G
18 E
19 D
20 C
21 A
22 B
23 E
24 D
25 H
26 D
27 C
BioMedical Admissions Test - Past Paper 2016
SECTION 2

Score Conversion

Total Mark BMAT Score


0 1.0
1 1.0
2 1.6
3 2.1
4 2.5
5 2.9
6 3.2
7 3.4
8 3.7
9 3.9
10 4.1
11 4.3
12 4.5
13 4.7
14 4.9
15 5.1
16 5.3
17 5.5
18 5.7
19 6.0
20 6.2
21 6.5
22 6.7
23 7.1
24 7.5
25 8.0
26 8.8
27 9.0

© UCLES 2016
BioMedical Admissions Test 4500/12

Wednesday 4th November 2015 30 minutes


 

SECTION 2 Scientific Knowledge and Applications

Instructions to Candidates

Please read this page carefully, but do not open the question paper until you are told
that you may do so.

A separate answer sheet is provided for this section. Please check you have one.
You also require a soft pencil and an eraser.

Please complete the answer sheet with your:

x BMAT candidate number


x Centre number
x date of birth
x name

Speed as well as accuracy is important in this section. Work quickly, or you may not
finish the paper. There are no penalties for incorrect responses, only points for correct
answers, so you should attempt all 27 questions. All questions are worth one mark.

Answer on the sheet provided. Questions ask you to show your choice between options
by shading a circle. If you make a mistake, erase thoroughly and try again.

Any rough work should be done on this question paper.

Calculators are NOT permitted.

Please wait to be told you may begin before turning this page.

This paper consists of 16 printed pages and 4 blank pages.

The question in this paper marked with an asterisk (* Q11) assumes


knowledge that is not currently on the BMAT specification.
© UCLES 2015
2

BLANK PAGE
3

1 Which one of the following is not part of a reflex to the stimulus of placing your hand on a hot
object?

A Sensory neuron transmits electrical impulse to the central nervous system.

B Muscle cells contract.

C Motor neuron transmits electrical impulse to muscle cells.

D Relay neurons pass electrical impulse from sensory to motor neurons.

E Brain transmits electrical impulse to relay neuron.

2 Which of the following organic compounds will decolourise bromine water by reacting with it at
room temperature?

1 ଶ ସ 

2 ‘Ž›’”‘’‡‡

3  ଶ ሺ ଷ ሻଶ

4  ଷ  ଶ 

A 1 only

B 2 only

C 1 and 3 only

D 2 and 4 only

E 3 and 4 only

F 1, 2 and 3 only

© UCLES 2015 [Turn over


4

3 The colour of the surface of an object has an effect on the rate of infrared radiation that it
absorbs and emits. As a result of this, some clothes which are otherwise identical will keep a
person warmer when outside in winter because of their differences in colour.

Which line in the table shows the correct comparison between black and white surfaces?

Better absorber of Better colour of clothes to keep


Better emitter of infrared
infrared a person warm in winter
A black black black

B black black white

C black white black

D black white white

E white black black

F white black white

G white white black

H white white white

4 A bag contains only 8 beads.

The beads are identical in all respects except colour.


3 of the beads are black and the other 5 beads are white.
A bead is taken at random from the bag and not replaced.
A second bead is then taken at random from the bag.

What is the probability that both beads are black?

͵
A
͵ʹ
͵
B
ʹͺ
ͻ
C
͸Ͷ
͵
D
ͳͶ
͵͹
E
ͷ͸
5

5 Which row identifies what is occurring during anaerobic respiration in animal cells?

Carbon dioxide formed Oxygen used Water formed

A no no no

B no no yes

C no yes no

D no yes yes

E yes no no

F yes no yes

G yes yes no

H yes yes yes

6 The diagram shows the suggested changes in energy labelled as a, b, c, d and e, during a
chemical change from reactants to products.

d
Products
Energy

c
a
Reactants e

Reaction progress

Which value is the correct energy change for the REVERSE reaction?

A –a

B d

C –c

D c–d

E e–b

© UCLES 2015 [Turn over


6

7 A 100% efficient step-down transformer decreases the voltage of an alternating current (a.c.)
electricity supply.

What effect does this step-down transformer have on the mean power transferred and the
alternating current delivered?

Power Current

A decreases decreases

B decreases does not change

C does not change decreases

D does not change increases

E increases does not change

F increases increases

8 PQR is an isosceles triangle in which ܲܳ ൌ ܴܲ ൌ ͸ cm and ܴܳ ൌ ͺ cm.

What is the value of the tangent of angle ܴܲܳ ?

ʹ
A
ξͳ͵
ʹ
B
ξͷ
ʹ
C
͵
͵
D
ʹ

E ξͷ
ʹ

F ξͳ͵
ʹ
7

9 The diagram shows the results of a breeding experiment using a homozygous black mouse and
a white mouse.
1

Mouse 1 was then allowed to mate with mouse 2. Using C for the dominant allele for coat colour
and c for the recessive allele, which answer below correctly identifies the details of their
offspring?

Details of offspring when mouse 1 and mouse 2 are mated


Percentage Phenotype(s) Genotype(s)
heterozygous (%)
A 100 black only all Cc

B 100 black and white all heterozygous

C 75 black only Cc and CC

D 50 black only homozygous and heterozygous

E 50 black and white CC, Cc and cc

F 50 black and white homozygous and heterozygous

G 0 black only all homozygous

10 Which of the following statements about the alkali metal rubidium is correct?

Rubidium and chlorine are formed when an aqueous solution of rubidium chloride is
A
electrolysed.

B Rubidium has higher melting and boiling points than sodium.

C Rubidium reacts more slowly with water than sodium, forming hydrogen gas.

D Rubidium is stored under oil.

E Rubidium sulfate has the formula „ସ

© UCLES 2015 [Turn over


8

* 11 Below are three statements about radioactivity or nuclear energy.

1 Neutrons emitted in nuclear fission can cause further fission.

2 The half-life of a radioactive substance is half the time taken for all its nuclei to
decay.
3 The process that produces heat and light in the Sun is called nuclear fission.

Which of these statements is/are correct?

A 1 only

B 2 only

C 3 only

D 1 and 2 only

E 1 and 3 only

F 2 and 3 only

G 1, 2 and 3

H None of them

12 Given that ܽ ൌ ଷ , ܾ ൌ ଷା௑ and ܿ ൌ ଷା௑ , where ܺ is a whole number greater than zero, which
ହା௑ ହ ହା௑
one of the following is true?

A ܽ ൏ ܾ ൏ ܿ for all values of ܺ

B ܽ ൏ ܿ ൏ ܾ for all values of ܺ

C ܾ ൏ ܽ ൏ ܿ for all values of ܺ

D ܾ ൏ ܿ ൏ ܽ for all values of ܺ

E ܿ ൏ ܽ ൏ ܾ for all values of ܺ

F ܿ ൏ ܾ ൏ ܽ for all values of ܺ

G The order of the fractions depends on the value of ܺ


9

13 A human consumes the same amount and type of food and drink on two consecutive days. He
also does the same activities on both days. However, one of the days is hot and the other is
cold.

Which row in the table correctly shows the mass of two substances found in the urine of this
human on the hot day compared with the cold day?

Mass of substance found in urine on a hot day compared with a cold day
water urea

A less less

B less same

C less more

D same same

E same more

F more less

G more same

H more more

© UCLES 2015 [Turn over


10

14 Cycloalkanes are alkane compounds that are in a ring shape.


Some examples of cycloalkanes are given below.

CH2 CH2
H2 C CH2 H2 C CH2
H2 C CH2

H2 C CH2 CH2 H2 C CH2


H2 C
CH2

Cyclohexane Cyclopentane Cyclobutane

Which statement about these compounds is correct?

A They have the general formula ୬ ଶ୬ାଶ

B They react rapidly with bromine water

C They are saturated compounds

D They burn in excess oxygen to form ଶ and ଶ

E They are not members of a homologous series

F They are giant covalent compounds


11

15 The diagram shows the only four forces acting on a model aircraft of mass 2.0 kg whilst flying.

lift from wings


25N

air resistance force from


(drag) 40N engine 50N

weight
20N

Which line in the table states the horizontal and vertical accelerations of the aircraft at this
instant?

Horizontal acceleration Vertical acceleration

A 5.0 m / s2 to the right 2.5 m / s2 upwards

B 5.0 m / s2 to the right 10 m / s2 downwards

C 5.0 m / s2 to the right zero

D 25 m / s2 to the right 10 m / s2 downwards

E 25 m / s2 to the right 2.5 m / s2 upwards

F 25 m / s2 to the right zero

G zero 2.5 m / s2 upwards

H zero 10 m / s2 downwards

© UCLES 2015 [Turn over


12

16 A city football club collected money for charity at all of its matches for a year. At the end of the
ଶ ସ
year the total collected was divided among three charities, A, B and C, in the ratio ͳ ‫׷‬ ‫׷‬ .
ଷ ହ
Charity C received £3 000.

What was the total amount collected for charity during the year?

A £3 750

B £7 000

C £8 250

D £9 000

E £9 250

F £12 375

17 The diagram shows part of the carbon cycle.

carbon dioxide in the air

1 4

carbon in plants carbon in decomposers

2 3
carbon in animals

Which row shows the numbered processes that use digestive or respiratory enzymes?

Process or processes that involve Process or processes that involve respiratory


digestive enzymes enzymes
A 1 only 2 and 3 only

B 2 only 1 and 4 only

C 3 only 2 and 4 only

D 4 only 2 and 3 only

E 2 and 3 only 1 only

F 3 and 4 only 2 only

G 1 and 4 only 3 only

H 2 and 3 only 4 only


13

18 The reaction between the gases S and T occurs according to the following equation.

ሺ‰ሻ ൅ ʹሺ‰ሻ ՜ ሺ‰ሻ ൅ ʹሺ‰ሻ ȟ ൌ  െͳʹ kJ mol–1

Which statement about this reaction is correct?

A A solid catalyst would slow down the rate of the reaction.

B If the chemical ‘T’ was a powder the reaction would be faster.

C A high activation energy would give a slower rate than a lower activation energy.

D Increasing the temperature would decrease the rate.

E The rate of reaction can be monitored by measuring the change in gas volume.

19 Part of a radioactivity decay series is represented below. It involves the change of a nucleus M
into a nucleus N by the emission of a beta-particle, followed by a further change into a nucleus
Q by the emission of an alpha particle. Four quantities V, W, X and Y are shown.

V β- α Y
W M ՜ XN ՜ Q

What are the expressions for X and Y?

X Y

A W–2 V–4

B W–2 V–2

C W–2 V

D W V–3

E W V–2

F W+1 V–3

G W+1 V–4

H W+1 V

© UCLES 2015 [Turn over


14

20 A class of ݊ pupils takes a spelling test. Their mean score for the test is ݉. Another pupil takes
the test and scores ݊. When this pupil’s result is included with the other results it is found that
the mean has decreased by 2.

Which equation below gives the correct expression for ݊ in terms of ݉?


݉
A ݊ൌ
݉െͳ
݉
B ݊ൌ
݉െ͵
ʹ
C ݊ൌ
݉െ͵
݉െʹ
D ݊ൌ
͵
݉൅ʹ
E ݊ൌ
݉െͳ
݉
F ݊ൌ
͵
݉൅ʹ
G ݊ൌ
͵

21 Mitochondria are the site of aerobic respiration in animal cells. A theory of the evolution of
animal cells states that these mitochondria may once have been aerobic bacteria that were
taken into the cytoplasm of a cell in an early ancestor of the animals, allowing the cells to gain
the ability to respire using oxygen.

Assuming this theory is correct, which of the following statements are true of these aerobic
bacteria and human white blood cells?

1 The structure of their DNA is a double helix.

2 They would both possess a cell wall.

3 They would both possess a nucleus.

4 They would both possess a cell membrane.

A 1 and 4 only

B 2 and 3 only

C 2 and 4 only

D 3 and 4 only

E 1, 2 and 3 only

F 1, 3 and 4 only
15

22 Which of the following have the same electron arrangement?


ଷହ ି ଷ଺ ା ସ଴ ଷଽ ା ସ଴ ା ସଵ ି
ଵ଻Ž ଵ଻Ž ଵ଼” ଵଽ ଶ଴ƒ ଵଽ 

A ଷଽ ା ସଵ ି ସ଴
ଵଽ , ଵଽ and ଵ଼” only

B ଷହ ି ଷ଺ ା ସ଴ ା
ଵ଻Ž , ଵ଻Ž and ଶ଴ƒ  only

C ଷହ ି ସ଴ ଷଽ ା
ଵ଻Ž , ଵ଼” and ଵଽ only

D ଷ଺ ା ସ଴ ସଵ ି
ଵ଻Ž , ଵ଼” and ଵଽ only

E ସ଴ ା ସ଴ ସଵ ି
ଶ଴ƒ , ଵ଼” and ଵଽ only

23 A car is being driven at 20 m / s when the driver sees a child run into the road. The driver’s usual
reaction time is 0.70 s, but this is doubled because the driver is tired. Once the driver applies the
brakes, the car is brought uniformly to rest in a further 3.3 s. What is the total distance travelled
by the car between when the driver first sees the child to when the car stops?

A 33 m

B 40 m

C 47 m

D 61 m

E 66 m

F 80 m

G 94 m

© UCLES 2015 [Turn over


16

24 Simplify:

ଶ௫ାଷ ଶ௫ିଷ
൅ െ ʹ
ଶ௫ିଷ ଶ௫ାଷ

A 0

ʹሺʹ‫ ݔ‬െ ͳሻ
B
ሺʹ‫ ݔ‬െ ͵ሻሺʹ‫ ݔ‬൅ ͵ሻ
ͳͺ
C
ሺʹ‫ ݔ‬െ ͵ሻሺʹ‫ ݔ‬൅ ͵ሻ
͵͸
D
ሺʹ‫ ݔ‬െ ͵ሻሺʹ‫ ݔ‬൅ ͵ሻ
ͺሺ‫ ݔ‬ଶ െ ʹሻ
E
ሺʹ‫ ݔ‬െ ͵ሻሺʹ‫ ݔ‬൅ ͵ሻ
ͳʹ
F
ʹ‫ ݔ‬െ ͵

25 The sex of a species of fruit fly is determined by the number of X chromosomes relative to the
number of non-sex chromosomes (A) in a cell. This is called the X : A ratio.

A fruit fly will be male if X : A = 0.5 : 1 and female if X : A = 1 : 1.

The Y chromosome contains genes necessary for making sperm.

Which row of the table correctly shows the sex of the five fruit flies with different numbers of
these chromosomes?

XAA XYAA XXAA XXYAA XXYYAA

A female female female male male

B female female male male male

C female male female male female

D female male female male male

E male female male female female

F male female male female male

G male male female female female

H male male female female male


17

26 The equation for the complete combustion of methane is:

 ସ ሺ‰ሻ ൅ ʹଶ ሺ‰ሻ ՜ ଶ ሺ‰ሻ ൅ ʹ ଶ ሺ‰ሻ

If 1.60 g of methane were completely burned in 8.00 g of oxygen (an excess) to produce 4.40 g
of carbon dioxide, what mass of oxygen is left unreacted?

(Ar: H = 1 ; C = 12 ; O = 16)

A 0.20 g

B 1.60 g

C 2.00 g

D 3.40 g

E 4.80 g

F 5.20 g

27 Consider the following three statements:

1 A mass of 4.0 kg requires a resultant force of 5.0 N to act on it in order to


accelerate it at 1.25 m / s2.
2 A wave of frequency 4.0 Hz travelling at a speed of 5.0 m / s has a wavelength of
1.25 m.
3 A voltage of 4.0 V applied across a 5.0 Ω resistor causes a current of 1.25 A to
flow.
Which statement(s) is / are true?
A 1 only

B 2 only

C 3 only

D 1 and 2 only

E 1 and 3 only

F 2 and 3 only

G 1, 2 and 3

H None of them

END OF TEST

© UCLES 2015 [Turn over


18

BLANK PAGE
19

BLANK PAGE
20

BLANK PAGE
BioMedical Admissions Test – Past Paper 2015

SECTION 2

Answer Key
Question Answer
1 E
2 C
3 B
4 B
5 A
6 A
7 D
8 E
9 D
10 D
11 A
12 B
13 B
14 C
15 A
16 E
17 H
18 C
19 G
20 D
21 A
22 C
23 D
24 D
25 G
26 B
27 D

© UCLES 2015
Score Conversion
Total Mark BMAT score
0 1.0
1 1.0
2 1.0
3 1.2
4 1.7
5 2.1
6 2.5
7 2.8
8 3.1
9 3.3
10 3.6
11 3.8
12 4.0
13 4.3
14 4.5
15 4.7
16 4.9
17 5.2
18 5.4
19 5.7
20 5.9
21 6.2
22 6.6
23 6.9
24 7.4
25 8.0
26 9.0
27 9.0

© UCLES 2015
BioMedical Admissions Test 4500/12

Wednesday 5th November 2014 30 minutes


 

SECTION 2 Scientific Knowledge and Applications

Instructions to Candidates

Please read this page carefully, but do not open the question paper until you are told
that you may do so.

A separate answer sheet is provided for this section. Please check you have one.
You also require a soft pencil and an eraser.

Please complete the answer sheet with your:

x BMAT candidate number


x centre number
x date of birth
x name

Speed as well as accuracy is important in this section. Work quickly, or you may not
finish the paper. There are no penalties for incorrect responses, only points for correct
answers, so you should attempt all 27 questions. All questions are worth one mark.

Answer on the sheet provided. Most questions ask you to show your choice between
options by shading a circle. If questions ask you to write in words or numbers, be sure to
write clearly in the spaces provided. If you make a mistake, erase thoroughly and try again.

Any rough work should be done on this question paper.

Calculators are NOT permitted.

Please wait to be told you may begin before turning this page.

This paper consists of 20 printed pages and 4 blank pages.

The question in this paper marked with an asterisk (* Q23) assumes


knowledge that is not currently on the BMAT specification.
© UCLES 2014
2

BLANK PAGE

© UCLES 2014
3

1 The following statements relate to the flow of blood through the heart.

1 Oxygenated blood flows through the right side of the heart.

2 The vena cava empties deoxygenated blood into the ventricle.

3 The heart pumps oxygenated blood through the pulmonary artery.

4 There is a valve that prevents backflow of blood from the aorta into the left
ventricle.

Which of these statements is / are correct?

A 1 only

B 2 only

C 4 only

D 1 and 3 only

E 1 and 4 only

F 1, 2 and 3 only

G 2, 3 and 4 only

2 Which of the following reactions are redox reactions?

1 —ସ ൅  ՜ — ൅ ସ

2 Ž ൅ ƒ  ՜ ƒŽ ൅ ଶ 

3 ‰ ൅ ଶ ସ  ՜ ‰ସ ൅ ଶ

4 „ሺଷ ሻଶ  ൅ ʹ ՜ „ ଶ ൅ ʹଷ

A 1 and 2 only

B 1 and 3 only

C 1 and 4 only

D 2 and 3 only

E 2 and 4 only

F 3 and 4 only

© UCLES 2014 [Turn over


4

3 Below are four statements about electromagnetic radiation.

1 Microwaves have a shorter wavelength than all other electromagnetic waves.

2 For identical amplitudes, waves with the largest wavelength transfer the most
energy.
3 The speed of electromagnetic waves is inversely proportional to their frequency.

4 Ultraviolet radiation can cause cataracts.

Which of these statements is / are correct?

A 1 only

B 2 only

C 3 only

D 4 only

E 1 and 3 only

F 1 and 4 only

G 2 and 3 only

H 2 and 4 only

4 Simplify:

௫ మ ିସ௫
௫ మ ିଵ଺

‫ݔ‬
A
Ͷ
‫ݔ‬
B
‫ݔ‬൅Ͷ
‫ݔ‬
C
‫ݔ‬െͶ
ͳ
D
Ͷ
‫ݔ‬െͶ
E
‫ ݔ‬െ ͳ͸

© UCLES 2014
5

5 Before a cell can divide by mitosis, DNA synthesis has to take place. Following DNA synthesis,
the DNA is separated into each half of the cell and then the cell divides.

The graph below shows the DNA content per cell over a period of time.

Which of the letters on the graph represent the sequence of the three events described above?

Cell divides DNA synthesis DNA separates

A J K L

B J L M

C K L M

D K M N

E L M N

F L M J

G M N J

H M N K

© UCLES 2014 [Turn over


6

6 Which row in the table correctly explains why an increase in temperature increases the rate of a
reaction?

Effect on collision Effect on proportion of


Effect on activation energy
frequency between collisions which are
of reaction
particles successful

A decreases no effect increases

B increases no effect no effect

C no effect increases no effect

D increases increases no effect

E decreases no effect no effect

F no effect no effect increases

G decreases increases increases

H no effect increases increases

7 Which one of the following is a unit of current?

A ampere / second

B coulomb × second

C joule / coulomb

D ohm / volt

E volt × ohm

F watt / volt

8 Given that Ͷ௣ ൈ ͺ௤ ൌ ʹ௡ , express ݊ in terms of ‫ ݌‬and ‫ݍ‬.

A ݊ ൌ‫݌‬൅‫ݍ‬

B ݊ ൌ ʹ‫ ݌‬൅ ͵‫ݍ‬

C ݊ ൌ ʹ‫ ݌‬൅ Ͷ‫ݍ‬

D ݊ ൌ‫݌‬൅‫ݍ‬൅ͷ

E ݊ ൌ ͸‫ݍ݌‬

© UCLES 2014
7

9 Insulin is a protein involved in the regulation of human blood glucose levels.

Genetic engineering can be used to allow the large-scale production of human insulin.

Which statement describes the process of genetic engineering in this case?

Taking insulin from a human and inserting it into the DNA of a bacterium. As the
A bacterium reproduces, it makes large quantities of insulin DNA that can be used to
treat human diabetes.
Taking insulin from a human and inserting it into the DNA of a bacterium. As the
B bacterium reproduces, it makes large quantities of insulin that can be used to treat
human diabetes.
Taking the insulin gene from a human chromosome and inserting it into the DNA of a
C bacterium. As the bacterium reproduces, it makes large quantities of insulin DNA that
can be used to treat human diabetes.
Taking the insulin gene from a human chromosome and inserting it into the DNA of a
D bacterium. As the bacterium reproduces, it makes large quantities of insulin that can
be used to treat human diabetes.
Taking the insulin gene from a human chromosome and replacing it in another human
E chromosome in the same human, so that it will work better to produce large quantities
of insulin.

10 Methanol can be oxidised by hydrogen peroxide to produce carbon dioxide and water.

‫ ܉‬ଷ  ൅ ͵ ଶ ଶ ՜ ଶ ൅ ‫ ܊‬ଶ 

What is the value of ‫ ܊‬when this equation is balanced?

A 3

B 4

C 5

D 6

E 7

© UCLES 2014 [Turn over


8

11 Two rods, X and Y, are made from different electrically insulating materials. A student rubs
rod X, which is initially uncharged, with a cloth, then holds it near to rod Y. The two rods repel
each other.

Which statement explains why repulsion occurs in this experiment?

A Rod X gains electrons from the cloth and rod Y is positively charged.

B Rod X gains electrons from the cloth and rod Y is uncharged.

C Rod X gains protons from the cloth and rod Y is negatively charged.

D Rod X gains protons from the cloth and rod Y is positively charged.

E Rod X loses electrons to the cloth and rod Y is negatively charged.

F Rod X loses electrons to the cloth and rod Y is positively charged.

G Rod X loses protons to the cloth and rod Y is negatively charged.

H Rod X loses protons to the cloth and rod Y is uncharged.

© UCLES 2014
9

12 In a town, the bearing of the library from the station is ‫ ל ݔ‬.

The theatre is due east of the library.

The theatre and the station are equidistant from the library.

What is the bearing (in degrees) of the station from the theatre?
‫ݔ‬
A Ͷͷ ൅  ቀ ቁ
ʹ
‫ݔ‬
B ͻͲ െ  ቀ ቁ
ʹ
‫ݔ‬
C ͻͲ ൅  ቀ ቁ
ʹ
D ͳͺͲ ൅ ‫ݔ‬
‫ݔ‬
E ʹʹͷ െ  ቀ ቁ
ʹ
‫ݔ‬
F ʹʹͷ ൅  ቀ ቁ
ʹ
‫ݔ‬
G ʹ͹Ͳ Ȃቀ ቁ
ʹ

© UCLES 2014 [Turn over


10

13 In humans, the water content in the blood is regulated via the hormonal system.

Which of the following occur when the water content of the blood is too low?
1 pituitary gland releases less ADH

2 pituitary gland releases more ADH

3 increase in water reabsorption by the kidneys

4 decrease in water reabsorption by the kidneys

5 increased reabsorption of glucose in the kidneys

6 decreased reabsorption of glucose in the kidneys

A 1 and 3 only

B 2 and 3 only

C 1 and 4 only

D 2 and 4 only

E 1, 3 and 5 only

F 2, 4 and 6 only

G 1, 4 and 5 only

H 2, 3 and 6 only

© UCLES 2014
11

14 Which of the following statements about but-1-ene are true?

1 it can form a polymer

2 it contains Ȃ single bonds only

3 it conforms to the general formula ୬ ଶ୬

4 it decolourises bromine water

5 it is saturated

A 1, 2 and 3 only

B 1, 2 and 5 only

C 1, 3 and 4 only

D 1, 3 and 5 only

E 2, 3 and 5 only

F 2, 3 and 4 only

G 2, 4 and 5 only

H 3, 4 and 5 only

© UCLES 2014 [Turn over


12

15 A container is filled with water at 20 °C and placed in a room that is also at 20 °C. The container
can be fitted with an internal electric cooling unit, in one of the three labelled positions P, Q or R.
The outside of the container can either be painted dull black, or be covered in shiny aluminium
foil.

In order to cool all the water as quickly as possible to 5 °C, in which position should the cooling
unit be fitted, and should the outside of the container be dull black or shiny?

Position of cooling unit Outside of container

A P black

B P shiny

C Q black

D Q shiny

E R black

F R shiny

© UCLES 2014
13

16 Three classes in a school all took the same test. Class 1 achieved a mean score of 61, Class 2
achieved a mean score of 63, and class 3 achieved a mean score of 70. The mean score of the
students for all three classes combined was 65. Class 1 contains twice as many students as
Class 2.

Which one of the following statements about the number of students in class 3 is true?

A Class 3 contains fewer students than Class 2.

B Class 3 contains the same number of students as Class 2.

C Class 3 contains more students than Class 2, but fewer than Class 1.

D Class 3 contains the same number of students as Class 1.

E Class 3 contains more students than Class 1.

17 Which of the following statements about lipid digestion in the small intestine is / are correct?

1 Emulsification by bile makes smaller lipid droplets, each with a smaller surface
area.
2 Bile contains an alkali to reduce the pH of the material from the stomach.

3 Lipase secreted in bile breaks bonds in lipids to produce glycerol and fatty acids.

A none of the statements

B 1 only

C 2 only

D 3 only

E 1 and 2 only

F 2 and 3 only

G 1 and 3 only

H 1, 2 and 3

© UCLES 2014 [Turn over


14

18 An organic compound is found to contain 6 parts of carbon, 1 part of hydrogen and 8 parts of
oxygen by mass.

6 g of a gaseous sample of the compound would have a volume of 2.4 dm3 at room temperature
and pressure.

Which formula (A–E) is the molecular formula for this compound?

(Ar: H = 1; C = 12; O = 16)

(1 mole of any gas occupies 24 dm3 at room temperature and pressure)

A  ଶ 

B ଶ ସ ଶ

C ଷ ଼ 

D ଷ ଺ ଷ

E ଺ ଼

19 The displacement / time graph shown represents a wave of wavelength 1.5 cm.


What is the speed of the wave?

A 0.33 cm / s

B 0.67 cm / s

C 0.75 cm / s

D 1.33 cm / s

E 1.5 cm / s

F 3.0 cm / s

© UCLES 2014
15

20 The diagram shows part of a glass structure. PQRS is a horizontal square with sides of 1 metre,
and point X is 4 metres vertically above P.

What is the cosine of the angle that XR makes with the horizontal?

ͳ
A
͵

B ξʹ
Ͷ
ξʹ
C
ʹξ͵
Ͷ
D
͵ξʹ
ʹ
E
ξ͵
ͳ
F
ξͳ͹
ͳ
G
ͳ͹

© UCLES 2014 [Turn over


16

21 In order to function, the cells of the brain need large amounts of energy. Any reduction in the
function of these cells can have serious consequences for the body. For example, a loss of
oxygen supply to the brain can cause unconsciousness within 5–10 seconds.

Which of the following statements are true?


1 Neurons in the brain are capable of relying on anaerobic respiration for long
periods of time.
2 The homeostatic systems of the body will constantly have to work to resist
temperature increases in the brain.
3 An overdose of insulin in the body could produce a serious loss of brain function,
such as inducing a coma.
4 During normal functioning of neurons in the brain, high levels of carbon dioxide
could be produced.

A 1 and 2 only

B 1 and 3 only

C 2 and 3 only

D 2 and 4 only

E 1, 3 and 4 only

F 1, 2 and 4 only

G 2, 3 and 4 only

© UCLES 2014
17

22 Graphene is a new material composed of carbon atoms arranged in tightly bound hexagons just
one atom thick.

The diagram shows a simplified structure of graphene.

Considering its structure, which of the properties below could be predicted about graphene?
1 high melting point

2 good electrical conductivity

3 soluble in water

A 1 only

B 2 only

C 3 only

D 1 and 2 only

E 1 and 3 only

F 2 and 3 only

G 1, 2 and 3

© UCLES 2014 [Turn over


18

* 23 When a particular nucleus of uranium-235 undergoes the process of nuclear fission, it absorbs
a neutron and then splits into a nucleus of barium and a nucleus of krypton, as well as releasing
further neutrons.

Which one of the rows of the table below gives the correct number of neutrons released and the
isotopes of barium and krypton produced by this nuclear reaction?

Number of neutrons Mass number of barium Mass number of krypton


released isotope produced isotope produced

A 2 141 92

B 2 142 94

C 3 140 92

D 3 140 94

E 3 141 92

F 3 142 94

© UCLES 2014
19

24 A test is developed to detect a certain medical condition. The test is not perfect, and sometimes
gives incorrect results. The behaviour of the test on 1000 randomly selected members of the
population is shown in this tree diagram, where the following notation is used:

x ‫ ܥ‬ൌ has the condition


x ‫ ܥ‬ᇱ ൌ does not have the condition
x ܶ ൌ tests positive for the condition
x ܶԢ ൌ tests negative for the condition

Three of the branches’ proportions are shown in the tree diagram:



x of the 1000 people have the condition
ଵ଴଴


x of those with the condition test positive for the condition


x of those without the condition test positive for the condition
ଵ଴

A person is selected at random from these 1000 people, and tests positive for the condition.

What is the probability that this person has the condition?

Ͷ
A
ͷ
ͻ
B
ͳͲ
ͻ
C
ʹͲ
ͺ
D
ͻͻ
ͺ
E
ͳͲ͹
Ͷ
F
ͷͲͲ
ͳͲ͹
G
ͳͲͲͲ

© UCLES 2014 [Turn over


20

25 A woman has a recessive genetic condition but neither of her parents has the condition.

Which one of the following could not be true?

A Both her parents are heterozygous for this gene.

B One maternal grandparent and one paternal grandparent have the condition.
One maternal grandparent and one paternal grandparent are heterozygous for this
C
gene.

D All her grandparents were carriers of the recessive allele.


Both parents are homozygous and a mutation occurred in the DNA of a gamete from
E
one of her parents.

26 Which of the following atoms / ions contain(s) exactly 18 electrons?

1 ͳͺ
ͺ

2 ͵Ͷ
ͳ͸

3 ͵ͷ
ͳ͹Ž

4 ͵͹ Ȃ
ͳ͹Ž 

5 ͶͲ ଶା
ʹͲƒ 

A 1 only

B 2 only

C 2 and 3 only

D 3 and 4 only

E 4 and 5 only

© UCLES 2014
21

27 The graph shows the variation with time of the height through which a crane lifts a mass of
20 kg.

Assume the gravitational field strength g is 10 N / kg, and that the effects of air resistance and
friction are negligible.

What is the power output of the crane when the mass is at a height of 10 m?

A 0.1 W

B 10 W

C 40 W

D 100 W

E 400 W

F 4000 W

© UCLES 2014
22

BLANK PAGE

© UCLES 2014
23

BLANK PAGE

© UCLES 2014
24

BLANK PAGE

© UCLES 2014
BioMedical Admissions Test – Past Paper 2014

SECTION 2

Answer Key
Question Answer
1 C
2 B
3 D
4 B
5 D
6 H
7 F
8 B
9 D
10 C
11 F
12 F
13 B
14 C
15 B
16 D
17 B
18 B
19 C
20 A
21 G
22 D
23 E
24 E
25 E
26 E
27 D

© UCLES 2014
Score Conversion
Total Mark BMAT score
0 1.0
1 1.0
2 1.0
3 1.6
4 2.0
5 2.4
6 2.7
7 3.0
8 3.2
9 3.5
10 3.7
11 4.0
12 4.2
13 4.4
14 4.6
15 4.9
16 5.1
17 5.4
18 5.7
19 5.9
20 6.3
21 6.6
22 6.9
23 7.4
24 7.9
25 8.5
26 9.0
27 9.0

© UCLES 2014
BioMedical Admissions Test 4500/12

Wednesday 6th November 2013 30 minutes


 

SECTION 2 Scientific Knowledge and Applications

Instructions to Candidates

Please read this page carefully, but do not open the question paper until you are told
that you may do so.

A separate answer sheet is provided for this section. Please check you have one.
You also require a soft pencil and an eraser.

Please complete the answer sheet with your:

x BMAT candidate number


x centre number
x date of birth
x name

Speed as well as accuracy is important in this section. Work quickly, or you may not
finish the paper. There are no penalties for incorrect responses, only points for correct
answers, so you should attempt all 27 questions. All questions are worth one mark.

Answer on the sheet provided. Most questions ask you to show your choice between options
by shading a circle. If questions ask you to write in words or numbers, be sure to write
clearly in the spaces provided. If you make a mistake, erase thoroughly and try again.

Any rough work should be done on this question paper.

Calculators are NOT permitted.

Please wait to be told you may begin before turning this page.

This paper consists of 16 printed pages and 4 blank pages.

The questions in this paper that are marked with an asterisk (* Qs: 2, 11, 17, 21)
assume knowledge that is not currently on the BMAT specification.

© UCLES 2013
2






















BLANK PAGE

‹UCLES 2013
3

1 :KLFKRIWKHIROORZLQJDSSOLHVWRERWKWKHQHUYRXVV\VWHPDQGWKHHQGRFULQH KRUPRQDO V\VWHP"

 FDQEHLQYROYHGLQKRPHRVWDVLV

 FDQLQYROYHFKHPLFDOV

 FDQLQYROYHWKHEUDLQ

A 1RQHRIWKHP

B RQO\

C RQO\

D RQO\

E DQGRQO\

F DQGRQO\

G DQGRQO\

H DQG

* 2 7KHIROORZLQJFKHPLFDOVDUHPL[HGWRJHWKHU,QZKLFKRIWKHPL[WXUHVZLOODGLVSODFHPHQWUHDFWLRQ
RFFXU"

 3E 12 DT $O V

Ϝ±
 .&O DT ) DT 

 $O 62  DT )H V

 &X62 DT =Q V

A RQO\

B DQGRQO\

C DQGRQO\

D DQGRQO\

E DQGRQO\

F DQGRQO\

‹UCLES 2013 >7XUQ2YHU


4

3 0LFURZDYH;UD\DQGLQIUDUHGUDGLDWLRQFDQDOOGDPDJHOLYLQJWLVVXHV

:KLFKRIWKHIROORZLQJVWDWHPHQWVFRUUHFWO\H[SODLQZK\WKLVGDPDJHRFFXUV"

 0LFURZDYHVFDXVHGDPDJHEHFDXVHWKH\DUHDEVRUEHGE\ZDWHUPROHFXOHV

 ;UD\VFDXVHGDPDJHEHFDXVHRIWKHLULRQLVLQJDELOLW\

 ,QIUDUHGZDYHVFDXVHGDPDJHEHFDXVHRIWKHLUDELOLW\WRSHQHWUDWHPDWWHU

A RQO\

B RQO\

C RQO\

D DQGRQO\

E DQGRQO\

F DQGRQO\

G DQG

4


‹UCLES 2013
5

5 7KHDFWLRQRIZKLFKHQ]\PH V FRXOGGHFUHDVHWKHS+RIDPL[WXUHRIFDUERK\GUDWHSURWHLQDQG
OLSLG"

 &DUERK\GUDVH

 3URWHDVH

 /LSDVH

A RQO\

B RQO\

C RQO\

D RURQO\

E RURQO\

F RURQO\

G RU

6 7KHIROORZLQJUHDFWLRQLVDWHTXLOLEULXP


5 J 6 J 7 J '+LVQHJDWLYH

ZKLFKFRQGLWLRQV $±( ZRXOGSURGXFHPRVWRIWKHSURGXFW7"

 Temperature  Pressure  R  S Catalyst 


A KLJK ORZ DGG UHPRYH SUHVHQW
B ORZ KLJK DGG DGG DEVHQW
C ORZ ORZ DGG DGG DEVHQW
D KLJK KLJK UHPRYH DGG SUHVHQW
E ORZ KLJK UHPRYH UHPRYH DEVHQW

‹UCLES 2013 >7XUQ2YHU


6

7 7KHGLDJUDPVKRZVWKUHHDSSURSULDWHPHWHUV34DQG5FRQQHFWHGLQWKHFRQYHQWLRQDOZD\LQD
FLUFXLW7KHVZLWFKLVLQLWLDOO\RSHQ



7KHVZLWFKLVQRZFORVHG:KDWKDSSHQVWRWKHUHDGLQJVRQHDFKPHWHU"

A 3GHFUHDVHV4GHFUHDVHV5GHFUHDVHV

B 3GHFUHDVHV4GHFUHDVHV5LQFUHDVHV

C 3GHFUHDVHV4LQFUHDVHV5GHFUHDVHV

D 3GHFUHDVHV4LQFUHDVHV5LQFUHDVHV

E 3LQFUHDVHV4GHFUHDVHV5GHFUHDVHV

F 3LQFUHDVHV4LQFUHDVHV5LQFUHDVHV

G 3LQFUHDVHV4GHFUHDVHV5LQFUHDVHV

H 3LQFUHDVHV4LQFUHDVHV5GHFUHDVHV

‹UCLES 2013
7

8 6LPSOLI\



9 ,QDUHIOH[DFWLRQLQZKLFKDSHUVRQWRXFKHVDKRWSODWHDQGSXOOVWKHLUDUPDZD\QHXURQVRI
GLIIHUHQWOHQJWKVDUHLQYROYHG:KLFKDQVZHULGHQWLILHVWKHUHODWLYHOHQJWKVRIWKHQHXURQV"

 Length of neuron
Longest Medium Shortest
A VHQVRU\ UHOD\ PRWRU
B PRWRU VHQVRU\ UHOD\
C UHOD\ PRWRU VHQVRU\
D PRWRU UHOD\ VHQVRU\
E UHOD\ VHQVRU\ PRWRU
F VHQVRU\ PRWRU UHOD\

‹UCLES 2013 >7XUQ2YHU


8

10 JRIVRGLXPFRPSOHWHO\UHDFWVZLWKZDWHUDWVWDQGDUGWHPSHUDWXUHDQGSUHVVXUH 673 :KDW


YROXPHRIK\GURJHQDW673LVSURGXFHGE\WKLVUHDFWLRQ"

>$VVXPHLQWKLVTXHVWLRQWKDWPROHRIDQ\JDVDW673KDVDYROXPHRIGP OLWUHV @

>$UYDOXHV+ 2 1D @


A FP

B FP

C FP

D FP

E FP

* 11 7KHGLDJUDPVVKRZWZRJODVVEORFNVLQDLU)RUWKHUD\VRIOLJKWVKRZQWKHFULWLFDODQJOHIRUWKH
JODVVDLUERXQGDU\LVƒ$UD\RIOLJKWLVVKRZQDSSURDFKLQJWKHERXQGDU\LQHDFKFDVHZLWKWKH
DQJOHRILQFLGHQFHODEHOOHG7ZRSRVVLEOHODEHOOHGGLUHFWLRQVLQZKLFKHDFKUD\PLJKWWUDYHODIWHU
UHDFKLQJWKHERXQGDU\DUHDOVRVKRZQ




:KLFKOLQHLQWKHWDEOHFRUUHFWO\VKRZVWKHGLUHFWLRQWDNHQE\PRVWRUDOORIWKHOLJKWDQGDOVRVWDWHV
ZKHWKHUWRWDOLQWHUQDOUHIOHFWLRQ 7,5 ZLOORFFXU"

 Diagram 1 Diagram 2


Direction T.I.R.? Direction T.I.R.?
A 3 1R 5 1R
B 3 1R 5 <HV
C 3 1R 6 1R
D 4 1R 6 1R
E 4 <HV 5 <HV
F 4 <HV 6 <HV

‹UCLES 2013
9

12 7KHVTXDUH$%&'LVSRVLWLRQHGVRWKDWLWVYHUWLFHVDUHDWWKHSRLQWVZLWKFRRUGLQDWHV

     DQG  



RDERXWWKHRULJLQDQGWKHQUHIOHFWHGLQWKHOLQH\
7KHVTXDUHLVURWDWHGFORFNZLVHWKURXJK [

:KLFKWUDQVIRUPDWLRQZLOOUHWXUQWKHVTXDUHWRLWVRULJLQDORULHQWDWLRQ"

A $UHIOHFWLRQLQWKH[D[LV

B $UHIOHFWLRQLQWKH\D[LV

C $UHIOHFWLRQLQWKHOLQH\ ±[

D $URWDWLRQRIRFORFNZLVHDERXWWKHRULJLQ

E $URWDWLRQRIRDQWLFORFNZLVHDERXWWKHRULJLQ

13 :KLFKRIWKHIROORZLQJLVnotQHHGHGLQRUGHUWRJHQHWLFDOO\HQJLQHHUEDFWHULDOFHOOVWRSURGXFHD
IOXRUHVFHQWSURWHLQIURPDMHOO\ILVK"

A OLJDVHHQ]\PH

B DSODVPLGRUYLUDOYHFWRU

C IOXRUHVFHQWSURWHLQIURPDMHOO\ILVK

D HQ]\PHVWRFXW'1$PROHFXOHV

‹UCLES 2013 >7XUQ2YHU


10

14 :KLFKRQHRIWKHIROORZLQJSDLUVGRnotKDYHWKHVDPHHOHFWURQLFVWUXFWXUH"




A 0J&ODQGWKUHHDWRPVRIDUJRQ

B &2DQG1

C &+DQG1+

D 12±DQG&2±

E 1D)DQGWZRDWRPVRI1H

15 7ZRUDGLRDFWLYHVRXUFHV;DQG<KDYHKDOIOLYHVRIKRXUVDQGKRXUVUHVSHFWLYHO\%RWK
GHFD\GLUHFWO\WRIRUPRQO\VWDEOHLVRWRSHV

7KHDFWLYLW\RIDVDPSOHRIWKHVRXUFH;ZDVPHDVXUHGE\DGHWHFWRUDVFRXQWVSHUPLQXWH
DQGVLPXOWDQHRXVO\WKHUDGLRDFWLYLW\RIDVDPSOHRIWKHVRXUFH<ZDVPHDVXUHGDVFRXQWVSHU
PLQXWH,PPHGLDWHO\DIWHUWKHPHDVXUHPHQWVWKHWZRVDPSOHVZHUHFRPELQHG

:KDWZDVWKHFRXQWUDWHZKHQWKHDFWLYLW\RIWKHFRPELQDWLRQRI;DQG<ZDVPHDVXUHGKRXUV
ODWHU"

>$VVXPHWKDWDOOUHDGLQJVLQWKLVTXHVWLRQKDYHEHHQFRUUHFWHGIRUEDFNJURXQGUDGLDWLRQ@

A FRXQWVSHUPLQXWH

B FRXQWVSHUPLQXWH

C FRXQWVSHUPLQXWH

D FRXQWVSHUPLQXWH

E FRXQWVSHUPLQXWH

F FRXQWVSHUPLQXWH

‹UCLES 2013
11

16 7KUHHYDULDEOHV[\DQG]DUHNQRZQWREHUHODWHGWRHDFKRWKHULQWKHIROORZLQJZD\V

z [LVGLUHFWO\SURSRUWLRQDOWRWKHVTXDUHRI]

z \LVLQYHUVHO\SURSRUWLRQDOWRWKHFXEHRI]

:KLFKRIWKHIROORZLQJFRUUHFWO\GHVFULEHVWKHUHODWLRQVKLSEHWZHHQ[DQG\"

A 7KHVTXDUHRI[LVGLUHFWO\SURSRUWLRQDOWRWKHFXEHRI\

B 7KHVTXDUHRI[LVLQYHUVHO\SURSRUWLRQDOWRWKHFXEHRI\

C 7KHFXEHRI[LVGLUHFWO\SURSRUWLRQDOWRWKHVTXDUHRI\

D 7KHFXEHRI[LVLQYHUVHO\SURSRUWLRQDOWRWKHVTXDUHRI\

E [LVGLUHFWO\SURSRUWLRQDOWR\

* 17 6LQFH'ROO\WKHVKHHSPDQ\RWKHUPDPPDOVKDYHDOVREHHQFORQHGE\VRPDWLFFHOOQXFOHDU
WUDQVIHU7KHJHQHWLFPDWHULDOIURPDERG\FHOOLVLQVHUWHGLQWRDQHJJFHOOWKDWKDVKDGLWVRZQ
QXFOHXVUHPRYHG7KHVXFFHVVUDWHUDQJHVIURPWRZKLFKLVZK\VRIHZFORQHGDQLPDOV
KDYHEHHQSURGXFHG

:KLFKRIWKHIROORZLQJDUHSRVVLEOHFRUUHFWUHDVRQVZK\FORQLQJPD\IDLO"

 $QHJJZLWKDQHZO\WUDQVIHUUHGQXFOHXVPD\QRWEHJLQWRGLYLGHRUGHYHORSSURSHUO\

 7KHVSHUPFHOOPD\QRWIHUWLOLVHWKHHJJFHOO

 ,PSODQWDWLRQRIWKHHPEU\RLQWRWKHVXUURJDWHPRWKHUPLJKWIDLO

 ,PSODQWHGVWHPFHOOVPD\QRWGLIIHUHQWLDWHSURSHUO\

 7KHHQXFOHDWHGHJJDQGWKHWUDQVIHUUHGQXFOHXVPD\QRWEHFRPSDWLEOH


A DQGRQO\

B DQGRQO\

C DQGRQO\

D DQGRQO\

E DQGRQO\

F DQGRQO\

‹UCLES 2013 >7XUQ2YHU


12

18 $QLPSXUHVDPSOHRIVRGLXPK\GUR[LGHKDVDPDVVRIJ$OOWKHVRGLXPK\GUR[LGHFRPSOHWHO\
UHDFWVZLWKDPLQLPXPRIFPRIPROGPK\GURFKORULFDFLG

:KDWLVWKHSHUFHQWDJHSXULW\RIWKHVRGLXPK\GUR[LGHVDPSOH"

>$UYDOXHV+ 2 1D &O @

A 

B 

C 

D 

E 

F 

19 7ZRUHVLVWRUVRI5RKPVDQG5RKPVDUHFRQQHFWHGLQVHULHVWRDEDWWHU\ZKLFKKDVDQHPIRI
9

:KLFKIRUPXODJLYHVWKHSRZHUGLVVLSDWHGE\UHVLVWRU5"

‹UCLES 2013
13

20 $VROLGVKDSHLVPDGHE\MRLQLQJWKUHHFXEHVWRJHWKHUZLWKWKHODUJHVWFXEHRQWKHERWWRPDQGWKH
VPDOOHVWRQWKHWRS:KHUHWKHIDFHVRIWZRFXEHVMRLQWKHFRUQHUVRIWKHVPDOOHUFXEHDUHDWWKH
PLGSRLQWVRIWKHVLGHVRIWKHODUJHUFXEH



7KHVLGHVRIWKHVPDOOHVWFXEHKDYHDOHQJWKRIFP:KDWLVWKHWRWDOVXUIDFHDUHDRIWKHVKDSH"

A FP

B FP

C FP

D FP

E FP

F FP

‹UCLES 2013 >7XUQ2YHU


14

* 21 :KLFKRIWKHIROORZLQJFRXOGEHIRXQGLQDQDGXOWOLYHUFHOO"

 JHQHIRUDP\ODVH

 VH[FKURPRVRPHV

 VWDUFK

A 1RQHRIWKHP

B RQO\

C RQO\

D RQO\

E DQGRQO\

F DQGRQO\

G DQGRQO\

H DQG

22 %\XVLQJVWDQGDUGWHFKQLTXHVWREDODQFHFKHPLFDOHTXDWLRQVDQGHQVXULQJWKDWWKHQHWFKDUJHLV
HTXDORQERWKVLGHVILQGWKHFRUUHFWYDOXHIRUµ H ¶LQWKHEDODQFHGHTXDWLRQEHORZ



A 

B 

C 

D 

E 

‹UCLES 2013
15

23 &RQVLGHUWKLVJUDSK



:KLFKRQHRIWKHIROORZLQJFRXOGWKHJUDSKnotUHSUHVHQWLIDOOTXDQWLWLHVDUHLQ6,XQLWV"

A 7KHYDULDWLRQRIWKHDFFHOHUDWLRQ \D[LV RIDERG\RIPDVVNJZLWKWKHUHVXOWDQWIRUFH


DFWLQJRQWKHERG\ [D[LV 

B 7KHYDULDWLRQRIWKHFXUUHQW \D[LV WKURXJKDȍUHVLVWRUZLWKWKHDSSOLHGYROWDJH [D[LV 

C 7KHYDULDWLRQRIWKHNLQHWLFHQHUJ\ \D[LV RIDERG\RIPDVVNJZLWKWKHVTXDUHRILWV


VSHHG [D[LV 

D 7KHYDULDWLRQRIWKHZDYHOHQJWK \D[LV RIZDYHVZLWKDVSHHGRIPVZLWKWKHLUIUHTXHQF\


[D[LV 

E 7KHYDULDWLRQRIWKHZRUNGRQH \D[LV E\DIRUFHRI1ZLWKWKHGLVWDQFHLWPRYHVWKURXJK


[D[LV 

‹UCLES 2013 >7XUQ2YHU


16

24 $EDJFRQWDLQVEOXHDQGUHGEDOOV7KUHHEDOOVDUHFKRVHQDWWKHVDPHWLPHDWUDQGRPIURPWKH
EDJ

:KDWLVWKHSUREDELOLW\WKDWH[DFWO\WZRRIWKHEDOOVDUHWKHVDPHFRORXU"

25 0DQ[FDWVZLWKWZRUHFHVVLYHDOOHOHVKDYHDWDLO+HWHUR]\JRXV0DQ[FDWVODFNDWDLO,QGLYLGXDOVZLWK
ERWKGRPLQDQWDOOHOHVGLHEHIRUHELUWK

:KLFKDQVZHUVKRZVWKHSHUFHQWDJHRI0DQ[FDWVZLWKRXWWDLOVLQDSRSXODWLRQIRUWKHWZRFURVVHV
JLYHQLQWKHWDEOH"

 Manx cat with a tail Manx cat without a tail


crossed with a crossed with a
Manx cat without a tail Manx cat without a tail
A  
B  
C  
D  
E  
F  
G  

‹UCLES 2013
17

26 7KHDGGLWLRQRI12DVDFDWDO\VWWRDPL[WXUHRI62DQG2VSHHGVXSWKHIROORZLQJUHDFWLRQ

62ò2ĺ62

7KHIROORZLQJUHDFWLRQVFRXOGEHLQYROYHGLQWKHSURFHVV

 12ĺ12

 ò12ĺ12

 12ò2ĺ12

 12ĺ12ò2

 6212ĺ62ò1

 6212ĺ6212

:KLFKRQHRIWKHIROORZLQJVKRZVWKHPRVWOLNHO\FRXUVHRIWKHRYHUDOOUHDFWLRQ"

A 

B 

C 

D 

E 

27 $UHVXOWDQWIRUFHRI1KDVDFFHOHUDWHGDERG\RIPDVVNJIURPUHVWXQWLOWKHSUHVHQW
PRPHQWDWZKLFKWLPHLWVNLQHWLFHQHUJ\LV-

,IWKLVIRUFHFRQWLQXHVWRDFWXQFKDQJHGKRZPXFKH[WUDNLQHWLFHQHUJ\ZLOOWKHERG\JDLQGXULQJWKH
QH[WVHFRQGV"

A -

B -

C -

D -

E -

‹UCLES 2013 >7XUQ2YHU


18






















BLANK PAGE

‹UCLES 2013
19






















BLANK PAGE

‹UCLES 2013 >7XUQ2YHU


20






















BLANK PAGE

‹UCLES 2013
BioMedical Admissions Test – Past Paper 2013

Section 2
Answer Key

Question Answer key


1 H
2 D
3 D
4 A
5 F
6 B
7 H
8 F
9 F
10 B
11 C
12 B
13 C
14 A
15 D
16 D
17 A
18 E
19 D
20 D
21 E
22 C
23 D
24 C
25 C
26 B
27 E

© UCLES 2013
BioMedical Admissions Test – Past Paper 2013

Section 2
Score Conversion

Total mark BMAT score


0 1.0
1 1.0
2 1.2
3 2.0
4 2.6
5 3.1
6 3.6
7 4.0
8 4.3
9 4.6
10 4.9
11 5.2
12 5.5
13 5.8
14 6.1
15 6.4
16 6.7
17 6.9
18 7.3
19 7.6
20 7.9
21 8.3
22 8.7
23 9.0
24 9.0
25 9.0
26 9.0
27 9.0

© UCLES 2013
BioMedical Admissions Test 4500/12

Wednesday 7th November 2012 30 minutes




SECTION 2 Scientific Knowledge and Applications

Instructions to Candidates

Please read this page carefully, but do not open the question paper until you are told
that you may do so.

A separate answer sheet is provided for this section. Please check you have one.
You also require a soft pencil and an eraser.

Please complete the answer sheet with your:

• BMAT candidate number


• centre number
• date of birth
• name

Speed as well as accuracy is important in this section. Work quickly, or you may not
finish the paper. There are no penalties for incorrect responses, only points for correct
answers, so you should attempt all 27 questions. All questions are worth one mark.

Answer on the sheet provided. Most questions ask you to show your choice between options
by shading a circle. If questions ask you to write in words or numbers, be sure to write
clearly in the spaces provided. If you make a mistake, erase thoroughly and try again.

Any rough work should be done on this question paper.

Calculators are NOT permitted.

Please wait to be told you may begin before turning this page.

This paper consists of 21 printed pages and 3 blank pages.

The questions in this paper that are marked with an asterisk (* Qs: 5, 6, 18, 22)
assume knowledge that is not currently on the BMAT specification.
2

BLANK PAGE

© UCLES 2012
3

1 Which of the following could result in a homeostatic response?

1 A rise in an internal condition.

2 A reduction in an internal condition.

3 A rise in an external condition.

4 A reduction in an external condition.

A 1 and 2 only

B 1 and 3 only

C 2 and 4 only

D 3 and 4 only

E 1, 2 and 3 only

F 1, 2, 3 and 4

2 2.74g of 1-bromobutane is reacted with excess aqueous sodium hydroxide to produce 1.11g of
butan-1-ol according to the equation below.

C4H9Br + NaHO → C4H9OH + NaBr

What is the percentage yield of butan-1-ol?

(Ar : H = 1; C = 12; O = 16; Na = 23; Br = 80)

A 26%

B 40%

C 54%

D 75%

© UCLES 2012 [Turn over


4

3 Nuclide N
X is an unstable isotope which decays in two stages into nuclide Z as shown:
R

N
R X → R −2P Y → QP Z

What are the values of P and Q?

P Q

A N-4 R+1

B N-4 R-1

C N-4 R-2

D N R-1

E N R-2

F N R-4

© UCLES 2012
5

4 The design in the diagram is formed from four circles which all touch at the top of the shape (the
diagram is not to scale).

The diameter of the smallest circle is d, the second is 2d, the third is 3d and the largest is 4d.

Find an expression for the area of the shading in the design.

5 2
A πd
2
B 4πd 2
1 2
C πd
4
D 10πd 2
3 2
E πd
2

© UCLES 2012 [Turn over


6
* 5 The diagrams below show part of the breathing apparatus of a human.

Which row of the table correctly identifies the locations affected by two chemicals found in
cigarettes and by two diseases associated with smoking?

Location affected

1 2 3 4

A Carbon monoxide Bronchitis Nicotine Emphysema

B Nicotine Bronchitis Emphysema Carbon monoxide

C Carbon monoxide Nicotine Bronchitis Emphysema

D Nicotine Emphysema Bronchitis Carbon monoxide

E Bronchitis Carbon monoxide Emphysema Nicotine

F Bronchitis Emphysema Carbon monoxide Nicotine

© UCLES 2012
7

* 6 The diagram represents the molecule lecithin, a chemical that is added to chocolate drinks to
prevent the oils separating out from water.

head tail

Which row of the table below correctly identifies the characteristics of the molecule?

Head Tail

A hydrophobic hydrophobic

B hydrophobic hydrophilic

C hydrophilic hydrophobic

D hydrophilic hydrophilic

neither hydrophobic nor


E hydrophilic
hydrophilic
neither hydrophobic nor
F hydrophilic
hydrophilic

© UCLES 2012 [Turn over


8

7 Students investigate a radioactive source. They place a detector close to a radioactive source
and take 5 readings over 5 minutes. They then place a thin sheet of paper between the detector
and the source, and again observe the counts over 5 minutes. Lastly they replace the paper
sheet with an aluminium one and observe the counts over 5 minutes. Their results are shown
below:

Nothing Paper Aluminium


Reading 1 100 101 30
Reading 2 98 102 31
Reading 3 99 96 28
Reading 4 103 101 33
Reading 5 101 103 27

What type(s) of radiation is being given off by the source?

A α only

B β only

C γ only

D α and β

E α and γ

F β and γ

© UCLES 2012
9

8 A formula which connects the 'going', G, with the 'rise', R, of stairs in a staircase is below:

G = 5 + 7( 9 − R )2 + 9 going

rise

Rearrange the formula to give R in terms of G.

( G − 5 )2 + 9
A R =9−
7
G −8
B R =9−
7

( G − 5 )2 − 9
C R =9+
7
G −8
D R =9+
7

( G − 5 )2 − 9
E R =9−
7

9 In one type of neurological disease, neurons are unable to detect a certain stimulus which would
normally cause pain. In a patient with this disease, which of the following would be true?

1 The patient would not show a reflex action to the stimulus.

2 The patient would be able to take appropriate action if they could also see the
application of the stimulus.
3 The patient would be able to sense pain but would not be able to respond to it.

4 The patient would be able to sense pain but would not be able to communicate
this to the brain.

A 1 and 2 only

B 1 and 3 only

C 1 and 4 only

D all of the statements

© UCLES 2012 [Turn over


10

10 Inorganic builders are put into detergents to act as a buffer, i.e. to keep the pH close to 7.
Sodium tripolyphosphate is a major one used. The equation for its formation is shown below:

aNaH2PO4 + bNa2HPO4 → cNa5P3O10 + dH2O

What are the values of a, b, c and d?

A 3, 1, 1, 3

B 6, 2, 2, 7

C 2, 4, 2, 2

D 1, 2, 1, 2

E 2, 2, 1, 2

© UCLES 2012
11

11 The diagrams show, not to scale, three different situations in which a force F acts. Also shown in
each case is a distance d.

Diagram 1: Diagram 2: Diagram 3:


Person sitting on a chair Wheelbarrow being lifted Weight being lifted by a motor

Which line in the table shows whether or not work is being done by force F in each situation and, if
so, whether the work done is equal to F x d?

Work being done by force F? Work done = F x d?

A only in diagrams 1 and 2 only in diagram 1

B only in diagrams 1 and 2 only in diagram 2

C only in diagrams 2 and 3 only in diagram 2

D only in diagrams 2 and 3 only in diagram 3

E in diagrams 1, 2 and 3 only in diagrams 1 and 2

F in diagrams 1, 2 and 3 only in diagrams 2 and 3

G only in diagrams 1 and 3 only in diagram 1

H only in diagrams 1 and 3 only in diagram 3

© UCLES 2012 [Turn over


12

12 Evaluate:

2 × 10 5
− (4 × 10 ) − (4 × 10 )
3 2

)
3
(5 × 10 −3 2

A − 6.2

B 160 − 20 5

C 140

D 240 − 20 5

E 1940

© UCLES 2012
13

13 The left diagram shows a plate containing a jelly with one species of bacterium evenly spread
through it. Placed on the jelly are three discs called P, Q and R. They have been treated as in
the key.

The right diagram shows the plate 48 hours later.

colonies of Key:
bacteria
P = disc with antibiotic at
P P strength ¼X

Q = disc with antibiotic at


Q R Q R strength ½X

R = disc with antibiotic at


strength X
S

Which of the following is/are possible explanations for the result?

1 Antibiotic at strengths X and ½X are equally as effective.

2 Bacterial resistance to this antibiotic occurs at all three strengths.

3 S may represent the maximum distance the antibiotic has diffused out of the disc.

A 1 only

B 2 only

C 3 only

D 1 and 2 only

E 1 and 3 only

F 2 and 3 only

G 1, 2 and 3

H none of these

© UCLES 2012 [Turn over


14

14 Azurite is a mineral made up of a mixture of CuCO3 and Cu(OH)2.

Which one of the following is a possible formula of azurite?

A Cu3CH4O4

B Cu3CH4O5

C Cu3CH4O6

D Cu3C2H2O6

E Cu3C2H2O7

F Cu3C2H2O8

15 The microwaves generated in a microwave oven travel through air at a speed of 3.0 x 108m/s,
with a wavelength of 12cm. They pass through plastic food containers, but at a reduced speed
of 2.0 x 108m/s.

What are the wavelength and frequency of these microwaves as they pass through a plastic
food container?

Wavelength (cm) Frequency (Hz)

A 8 1.7 x 109

B 8 2.5 x 109

C 8 3.8 x 109

D 12 1.7 x 109

E 12 3.8 x 109

F 18 1.7 x 109

G 18 2.5 x 109

H 18 3.8 x 109

© UCLES 2012
15

16 In the triangle ABC shown below (not to scale):

1 2
tan A = and tan B =
6 3

M is the midpoint of AB.

What is the value of tan θ

1
A
9
5
B
12
4
C
9
1
D
2
5
E
6

© UCLES 2012 [Turn over


16

17 Consumption of alcohol leads to an increase in dilute urine production by reducing the level of
the hormone ADH.

Which of the following are correct statements about alcohol and/or ADH?

1 ADH travels in the bloodstream.

2 There is a positive correlation between the level of ADH and the production of
dilute urine.
3 Increased ADH leads to increased formation of dilute urine.

4 A reduced level of ADH could lead to dehydration.

A 1 and 2 only

B 1 and 3 only

C 1 and 4 only

D 2 and 3 only

E 2 and 4 only

F 3 and 4 only

* 18 Vanadium is a metal that lies just above zinc in the reactivity series.

Which one of the following could not be used to obtain the metal from its ore?

A electrolysis of the molten chloride

B heating of the chloride with metallic sodium

C heating the oxide with metallic aluminium

D treating a solution of vanadium sulfate with metallic iron

E treating a solution of vanadium chloride with metallic magnesium

© UCLES 2012
17

19 The circuit shows three lamps and two ammeters in a circuit.

Lamp X 'blows' (the filament breaks).

What happens to the reading on each ammeter, and to the total resistance of the circuit?

Reading on Reading on Total resistance


ammeter 1 ammeter 2 of circuit
A decreases decreases decreases

B decreases decreases increases

C decreases increases decreases

D decreases increases increases

E increases decreases decreases

F increases decreases increases

G increases increases decreases

H increases increases increases

© UCLES 2012 [Turn over


18

20 In a fairground game there are two bags, each of which contains 4 coloured balls. There are a
total of 4 red balls, 3 yellow balls and 1 blue ball. The player chooses one of the bags and
removes two balls without replacing them. If the two balls are the same colour then the player
wins. The player is equally likely to choose either bag and the balls are arranged to give the
smallest possible probability for the player to win.

What is the probability that the player wins?

A 16

B 14

C 9 24

D 12

E 34

21 In a monohybrid genetic cross a ratio of phenotypes of 3:0 rather than the usual 3:1 ratio was
seen. This could be due to:

1 offspring with both dominant alleles not surviving.

2 only a small number of offspring being produced.

3 chance.

A 1 only

B 2 only

C 1 and 2 only

D 2 and 3 only

E 1, 2 and 3

© UCLES 2012
19

* 22 Tritium, symbol T, is hydrogen with a mass number of 3. It is radioactive and undergoes β


decay.

Which of the following could be the only product(s) after a quantity of HTO undergoes decay?

1 HeOH

2 H2O, O2 and He

3 H2O, H2 and He

A 1 only

B 2 only

C 3 only

D 1 and 2 only

E 1 and 3 only

F 2 and 3 only

© UCLES 2012 [Turn over


20

23 A cyclist and a bike have a combined mass of 100kg. The cyclist free-wheels (rolls without
pedalling) at a constant speed of 0.8m/s down a 1 in 10 slope (this means that the cyclist
descends 1.0m for each 10m travelled along the road, as shown in the diagram.)

Calculate the loss in gravitational potential energy as he loses 100m in vertical height and
hence calculate the total resistive force on the cyclist.
[g = 10N/kg]

Loss in gravitational
Resistive force (N)
potential energy (J)
A 3200 3.2

B 3200 32 99

C 3200 32 100

D 100 000 100

E 100 000 1000 99

F 100 000 1000 101

24 The new sign for a local business contains two different sections. One of the sections will be
produced from wood, while the other will be metal. Metal is three times as expensive as wood.
The cost of metal needed for each sign is proportional to the diameter of the sign, while the cost
of wood needed is proportional to the square of the diameter. If the diameter of the sign is
doubled, then the total cost of the materials will be tripled.

What percentage (to the nearest 1%) of the sign is metal?

A 25%

B 33%

C 50%

D 67%

E 75%

© UCLES 2012
21

25 What is the minimum number of people shown in the family pedigree who must be
heterozygous for the two situations described in the table in the absence of any new mutations?

Only U shows a Only R and U show a


recessive condition recessive condition
A 1 1

B 1 2

C 2 3

D 2 4

E 3 4

F 3 5

G 4 5

26 A mixture of C2H4(g) and H2(g) is placed in a sealed container. Some finely divided nickel is
added as a catalyst to the mixture. The temperature rises to 150°C and the two gases slowly
react to give C2H6(g).

Which row of the table correctly identifies how the pressure changes after the nickel is added?

Initially During the course of the reaction

A stays the same stays the same

B stays the same decreases

C stays the same increases

D increases stays the same

E increases decreases

F increases increases

© UCLES 2012 [Turn over


22

27 A sound wave is produced by a loudspeaker cone, which creates pulses of pressure by moving
back and forth between two points X and Y as shown in the diagram.

The distance between points X and Y is 5.0mm and the loudspeaker produces pulses of
pressure every 0.2 milliseconds.

The following statements about the sound wave produced are made:

P It has a speed of 25m/s

Q It has an amplitude of 5.0mm

R It has a wavelength of 5.5mm

S It has a fundamental frequency of 5.0kHz

Which of these statements can be correctly deduced from the information given?

A P only

B P and Q only

C P and R only

D P, R and S only

E Q and S only

F R and S only

G S only

© UCLES 2012
23

BLANK PAGE

© UCLES 2012 [Turn over


24

BLANK PAGE

© UCLES 2012
BioMedical Admissions Test – Past Paper 2012 Section 2

Answer Key

Question Answer
1 F
2 D
3 B
4 A
5 B
6 C
7 F
8 E
9 A
10 D
11 D
12 E
13 E
14 F
15 B
16 C
17 C
18 D
19 D
20 B
21 D
22 B
23 D
24 A
25 E
26 E
27 G

© UCLES 2012
Score Conversion

Total Mark BMAT score


0 1.0
1 1.0
2 1.0
3 1.2
4 1.9
5 2.4
6 2.8
7 3.2
8 3.6
9 4.0
10 4.3
11 4.6
12 5.0
13 5.3
14 5.6
15 5.9
16 6.2
17 6.6
18 6.9
19 7.3
20 7.6
21 8.0
22 8.5
23 9.0
24 9.0
25 9.0
26 9.0
27 9.0

© UCLES 2012
BioMedical Admissions Test 4500/12

Wednesday 2nd November 2011 30 minutes




SECTION 2 Scientific Knowledge and Applications

Instructions to Candidates

Please read this page carefully, but do not open the question paper until you are told
that you may do so.

A separate answer sheet is provided for this section. Please check you have one.
You also require a soft pencil and an eraser.

Please complete the answer sheet with your:

 BMAT candidate number


 centre number
 date of birth
 name

Speed as well as accuracy is important in this section. Work quickly, or you may not
finish the paper. There are no penalties for incorrect responses, only points for correct
answers, so you should attempt all 27 questions. All questions are worth one mark.

Answer on the sheet provided. Most questions ask you to show your choice between options
by shading a circle. If questions ask you to write in words or numbers, be sure to write
clearly in the spaces provided. If you make a mistake, erase thoroughly and try again.

Any rough work should be done on this question paper.

Calculators are NOT permitted.

Please wait to be told you may begin before turning this page.

This paper consists of 19 printed pages and 1 blank page.

The questions in this paper that are marked with an asterisk (* Qs: 1, 21)
assume knowledge that is not currently on the BMAT specification.
2
* 1 Gland Hormone One function

Adrenal Adrenaline

Oestrogen Female secondary sexual characteristics

Antidiuretic hormone (ADH)

Testes Male secondary sexual characteristics

Pancreas Regulates blood glucose level

Which word or statement does not correctly fit into one of the gaps left in the table?

A Insulin

B Increases heartbeat rate

C Pituitary

D Testosterone

E Ovary

F Carbohydrase

G Regulates water level in blood

2 A metal, X, is in group III of the periodic table. A non-metal, Y, is in group VI of the periodic
table. They react together to form a compound.

What is the formula of the compound?

A X2Y

B X2Y3

C X3Y2

D X3Y6

E X6Y3

© UCLES 2011
3

3 Two identical cars, P and Q, start at the same level. Car P moves at a constant speed of 10m/s up a
hill to a height of 25m in a time of 20s. In the same time car Q moves at a constant speed of 20m/s up
a hill to a height of 50m.

What are the kinetic energies of the cars while they are travelling up the hills, and what are their
gravitational potential energies once they have reached the top?

kinetic energy gravitational potential energy

A car Q has twice as much as car P car Q has twice as much as car P

B car Q has twice as much as car P car Q has four times as much as car P

C car Q has four times as much as car P car Q has twice as much as car P

D car Q has four times as much as car P car Q has four times as much as car P

4 Simplify:

1

3 3
3 x(3 x )

1
A
9
B 1

C 81

x2
D
9
E x2

F 81x 2

© UCLES 2011 [Turn over


4

5 The following statements relate to typical nuclear division in human cells:

1 mitosis results in variation within the species

2 meiosis results in the production of genetically identical cells

3 mitosis results in the production of diploid cells

4 meiosis results in the production of haploid cells

5 mitosis results in the production of two daughter cells

Which statements are correct?

A 1, 2 and 3 only

B 1, 2 and 4 only

C 1, 4 and 5 only

D 2, 3 and 5 only

E 2, 4 and 5 only

F 3, 4 and 5 only

6 When molecules collide, for a reaction to take place, two conditions must be met. Firstly, they
must have sufficient energy to react and secondly, they must have the right orientation. This
means that the ends of the molecules that are going to react must be in contact with each other.

Raising the temperature speeds up a chemical reaction.

Which of the following could be responsible for this?

1 More collisions take place.

2 The average collision has more energy.

3 The orientation of the molecules is more favourable.

A 1 only

B 2 only

C 3 only

D 1 and 2 only

E 1 and 3 only

F 2 and 3 only

G 1, 2 and 3

© UCLES 2011
5

7 Which one of the following statements about nuclear physics is true?

A The process of emission of a gamma ray from a nucleus is called nuclear fission.

B The half life of a radioactive substance is half the time taken for its nuclei to decay.
The number of neutrons in a nucleus is its atomic number (proton number) minus its
C
mass number.

D The process used in nuclear power stations is nuclear fusion.


When a nucleus emits a beta particle, there is no change in the number of particles it
E
contains.
When a nucleus emits an alpha particle, one of its neutrons becomes a proton plus an
F
electron.

8 If you look at a clock and the time is 9.45, what is the angle between the hour and the minute
hands?

A 0°

B 7.5°

C 15°

D 22.5°

E 30°

© UCLES 2011 [Turn over


6

9 Here are five statements about natural selection:

1 Individuals within a species show variation.

2 Individuals within a species compete with each other for, among other things, resources.

3 Individuals with advantageous adaptations are more likely to survive to adulthood.

4 Only individuals with advantageous adaptations will be able to breed.

5 Alleles for advantageous adaptations are more likely to be inherited.

Which of the above statements are correct?

A None

B 1, 2, 3 & 4 only

C 1, 2, 3 & 5 only

D 1, 3, 4 & 5 only

E 2, 3, 4 & 5 only

F All

© UCLES 2011
7

10 Cyclohexene, C6H10, can be represented as:

Use this information to calculate the relative molecular mass of the hydrocarbon shown below:

[Ar values: H = 1; C = 12]

A 108

B 126

C 134

D 138

E 150

© UCLES 2011 [Turn over


8

11 Consider this circuit.

6V

3Ω 3Ω

Which line in the table gives the current flowing in the ammeter, in amps, when the switch is
open, and when it is closed?

ammeter reading/A

switch open switch closed

A 0.0 1.0

B 0.0 2.0

C 1.0 0.0

D 1.0 1.0

E 1.0 2.0

F 2.0 0.0

G 2.0 1.0

H 2.0 2.0

© UCLES 2011
9

12 w, x, y and z are integers such that w < x2, x > y2, y2 < z2 and x > z.

Which one of the following inequalities must be true?

A w<x

B w>y

C w<z

D x>y

E y<z

13 The table below shows information relating to gas exchange in an active muscle when blood first
enters that muscle.

Which row of the table is correct?

concentration oxygen concentration


oxygen
of carbon concentration process of gas of carbon
concentration
dioxide in the in red blood exchange dioxide in
in muscle cells
plasma cells muscle cells

A high low diffusion high low

B high low osmosis high low

C high low osmosis low high

D low high diffusion high low

E low high diffusion low high

F low high osmosis low high

© UCLES 2011 [Turn over


10

14 Which of the following (A-E) correctly identifies all of the compounds from the list below that
contain covalent bonds in their structure?

CO2 (g) Ca(OH)2 (s) H2SO4 (l) MgCO3 (s) NaCl (s) Na2O (s) Na3PO4 (s) SO2 (g) SiO2 (g)

A CO2 (g), SO2 (g), SiO2 (g)

B Ca(OH)2 (s), H2SO4 (l), MgCO3 (s), NaCl (s), Na2O (s), Na3PO4 (s)

C CO2 (g), Ca(OH)2 (s), H2SO4 (l), MgCO3 (s), Na3PO4 (s), SO2 (g), SiO2 (g)

D NaCl (s), Na2O (s)

E All of the compounds

15 A bullet of mass 50g is fired from a rifle with a velocity of 300m/s. It hits a bank of earth and after
travelling 60cm into the bank comes to rest.

What is the average stopping force of the earth in the bank on the bullet?

A 37.5N

B 3.75 x 103N

C 3.75 x 104N

D 3.75 x 106N

© UCLES 2011
11

16 The graphs of the following equations are drawn:

1 y = 3x - 2

2 y = x2

3 y = 1 – x2

4 y=x+6

Which pair of graphs do not intersect?

A 1 and 2

B 1 and 3

C 2 and 3

D 2 and 4

E 3 and 4

© UCLES 2011 [Turn over


12

17 The genetic condition represented by the shading is caused by the presence of at least one
allele for the condition.

P Q

R S T

Which of the following are possible reasons why U has the condition?

1 The condition is dominant.

2 The sperm from T carried the allele for the condition.

3 A mutation present in a egg of S.

A 1 and 2 only

B 1 and 3 only

C 2 and 3 only

D 1, 2 and 3

E None of the above

© UCLES 2011
13

18 Nitrogen Monoxide is prepared by reacting copper with nitric acid.

What should the values of a, b, x and y be in order to balance the following equation?

aCu + bHNO3 → xCu (NO3)2 + yH2O + 2NO

A a = 2, b = 4, x = 2, y = 2

B a = 6, b = 16, x = 6, y = 8

C a = 1, b = 4, x = 1, y = 2

D a = 4, b = 10, x = 4, y = 5

E a = 3, b = 8, x = 3, y = 4

© UCLES 2011 [Turn over


14

19 Which graph correctly shows how the resistance (R) varies with applied voltage (V) for a resistor
at constant temperature?

R R

A B

0 0
0 V 0 V

R R

C D

0 0
0 V 0 V

R R

E F

0 0
0 V 0 V

© UCLES 2011
15

20 The diagram shows three similar right-angled triangles.

1cm

3cm

What is the area of the largest triangle?

5 2
A cm
3
50 2
B cm
27
C 5cm2

D 15cm2
50 2
E cm
3

© UCLES 2011 [Turn over


16

* 21 Using the table, select the correct answer from the table.

Cell Quantity of nuclear DNA


P 1
Q 2
R 0

P Q R

A gamete cheek cell fetal body cell

B zygote enucleated egg cell red blood cell

C sperm cell adult stem cell white blood cell

D egg cell nerve cell enucleated egg cell

E red blood cell fertilised egg cell embryo cell

22 An ore of lead contains 70% of PbS.

Calculate the mass of lead that can be extracted from 478kg of the ore.

[Ar: Pb = 207; S = 32]

A 28.98kg

B 41.40kg

C 144.90kg

D 289.80kg

E 414.00kg

© UCLES 2011
17

23 A ray of orange light travelling through air has a speed of 3.0 x 108m/s and a wavelength of
600nm. (1nm = 10-9m)

What could be the speed, frequency and wavelength of this orange light when travelling through
glass?

Speed/ms-1 Frequency/Hz Wavelength/nm

A 2.0 x 108 3.3 x 1014 400

B 2.0 x 108 3.3 x 1014 600

C 2.0 x 108 5.0 x 1014 400

D 2.0 x 108 5.0 x 1014 600

E 3.0 x 108 3.3 x 1014 400

F 3.0 x 108 3.3 x 1014 600

G 3.0 x 108 5.0 x 1014 400

H 3.0 x 108 5.0 x 1014 600

24 I have two six-sided dice, each with faces numbered from 1 to 6. One of the dice is fair, but the
other is not – it will land on numbers 1 to 5 with equal probability, but lands on 6 with a different
probability.

1
When I roll the dice the probability that I get a total of 12 is .
18

What is the probability that I get a total of 2 when I roll the dice?

1
A
72
1
B
45
1
C
36
1
D
18
1
E
9

© UCLES 2011 [Turn over


18

25 The graph below shows how one factor in the internal environment in a person changes, and is
returned to a normal level.

Raised level

1 3

Normal level

Time

If someone has a condition which makes their homeostatic system less responsive, how would
the shape of the graph be altered?

A 1 would be earlier

B 1 would be less steep

C 2 would be earlier

D 2 would be higher

E 3 would be steeper

F 3 would be earlier

26 An impurity in petroleum is dimethylsulphide, CH3SCH3. When dimethylsulphide is burnt in an


excess of air, which one of the following balanced equations represents the reaction that takes
place?

A CH3SCH3 + 3O2 → 2CO2 + 2H2O + H2S

B 2CH3SCH3 + 7O2 → 4CO2 + 6H2O + 2S

C 4CH3SCH3 + 12O2 → 6CO2 + 12H2O + 2CS2

D 2CH3SCH3 + 7O2 → 4CO + 6H2O + 2SO2

E 2CH3SCH3 + 9O2 → 4CO2 + 6H2O + 2SO2

© UCLES 2011
19

27 At the front of a long column of soldiers is a man regularly hitting a drum 50 times a minute. The
soldiers are told to place their left foot down on the ground when they hear the drum beat. The
column is so long that the soldiers at the back put down their left feet at the same time as the
soldiers in the front put down their right feet.

What is the minimum length of the column of soldiers?

[Speed of sound in air is 330m/s]

A 165m

B 198m

C 330m

D 396m

E 660m

F 792m

END OF TEST

© UCLES 2011
20

BLANK PAGE

© UCLES 2011
BioMedical Admissions Test – Past Paper 2011 Section 2

Answer Key

Question Answer
1 F
2 B
3 C
4 C
5 F
6 D
7 E
8 D
9 C
10 D
11 B
12 D
13 E
14 C
15 B
16 E
17 D
18 E
19 A
20 B
21 D
22 D
23 C
24 B
25 D
26 E
27 B

BMAT Past Paper 2011 Section 2 – © Copyright UCLES 2011


BioMedical Admissions Test – Past Paper 2011 Section 2

Score Conversion

Total Mark BMAT Score


0 1.0
1 1.0
2 1.0
3 1.0
4 1.5
5 2.0
6 2.4
7 2.9
8 3.2
9 3.6
10 3.9
11 4.3
12 4.6
13 4.9
14 5.2
15 5.5
16 5.8
17 6.1
18 6.4
19 6.7
20 7.0
21 7.3
22 7.7
23 8.2
24 8.7
25 9.0
26 9.0
27 9.0

BMAT Past Paper 2011 Section 2 – © Copyright UCLES 2011


*9399653539*

BioMedical Admissions Test 4500/12

Wednesday 3 November 2010 Morning 30 minutes

SECTION 2 Scientific Knowledge and Applications


*1752218110*

Instructions to Candidates

Please read this page carefully, but do not open the question paper until you are told that
you may do so.

A separate answer sheet is provided for this section. Please check you have one.
You also require a soft pencil and an eraser.

Please complete the answer sheet with your BMAT candidate number, centre number, date of birth
and name.

Speed as well as accuracy is important in this section. Work quickly, or you may not finish the
paper. There are no penalties for incorrect responses, only points for correct answers, so you
should attempt all 27 questions. All questions are worth one mark.

Answer on the sheet provided. Most questions ask you to show your choice between options by
shading a circle. If questions ask you to write in words or numbers, be sure to write clearly in the
spaces provided. If you make a mistake, erase thoroughly and try again.

Any rough work should be done on this question paper.

Dictionaries and calculators may NOT be used.

Please wait to be told you may begin before turning this page.

This paper consists of 21 printed pages and 3 blank pages.

The question in this paper marked with an asterisk (* Q9) assumes


knowledge that is not currently on the BMAT specification.
© UCLES 2010
2

BLANK PAGE

© UCLES 2010
3

1 Which row of the table correctly describes what happens when body temperature rises in a
human?

temperature change arterioles (small hair erector capillaries in the


detected by arteries) muscles skin
move nearer skin
A cerebral cortex dilate relax
surface
move away from
B cerebral cortex constrict contract
skin surface

C cerebral cortex dilate contract do not move

move away from


D hypothalamus constrict relax
skin surface

E hypothalamus dilate relax do not move

move nearer skin


F hypothalamus constrict contract
surface

2 A compound of iodine and oxygen contains 63.5g of iodine and 20.0g of oxygen.

Which of the following would be its formula?

(Ar : I = 127; O = 16)

A IO

B IO2

C I2O

D I2O3

E I2O5

F I5O2

© UCLES 2010 [Turn over


4

3 In a laboratory experiment, protactinium-234 undergoes radioactive decay by β-emission into


uranium-234.

The table below describes how the mass of uranium-234 present in the sample varies with time
from the start of the experiment:

time / min mass of u-234 / mg


0.0 0.0
1.2 8.0
2.4 12.0
3.6 14.0
4.8 15.0
6.0 15.5
7.2 15.7
8.4 15.9
9.6 15.9
10.8 16.0
12.0 16.0

Using the information in the table, approximately what is the half-life of protactinium-234?

A 1.2 minutes

B 2.4 minutes

C 6.0 minutes

D 9.6 minutes

E 10.8 minutes

F 12.0 minutes

4 I have two containers with different capacities. Initially, the larger one is full of water and the
smaller one is empty. I pour water from the larger container into the smaller container until they
contain the same volume of water. The volume of water in the large container is now p times its
capacity and the volume of water in the small container is q times its capacity.

Which one of the following statements about p and q must be true?

A p + q = 1 (but p and q are not necessarily both 0.5)

B p = 0.5 and q = 0.5

C p = 0.5 and q > 0.5

D p > 0.5 and q = 0.5

E p > 0.5 and q > 0.5

© UCLES 2010
5

5 The following statements are about hormone levels and their effect:

1 Increasing levels of insulin cause an increase in blood glucose levels.

2 Increasing levels of oestrogen increases the thickness of the inner lining of the
uterus.
3 Increasing levels of adrenaline increases the heart rate.

Which of the statements are true?

A 1 and 2

B 1 and 3

C 2 and 3

D all of the above

E none of the above

6 Carbon, in the form of coke, is used to reduce iron oxide in a blast furnace. The three stages
are shown below:

1 C + O2 → CO2

2 CO2 + C → 2CO

3 3CO + Fe2O3 → 2Fe + 3CO2

If 12g of carbon is used in stage 2 and all the carbon monoxide produced is used in stage 3,
what mass of carbon dioxide is produced in stage 3?

(Ar : C = 12; O = 16)

A 17.8g

B 35.6g

C 44g

D 88g

E 132g

© UCLES 2010 [Turn over


6

7 The depth of water in a particular tidal harbour varies with time as shown in the graph:

20

18

16

14
depth / metres

12

10

0
0 2 4 6 8 10 12 14 16 18 20 22 24 26
time / hours

If the variation in depth caused by the effect of the tide is considered as a wave, what are the
amplitude and frequency of this wave?

amplitude / metres frequency / hertz

A 3 1/(12 x 3600)

B 3 3600/12

C 6 1/(24 x 3600)

D 6 3600/24

E 8 1/(12 x 3600)

F 8 3600/12

G 16 1/(24 x 3600)

H 16 3600/24

© UCLES 2010
7

8 A children's game is played on a square grid starting in the centre. Players spin two spinners to
decide how to move their counters. The first spinner decides the direction (Left, Right, Up or
Down) and the second spinner decides the distance (1, 2, 3 or 4 squares).

L R 1 2
U D 3 4
What are the chances that, after two moves, a player is exactly back where they started?

A 1/256

B 1/64

C 1/16

D 1/8

E 1/4

* 9 Which one of the following correctly completes the statement:

During the process of evolution, natural selection will favour individuals with...

A an advantageous gene pool.

B an advantageous allele.

C a high reproductive capacity.

D a wide geographic distribution.

E a narrow geographic distribution.

© UCLES 2010 [Turn over


8

10 Which two of the following are produced by the complete combustion of fuels?

1 CH4

2 CO

3 CO2

4 H2O

5 He

6 NH3

A 1 and 3

B 2 and 4

C 2 and 6

D 3 and 4

E 3 and 6

F 4 and 5

© UCLES 2010
9

11 When radioactive isotopes decay, they sometimes have to go through a succession of


disintegrations to reach a stable isotope. These are called decay chains, and involve the
successive emission of numerous α and/or β particles.

One such isotope is radon-219 ( 219


86 Rn), which goes through a chain in which three α particles
and two β particles are emitted before reaching a stable isotope.

What are the atomic and mass numbers of the resulting stable isotope?

atomic number mass number

A 80 207

B 80 211

C 82 207

D 82 215

E 85 211

F 85 219

G 86 215

H 86 219

12 The mean time for running a race by a group of 20 people was 54 seconds. The times for a
second group of people were added and the value of the mean went up to 56 seconds.

Which formula represents the relationship between the number of people in the second group, P,
and the mean time of the second group, T?

40
A P=
T − 54

1080
B P=
T − 54

40
C P=
T − 56

1080
D P=
T − 56

1120
E P= − 20
T

© UCLES 2010 [Turn over


10

13 Signals are transmitted from one neuron to the next neuron, by molecules. These statements
are about this process:

1 Transmitter molecules are formed in the receptors.

2 The signal is transmitted across the synapse by osmosis.

3 Transmitter molecules are released once the signal has been transmitted across
the synapse.
4 The release of transmitter molecules is triggered by an impulse.

5 The signal is transmitted across the synapse by diffusion.

Which of the above statements are correct?

A 1 and 2

B 1 and 3

C 1 and 5

D 2 and 4

E 3 and 4

F 4 and 5

© UCLES 2010
11

14 Which of the following ionic equations are correct?

1 X+ + e- → X

2 X- - e- → X

3 O2- + 2e- → O

4 O2- - e- → O2

5 2I- - 2e- → I

6 Ca2+ + 2e- → Ca

A 1, 2 and 6

B 1, 3 and 5

C 1, 4 and 5

D 2, 3 and 6

E 2, 4 and 5

F 3, 4 and 6

© UCLES 2010 [Turn over


12

15 The circuit shows five identical filament bulbs designed to work at 12V connected in a circuit
with two switches. Switch P is initially open and switch Q is initially closed.

12V

switch P

bulb X bulb Y

switch Q

Switch P is then closed and switch Q is opened.

Compared with their brightness before these changes were made, how has the brightness of
bulbs X and Y changed?

bulb X bulb Y

A brighter brighter

B brighter dimmer

C dimmer brighter

D dimmer dimmer

E unchanged brighter

F brighter unchanged

© UCLES 2010
13

16 A shape is formed by drawing a triangle ABC inside the triangle ADE.


BC is parallel to DE.

AB = 4cm BC = xcm DE = x + 3cm DB = x - 4cm

A
diagram not
to scale

B C

D E
Calculate the length of DE.

A 5cm

B 7cm

C 9cm

D 4 + 2 7 cm

E 7 + 2 7 cm

© UCLES 2010 [Turn over


14

17 In the family tree shown below, both P and Q are carriers of a recessive allele which
causes a condition. Only individuals R and X have the condition.

P Q

R S T U V

W X

What is the percentage likelihood of S, T and U each being a carrier?

Percentage likelihood of being a carrier

S T U

A 25 25 50

B 25 25 100

C 25 50 100

D 50 50 50

E 50 50 100

F 100 50 50

G 100 100 50

© UCLES 2010
15

18 Magnesium hydrogen phosphate contains the following ions: Mg2+, H+ and PO43-.

Which one of the following is a possible formula for magnesium hydrogen phosphate?

A Mg(HPO4)2

B Mg(H2PO4)2

C MgH3PO4

D Mg(H3PO4)2

E Mg2HPO4

F Mg2H2PO4

© UCLES 2010 [Turn over


16

19 The diagrams below show velocity-time or distance-time graphs for 4 different objects, P, Q, R
and S.

12 70
60
velocity / m/s

velocity / m/s
10
50
8
40
6
30
4 20
2 10
0 0
0 4 8 12 16 20 24 28 32 0 4 8 12 16 20 24
time / s time / s
P Q

14 175
12 150
distance / m

distance / m
10 125
8 100
6 75
4 50
2 25
0 0
0 1 2 3 4 5 6 0 60 120 180 240
time / s time / s
R S

Which graph(s) show an object accelerating at 2.4m/s2?

A P only

B Q only

C R only

D S only

E P and Q

F Q and R

G P and S

© UCLES 2010
17

20 The total surface area of a cylinder is numerically the same as its volume.
The radius of the cylinder is r cm, the height is h cm.

Express h in terms of r.

2r
A h=
r−2

2r
B h=
r+2

C h=r+2

D h = r−2

E h = 2r (r − 2)

21 The following statements are about nuclear division by meiosis and mitosis.

1 In animals meiosis only occurs in the reproductive organs.

2 Mitosis can result in the formation of clones.

3 Meiosis results in two nuclei.

4 Mitosis results in four nuclei.

5 Meiosis does not occur during asexual reproduction.

Which of these statements are true?

A 1, 2 and 3

B 1, 2 and 5

C 1, 3 and 4

D 2, 4 and 5

E 3, 4 and 5

© UCLES 2010 [Turn over


18

22 A student prepared nitrobenzene by the following reaction.

C6H6 + HNO3 → C6H5NO2 + H2O


benzene nitrobenzene

Starting with 3.9g of benzene, the student obtained 3.69g of nitrobenzene.

What is the percentage yield?

(Ar : H = 1; C = 12; N = 14; O = 16)

A 11%

B 53%

C 60%

D 95%

23 In an ornamental fountain, water is squirted vertically upwards through a nozzle by a pump.


5kg of water pass through the nozzle each second, and the water reaches a height of 5m after
leaving the nozzle.

What is the power of the pump (assuming 100% efficiency), and at what speed does the water
leave the nozzle?

(Take g to be 10N/kg)

power of pump / W speed of water / m/s

A 5 5

B 5 50

C 25 10

D 25 100

E 50 5

F 50 50

G 250 10

H 250 100

© UCLES 2010
19

24 A design is set up by joining the points which are one third of the way along the sides of a
square. This forms a second square as shown.

This process is repeated.

Calculate the area of the fourth square as a fraction of the original square.

1
A
27

5 5
B
81

125
C
729

25
D
81

5 5
E
27

© UCLES 2010 [Turn over


20

25 The hormonal and nervous systems are often compared because of their similar roles in the
body.

Which of the following statements about the two systems are true?

1 The nervous system relies only on electrical impulses to work.

2 Both the hormonal and nervous systems activate target structures.

3 The fastest responses in the body are produced by the nervous system.

4 Only the hormonal system relies on the activity of chemicals.

5 Parts of the hormonal system may be controlled by the central nervous system.

A 1, 2 and 3

B 1, 2 and 4

C 1, 2 and 5

D 2, 3 and 4

E 2, 3 and 5

F 3, 4 and 5

© UCLES 2010
21

26 Cyclohexene, C6H10, is drawn as:

Apply this information for the steroid-type structure shown below to find the total number of
carbon atoms in the molecule.

CH3 CO2H

CH3

A 17

B 20

C 21

D 22

E 26

F 27

© UCLES 2010 [Turn over


22

27 A car of mass 800kg moves up an incline of 1 in 20 (1 in 20 means for every 20m along the road
the car gains 1m in height) at a constant speed of 20m/s. The frictional force opposing motion is
500N.

How much work has been done by the engine after the car has moved 50m?

A 20kJ

B 25kJ

C 27kJ

D 45kJ

E 65kJ

F 160kJ

END OF TEST

© UCLES 2010
23

BLANK PAGE

© UCLES 2010
24

BLANK PAGE

Disclaimer
The opinions expressed in this paper are not to be taken as the opinions of the University of Cambridge
Local Examinations Syndicate (UCLES) or any of its subsidiaries.

Permission to reproduce items where third-party owned material protected by copyright is included has
been sought and cleared where possible. Every reasonable effort has been made to trace copyright
holders, but if any items requiring clearance have unwittingly been included, UCLES will be pleased to
make amends at the earliest possible opportunity.

© UCLES 2010
BioMedical Admissions Test – Past Paper 2010 Section 2

Answer Key

Question Answer
1 E
2 E
3 A
4 C
5 C
6 D
7 A
8 C
9 B
10 D
11 C
12 C
13 F
14 A
15 B
16 C
17 E
18 B
19 B
20 A
21 B
22 C
23 G
24 C
25 E
26 B
27 D

BMAT Past Paper 2010 Section 2 – © Copyright UCLES 2010


BioMedical Admissions Test – Past Paper 2010 Section 2

Score Conversion

Total Mark BMAT Score


0 1.0
1 1.0
2 1.0
3 1.6
4 2.2
5 2.7
6 3.1
7 3.4
8 3.8
9 4.1
10 4.4
11 4.6
12 4.9
13 5.2
14 5.4
15 5.7
16 6.0
17 6.2
18 6.5
19 6.8
20 7.1
21 7.4
22 7.7
23 8.1
24 8.6
25 9.0
26 9.0
27 9.0

BMAT Past Paper 2010 Section 2 – © Copyright UCLES 2010


BioMedical Admissions Test 4500/12

Wednesday 4th November 2009 30 minutes


*1811945425*

SECTION 2 Scientific Knowledge and Applications

Instructions to Candidates

Please read this page carefully, but do not open the question paper until you are told
that you may do so.

A separate answer sheet is provided for this section. Please check you have one.
You also require a soft pencil and an eraser.

Please complete the answer sheet with your:

• BMAT candidate number


• centre number
• date of birth
• name

Speed as well as accuracy is important in this section. Work quickly, or you may not
finish the paper. There are no penalties for incorrect responses, only points for correct
answers, so you should attempt all 27 questions. All questions are worth one mark.

Answer on the sheet provided. Most questions ask you to show your choice between options
by shading a circle. If questions ask you to write in words or numbers, be sure to write
clearly in the spaces provided. If you make a mistake, erase thoroughly and try again.

Any rough work should be done on this question paper.

Calculators are NOT permitted.

Please wait to be told you may begin before turning this page.

This paper consists of 18 printed pages and 2 blank pages.

The question in this paper marked with an asterisk (* Q9) assumes


knowledge that is not currently on the BMAT specification.
2

BLANK PAGE

BMAT Section 2 © UCLES 2009


3

1 Individual A in the family pedigree below is homozygous dominant and individual B is


homozygous recessive for a particular feature.

A B

C D E

What is the percentage probability that individual F is homozygous recessive if:

i) E is homozygous recessive ii) E is heterozygous

A 50 0

B 25 50

C 50 25

D 25 0

E 0 25

BMAT Section 2 © UCLES 2009 [Turn over


4

2 Which of the following could take part in an addition polymerisation?

1 CHI3

2 C24H48

3 C3H7Br

4 C4H6Cl2

5 C8H12Cl4

A 1, 2 and 3

B 1, 2 and 4

C 1, 3 and 5

D 2, 3 and 4

E 2, 4 and 5

F 3, 4 and 5

BMAT Section 2 © UCLES 2009


5

3 Shortly after opening her parachute, a free-fall parachutist of mass 60kg experiences the forces
shown in the diagram.

weight = 600N

Which line in the table gives the size and direction of the acceleration of the parachutist at this
instance?

size of acceleration (m/s2) direction of acceleration

A 5.0 downwards

B 10.0 downwards

C 5.0 upwards

D 10.0 upwards

E 0.0 -

BMAT Section 2 © UCLES 2009 [Turn over


6

4 A bag contains x red balls, y blue balls and z yellow balls. One ball at random is taken out and
replaced. A second ball at random is taken out and replaced.

If the balls are identical in all respects except colour and are well mixed, what is the probability
that the first ball was red and the second blue?

A (x+y)/(x+y+z)2

B xy/(x+y+z)(x+z)

C xy/(x+y+z)2

D (x+y)/(x+y+z)

E xy/(y+z)(x+z)

5 Which one of the following statements is true about the members of a clone?

A Members of a clone will always show identical features at maturity.

B Multiple births, such as twins, are always members of a clone.

C They are produced only during genetic engineering.

D They are the result of a mutation.

E They contain identical DNA.

6 Which one of the following covalent substances could exist as a giant structure?

A SiCl4

B SiO2

C ICl

D Cl2O

E H2S

F CF4

BMAT Section 2 © UCLES 2009


7

7 Which of the following is a correct unit of potential difference (voltage)?

A amp per ohm

B coulomb per joule

C joule per second

D newton per coulomb

E watt per amp

8 A cube has unit length sides. What is the length of a line joining a vertex to the midpoint of one
of the opposite faces (the dashed line in the diagram below)?

A 2

3
B
2
5
C
2

D 3

E 5

BMAT Section 2 © UCLES 2009 [Turn over


8

* 9 The data below considers the risk of having a cancer in the neck region due to drinking alcohol
and one genetic factor (presence of a mutant allele for the CYP1A1 gene). A risk value of 1.0 is
the mean average risk in the human population.

Number of mutant CYP1A1


Alcohol intake Risk value
alleles present
0 1.0
Light drinker 1 1.5
2 1.8
0 4.0
Heavy drinker 1 4.5
2 6.0

Using only the data from the table, which of the following tentative conclusions about the risk of
having a cancer of the neck, is not correct?

A Overall, the presence of at least one mutant allele increases the risk.

B Overall, there is a positive correlation between the risk and alcohol intake.

C Heavy drinking is the main factor in increasing the risk.

D The presence of at least one mutant allele is the main factor for increasing risk.
The presence of two mutant alleles increases the risk more for heavy drinkers than
E
light drinkers.

10 2.00g of a carbon compound is burnt in excess oxygen.

4.77g of carbon dioxide is produced.

What percentage (to the nearest 1%) of carbon is in the original compound?

[Ar: C=12; O=16]

A 21%

B 26%

C 42%

D 52%

E 65%

BMAT Section 2 © UCLES 2009


9

11 It is known that a radioactive source emits a single type of radiation. Detectors are placed in
the two positions shown. The graph shows how the readings change over time.

30cm

source detector 1 detector 2

1m

240

220
detector 1
200

180
count rate (counts/min)

160

140

120

100

80

60

40
detector 2
20

0
0 1 2 3 4 5 6 7 8 9 10 11 12 13 14 15 16
time (hours)

Which type of radiation does the source emit, and what is its half-life?

type of radiation half-life (hours)


A alpha 2.40

B alpha 2.76

C beta 2.40

D beta 2.76

E gamma 2.40

F gamma 2.76

BMAT Section 2 © UCLES 2009 [Turn over


10

12 yx
The symbol ! defines a mathematical binary operation such that y!x= for all
x
positive integers.

What is the value of (2 ! 3) ! 2?

A 2x33

B 8/3

C 32/9

D 4

E 25 2 9

13 Which one of the following movements will be severely and immediately disrupted if the oxygen
concentration in the blood becomes very low?

A carbon dioxide from the blood into the alveoli

B glucose from the lumen of the intestine into the villi

C oxygen from the alveoli into the blood

D urea from the blood plasma into the nephron

E water from the blood plasma into muscle cells

F water from the blood plasma into the nephron

BMAT Section 2 © UCLES 2009


11

14 The order of reactivity of four elements is J>M>Q>T i.e. J is the most reactive and T the least
reactive.

A series of reactions are given below:

1 MCl2 + T → TCl2 + M

2 MSO4 + J → JSO4 + M

3 TO + Q → QO + T

4 QO + J → JO + Q

5 JSO4 + T → TSO4 + J

6 MO + Q → QO + M

Which three reactions could take place?

A 1, 2 and 3

B 2, 3 and 4

C 4, 5 and 6

D 1, 4 and 6

BMAT Section 2 © UCLES 2009 [Turn over


12

15 The graph represents the motion of a vehicle during part of a journey.

30

20
speed (m/s)

10

0
0 1 2 3 4 5 6 7 8
time (min)
What is the best estimate of the distance travelled during the part of the journey shown?

A 100.00m

B 107.50m

C 115.00m

D 6.00km

E 6.45km

F 6.90km

BMAT Section 2 © UCLES 2009


13

16 Evaluate:

⎛ ⎞
⎜ 2 ×103 + 8 ×102 ⎟
⎜ 1 ⎟
⎜ + 3 ×10 ⎟
−4

⎝ 2500 ⎠

A 20

B 140

C 200

D 625

E 1700

F 2000

G 20000

17 Which of the four statements below about natural selection are correct?

1 Competition occurs between individuals of the same species.

2 Competition occurs between individuals of different species.

3 Selection can lead to evolution.

4 Selection can lead to extinction.

A 1 and 3 only

B 2 and 4 only

C 1, 2 and 4 only

D 1, 3 and 4 only

E 2, 3 and 4 only

F All of the above

BMAT Section 2 © UCLES 2009 [Turn over


14

18 The equation for the preparation of nitrogen monoxide is:

a Cu + b HNO3 a Cu (NO3)2 + c H2O + 2NO

What is the value of b?

A 2

B 4

C 6

D 8

E 12

F 16

19 An object of mass 5kg falls from rest and hits the ground at a speed of 20m/s. Air resistance is
negligible.

From what height has the object fallen?

Take g to be 10m/s2

A 10m

B 20m

C 50m

D 100m

E 200m

F 1000m

BMAT Section 2 © UCLES 2009


15

20 A solid sphere of radius r fits inside a hollow cylinder. The cylinder has the same internal
diameter and length as the diameter of the sphere.

The volume of a sphere is 4 π r 3 , where r is the radius of the sphere.


3

What fraction of the space inside the cylinder is taken up by the sphere?

A 1
4

B 1
3

C 1
2

D 2
3

E 3
4

21 The diagram shows a section through an alveolus. Gases diffuse through the alveolar and
capillary walls.

Which row explains why gaseous exchange occurs in the alveolus?

Concentration of:
oxygen in oxygen in carbon dioxide in carbon dioxide in
alveolus capillary alveolus capillary
A high low high low
B low high low high
C high high high low
D high low low high
E low high high low
F low low low high

BMAT Section 2 © UCLES 2009 [Turn over


16

22 At room temperature, a reaction is very fast if no bonds are broken while it takes place, and is
slow if bonds have to be broken.

1 H2 + I2 → 2HI

2 Ag+ + Cl - → AgCl

3 ĊH3 + ĊH3 → C2H6

4 OH- + CH3Br → CH3OH + Br -

Which of the following are the two fastest reactions?

A 1 and 2

B 1 and 3

C 1 and 4

D 2 and 3

E 2 and 4

F 3 and 4

23 A train consists of a powered engine pulling two unpowered carriages.

carriage 2 carriage 1 powered engine


T 15000N

The engine has a mass of 20000kg, and each carriage has a mass of 5000kg. When the engine
accelerates from rest it develops a thrust (driving force) of 15000N as shown.

Ignoring resistive forces, what is the tension (pulling force) T in the coupling between carriage 1 and
carriage 2?

A 2500N

B 3750N

C 5000N

D 7500N

E 15000N

BMAT Section 2 © UCLES 2009


17

24 The equation gives y in terms of x.

2
⎛x ⎞
y = 5 ⎜ − 3 ⎟ − 10
⎝2 ⎠

Rearrange the equation to give an expression for x in terms of y.

y + 10
A ±2 +6
5
y + 10
B ±2 −6
5
y − 10
C ±2 +6
5
y − 10
D ±2 −6
5
y + 50
E ±2 +6
5
y + 10
F ±2 +3
5

BMAT Section 2 © UCLES 2009 [Turn over


18

25 The following five statements relate to homeostasis (the maintenance of stable internal
conditions of the body):

1 Insulin helps to control the water content of the body.

2 Homeostasis depends only on hormones.

3 The pancreas helps to control the glucose content of the blood.

4 Both the nervous and hormonal systems are involved in homeostasis.

5 The skin is used to help control the body temperature.

Which statements are correct?

A 1, 2 and 3 only

B 1, 3 and 4 only

C 1, 4 and 5 only

D 2, 3 and 5 only

E 2, 4 and 5 only

F 3, 4 and 5 only

26 Naturally occurring chlorine is a mixture of two isotopes with mass number 35 and 37. The
isotope with mass number 35 is three times as common as the isotope with mass number 37.
Naturally occurring bromine is a mixture of two isotopes with mass numbers 79 and 81. They
are present in equal amounts.

What fraction of the naturally occurring compound CH2BrCl has a relative molecular mass of
128?

[Ar: H = 1; C = 12]

A 1
8

B 1
4

C 3
8

D 1
2

E 5
8

BMAT Section 2 © UCLES 2009


19

27 The first graph shows the variation of the displacement of particles with distance along a wave at a
particular instant in time:

18
displacement (mm)

60
0
distance (m)

-18

The second graph shows the variation with time of the displacement of a particular particle in this
wave:

18
displacement (mm)

0.6
0
time (s)

-18

What is the speed of this wave?

A 30m/s

B 50m/s

C 90m/s

D 100m/s

E 150m/s

F 300m/s

BMAT Section 2 © UCLES 2009


20

BLANK PAGE

BMAT Section 2 © UCLES 2009


BioMedical Admissions Test – Past Paper 2009 Section 2

Answer Key

Question Answer
1 C
2 E
3 C
4 C
5 E
6 B
7 E
8 B
9 D
10 E
11 C
12 C
13 B
14 B
15 D
16 F
17 F
18 D
19 B
20 D
21 D
22 D
23 A
24 A
25 F
26 C
27 E

BMAT Past Paper 2009 Section 2 – © Copyright UCLES 2009


BioMedical Admissions Test – Past Paper 2009 Section 2

Score Conversion

Total Mark BMAT Score


0 1.0
1 1.0
2 1.0
3 1.5
4 2.0
5 2.4
6 2.8
7 3.2
8 3.5
9 3.8
10 4.1
11 4.3
12 4.6
13 4.9
14 5.1
15 5.4
16 5.7
17 6.0
18 6.2
19 6.5
20 6.9
21 7.2
22 7.6
23 8.0
24 8.5
25 9.0
26 9.0
27 9.0

BMAT Past Paper 2009 Section 2 – © Copyright UCLES 2009


BioMedical Admissions Test 4500/02

Wednesday 5th November 2008 Morning 30 minutes


*8526376966*

SECTION 2 Scientific Knowledge and Applications

Instructions to Candidates

Please read this page carefully, but do not open the question paper until you are told
that you may do so.

A separate answer sheet is provided for this section. Please check you have one.
You also require a soft pencil and an eraser.

Please first write your name, date of birth, BioMedical Admissions Test candidate number
and centre number in the spaces provided on the answer sheet.
Please write very clearly.

Speed as well as accuracy is important in this section. Work quickly, or you may not
finish the paper. There are no penalties for incorrect responses, only points for correct
answers, so you should attempt all 27 questions.
Unless otherwise stated, all questions are worth one mark.

Answer on the sheet provided. Many questions ask you to show your choice between
options by shading a circle (or circles, if specified in the question).
If questions ask you to write in words or numbers, be sure to write clearly in the spaces
provided. If you make a mistake, erase thoroughly and try again.

Any rough work should be done on this question paper.

Calculators are NOT permitted.

Please wait to be told you may begin before turning this page.

This paper consists of 17 printed pages and 3 blank pages.

The questions in this paper that are marked with an asterisk (* Qs: 21, 25)
assume knowledge that is not currently on the BMAT specification.
2

BLANK PAGE

BMAT 2008: Section 2 - © Copyright UCLES 2008


3

1 Which one of the following involves active transport?

A movement of carbon dioxide into alveoli in the lungs

B uptake of alcohol (ethanol) through the lining of the stomach

C secretion of sweat onto the skin's surface

D re-absorption of glucose in the kidney tubules

E loss of urine from the urethra

F transfer of oxygen into the blood from the alveoli

2 Element Y forms an ion Y 3- with the electronic configuration 2, 8, 8.

To which group and period of the periodic table does Y belong?

Group Period
A 3 3
B 3 4
C 5 3
D 5 4
E 8 3
F 8 4

BMAT 2008: Section 2 - © Copyright UCLES 2008


4

3 A radioactive isotope X decays into the stable isotope Y with a half-life of 20 s.

Which graph correctly shows how the mass (m) of Y present in an initially pure sample of X
varies with time (t)?

A B
m m
100 100

75 75

50 50

25 25

0 0
0 20 40 60 80 100 t/s 0 20 40 60 80 100 t/s

C D
m m
100 100

75 75

50 50

25 25

0 0
0 10 20 30 40 t/s 0 10 20 30 40 t/s

E F
m m
100 100

75 75

50 50

25 25

0 0
0 5 10 15 20 t/s 0 5 10 15 20 t/s

BMAT 2008: Section 2 - © Copyright UCLES 2008


5

4 In the formula P = 2r 2t

r = 3 x 10-3 t = 2.5 x 104

Calculate the value of P. Leave your answer in standard form.

A 7.5 x 10-2

B 45 x 10-2

C 75 x 10-2

D 4.5 x 10-1

E 7.5 x 10-1

F 4.5

5 In mice the recessive form of a gene causes the death of the fetus if present in the homozygous
state. Mice heterozygous for this gene were crossbred.

What percentage of the live offspring would be homozygous dominant?

A 0%

B 25 %

C 33 %

D 50 %

E 66 %

F 75 %

BMAT 2008: Section 2 - © Copyright UCLES 2008


6

6 Look at the equation for making chlorine gas and decide which answer will provide you with the
numbers to balance it

aKMnO4 + bHCl ΠaKCl + aMnCl2 + xH2O + yCl2

A a = 1, b = 8, x = 4, y = 3

B a = 1, b = 8, x = 4, y = 4

C a = 1, b = 4, x = 2, y = 1

D a = 2, b = 16, x = 4, y = 6

E a = 2, b = 16, x = 8, y = 5

7 n
The nth term of a sequence is .
n +1

What is the expression for the difference between the (n+1)th term and the nth term?

−1
A
(n + 2)(n + 1)
1
B
(n + 2)(n + 1)
n2 + n
C
(n + 2)(n + 1)
1
D 2
n +2
1
E
n(n + 1)

BMAT 2008: Section 2 - © Copyright UCLES 2008


7

8 The diagram shows a crane that lifts a load with a mass of 100 kg at a constant speed of 0.4 m/s
for 5 seconds. (g=10 N/kg)

Cable P

100 kg

Here are 3 statements about this activity:

1 The gain in gravitational potential energy of the load is 2000 J.

2 The tension in the cable at point P is 2000 N.

3 The load accelerates at 10 m/s2.

Which statements is/are true?

A 1 only

B 2 only

C 3 only

D 1 and 2

E 1 and 3

F 2 and 3

BMAT 2008: Section 2 - © Copyright UCLES 2008


8

9 The graph shows oxygen supply and demand in muscle cells during exercise.

Which statement correctly describes what is happening in the muscle cells between 0 and 11
minutes?

A Anaerobic respiration only is taking place

B More oxygen is being supplied than is required

C Aerobic respiration only is taking place

D Both anaerobic and aerobic respiration are taking place

10 Which one of the following would not be formed by cracking the hydrocarbon C12H26?

A CH4

B C2H4

C C6H16

D C8H18

E C12H24

F H2

BMAT 2008: Section 2 - © Copyright UCLES 2008


9

11 Here are four statements about radioactivity:

1 A beta particle is an electron that is emitted from the outermost electron shell of
an atom.
2 A beta particle is emitted from the nucleus of the atom when a proton changes
into an electron.
3 A beta particle is emitted from the nucleus of the atom when a neutron changes
into an electron.
4 After a beta particle is emitted the mass number of the nucleus of the atom is
unchanged.
Which statement(s) is/are true?

A 1 only

B 2 only

C 3 only

D 4 only

E 1 and 2

F 3 and 4

12 During the breakdown of a solution containing carbohydrates, proteins and lipids by


carbohydrase, protease and lipase enzymes, the pH changed from neutral due to the products
formed.

Which of the options, A to E, correctly shows which constituents contribute to the change in pH?

Carbohydrate Protein Lipid


A yes yes yes
B no yes no
C no no yes
D yes yes no
E no yes yes

BMAT 2008: Section 2 - © Copyright UCLES 2008


10

13 In the diagram below, the triangles are congruent, PQ is a continuation of NL and both L and P
are equidistant from the line X = 0 and from the line Y = 0.

L M

x
P R

Of the transformations (1 to 5) described below, which one of the combinations (A to E) will map
the triangle LMN onto the triangle PQR?

1 Reflection in the line y = 0

2 Rotation of 90º about the origin

3 Reflection in the line y = x

4 Reflection in the line x = 0

5 Rotation of 180º about the origin

A 1 followed by 2

B 3 followed by 4

C 5 followed by 4

D 2 followed by 5

E 4 followed by 5

BMAT 2008: Section 2 - © Copyright UCLES 2008


11

14 In the manufacture of hydrogen from methane and steam, carbon monoxide is produced as an
unwanted side product. The amount of carbon monoxide is reduced by mixing the gases with
steam when the following reversible reaction takes place.

CO (g) + H2O (g) CO2 (g) + H2 (g) ∆H negative (exothermic)

When this reaction reaches equilibrium, which of the following changes would result in more
carbon monoxide being removed?

A Adding a catalyst

B Increasing pressure

C Decreasing pressure

D Decreasing temperature

E Adding nitrogen at constant pressure

15 A car is accelerated from rest along a horizontal road by a constant thrust force produced by the
engine. The car eventually reaches a terminal speed, and the graphs below show the variation
with time of three quantities (X, Y and Z) for the car:

X Y Z

time time time

Which line in the table could correctly identify the quantities X, Y and Z?

X Y Z
A acceleration drag force kinetic energy

B acceleration mass weight

C potential energy velocity kinetic energy

D potential energy drag force weight

E resultant force mass kinetic energy

F resultant force velocity weight

BMAT 2008: Section 2 - © Copyright UCLES 2008


12

16 A right-angled triangle is shown.

Diagram not to scale


6+ 6

4- 6

Calculate the area of the triangle.

A 9− 6

B 9− 3

C 52 + 4 6

D 9+ 6

E 15 + 5 6

17 The reflex arc enables the body to make rapid responses to certain stimuli.

Sensory Motor
1 2 neurone 3 neurone 4 5

Which row in the table correctly identifies the elements 1 to 5 in the reflex arc shown above?

receptor response stimulus muscles CNS


A 5 4 3 2 1
B 2 5 1 4 3
C 1 4 3 2 5
D 5 1 2 3 4
E 1 2 5 4 3
F 2 5 1 3 4

BMAT 2008: Section 2 - © Copyright UCLES 2008


13

18 The general formula for the alkane series is CnH2n+2

The amide series has the same saturated structure as the alkanes but with one H substituted by
the -CONH2 group.

What is the general formula for the amide series?

A CnH2nON

B CnH2n+2ON

C CnH3n-1ON

D Cn+1H2nON

E Cn+1H2n+1ON

F Cn+1H2n+3ON

19 I stop while out walking and take the bearing of a windmill and note it as θ º. I then walk 5 km
north and take the bearing again – it is now 2 θ.

How far away, in km, was the windmill from the position where I took the first bearing?

A 5 cos θ

B 5 cos 2 θ

C 10 cos θ

D 5 sin 2 θ

E 10 sin θ

BMAT 2008: Section 2 - © Copyright UCLES 2008


14

20 The diagram below shows three identical resistors and a battery that supplies a constant 12 V.

12 V

X Z

With the switch open, as shown, the current in resistor Y is 20 mA.


When the switch is closed, what is the current in resistor X, and what is the potential difference
(voltage) across resistor Z?

Current in Potential
X (mA) difference
across Z (V)
A 13 ⅓ 4
B 13 ⅓ 6
C 20 4
D 20 6
E 30 4
F 30 6

BMAT 2008: Section 2 - © Copyright UCLES 2008


15

* 21 The four statements are about the movement of blood during one complete circuit of the body.

1 Blood flowing from the lungs flows through the heart twice.

2 All the blood flowing from the lungs flows through the kidneys.

3 All the blood flowing from the liver flows through the lungs.

4 Only blood flowing from the digestive system flows through the liver.

Which of the above statements are correct?

A 1 and 2 only

B 1 and 3 only

C 1 and 4 only

D 2 and 3 only

E 2 and 4 only

F 3 and 4 only

22 In addition to diamond and graphite, another allotrope of carbon is Buckminster fullerene, C60,
which is in the shape of a sphere.

Which one of the following statements applies to all three allotropes?

A Weak intermolecular forces hold the structures together.

B Carbon atoms are bonded to four other carbon atoms.

C They have similar physical properties.

D They all react with O2 to give the same products.

E They are electrical insulators.

BMAT 2008: Section 2 - © Copyright UCLES 2008


16

23 A lorry of mass m, and travelling initially at speed v along a horizontal road, is brought to rest by
an average braking force F in time t.

Ignoring any other resistive forces, what distance is travelled by the lorry during this time?

F
A
mg

mgv
B
F

mv 2
C
2F
v2
D
2g

E vt

F 2vt

24 A quadratic equation has solutions a and b.

a + b = -5 ab = 3

What is the equation?

A x2 + 5 x + 3 = 0

B x2 - 5 x - 3 = 0

C x2 - 5 x + 3 = 0

D x2 + 3 x + 5 = 0

E x2 + 5 x - 3 = 0

F x2 + 3 x - 5 = 0

BMAT 2008: Section 2 - © Copyright UCLES 2008


17

* 25 Red-green colour blindness is a condition caused by a recessive allele. The gene is carried on
the X chromosome.

A man is red-green colour blind, which of his relatives must carry the recessive allele?

A father and grandfather

B mother and grandmother

C son and daughter

D mother and daughter

E father and son

F brother and sister

26 A sample of an alkali XOH of mass 2.8 g was dissolved in water.

This solution was neutralised by 12.5 cm3 of sulphuric acid of concentration 2.0 mol dm-3.

2 XOH (aq) + H2SO4 (aq) → X2SO4 (aq) + 2 H2O (l)

What is the relative atomic mass of X?

(Ar : H = 1, O = 16, S = 32)

A 13

B 26

C 39

D 52

E 65

F 78

BMAT 2008: Section 2 - © Copyright UCLES 2008


18

27 Taking the speed of sound in air as 300 m/s and in steel as 4800 m/s. A worker lies next to a
railway line and hears the whistle of a train through the steel rails and 1.5 s later hears the
same whistle through the air.

How far away was the train when its whistle sounded?

A 200 m

B 450 m

C 480 m

D 4500 m

E 6750 m

F 7200 m

BMAT 2008: Section 2 - © Copyright UCLES 2008


19

BLANK PAGE

BMAT 2008: Section 2 - © Copyright UCLES 2008


20

BLANK PAGE

BMAT 2008: Section 2 - © Copyright UCLES 2008


BMAT 2008 Section 2

Answer
1 D
2 C
3 B
4 D
5 C
6 E
7 B
8 A
9 D
10 C
11 D
12 E
13 D
14 D
15 F
16 A
17 B
18 F
19 C
20 F
21 B
22 D
23 C
24 A
25 D
26 C
27 C
BioMedical Admissions Test 4500/02

Wednesday 31 October 2007 Morning 30 minutes

SECTION 2 Scientific Knowledge and Applications

Instructions to Candidates

Please read this page carefully, but do not open the question paper until
you are told that you may do so.

A separate answer sheet is provided for this section. Please check you have one.
You also require a soft pencil and an eraser.

Please first write your name, date of birth, BioMedical Admissions Test candidate
number and centre number in the spaces provided on the answer sheet.
Please write very clearly.

Speed as well as accuracy is important in this section. Work quickly, or you


may not finish the paper. There are no penalties for incorrect responses, only
points for correct answers, so you should attempt all 27 questions.
Unless otherwise stated, all questions are worth one mark.

Answer on the sheet provided. Many questions ask you to show your choice
between options by shading a circle (or circles, if specified in the question).
If questions ask you to write in words or numbers, be sure to write clearly in the
spaces provided. If you make a mistake, erase thoroughly and try again.

Any rough work should be done on this question paper.

Calculators are NOT permitted.

Please wait to be told you may begin before turning this page.

This paper consists of 15 printed pages and 5 blank pages.

The questions in this paper that are marked with an asterisk (* Qs: 12, 13, 15, 16, 19)
assume knowledge that is not currently on the BMAT specification.
2

BLANK PAGE

BMAT 2007: Section 2 - © Copyright UCLES 2007


3

1 The table shows the percentages of some substances measured in fluid taken from a
human kidney.

substance %
water 99
proteins 0
glucose 0.1
urea 0.03

Which letter on the diagram shows the most likely source of the fluid?

2 Element x is in the second period of the Periodic Table. An atom of element y has six more
protons than an atom of element x.

Which statement must be correct?

A Elements x and y are in the same period of the table.

B Elements x and y have the same number of electrons in the first shell.

C Element y has six more electrons in its outer shell than does element x.

D The nucleon number of element y is six more than that of element x.

BMAT 2007: Section 2 - © Copyright UCLES 2007


4

3 A detector of radioactivity shows background radiation to be 20 counts per minute with no


radioactive sources nearby. A radioactive source which emits alpha particles and beta
particles is brought very close to the detector and the detector reading increases to 280
counts per minute. A sheet of paper is then placed between the source and the detector,
and the detector reading drops to 60 counts per minute.

How many counts per minute were caused by alpha particles and by beta particles from the
source?

4 The equation connects the variables A, x, y, z, P and Q.

( x + y)2 z
A= Q
P

The following changes are made:


x and y are both increased by 50 %
z is decreased by 20 %
P is doubled and Q remains the same.

What is the resulting percentage change in A?

A 10 % decrease

B 10 % increase

C 2.5 % increase

D 2.5 % decrease

E 20 % decrease

F 20 % increase

BMAT 2007: Section 2 - © Copyright UCLES 2007


5

5 The three statements below are about exhaling.

1 The ribs swing down and inwards during exhaling.

2 The diaphragm muscles contract during exhaling.

3 The pressure in the lungs increases during exhaling.

Which statements are correct?

A 1 only

B 2 only

C 3 only

D 1 and 2 only

E 1 and 3 only

F 2 and 3 only

6 The oscillation period, t, for a spiral spring (weight w, radius R made from wire length l,
radius r with a modulus of rigidity n) with a weight W hanging from it is given by:

2lR 2 (W + w / 3)
t = 2π
nπ r 4 g

If such a spring is used to measure the weight, W, of an object we need to rearrange the
equation so W is the subject.

Which of the following is the correct rearrangement of the equation?

nr 4 gt 2 w
W = -
A 8πlR 2 3

n π r 4 gt 2 w
W= -
B 8lR 2 3

3nr 4 gt 2
W = - 3w
C 8πlR 2

n π r 4 gt 2
D W= - 3w
8lR 2

BMAT 2007: Section 2 - © Copyright UCLES 2007


6

7 A 100 % efficient transformer has 1500 turns on its primary coil. The input to the
transformer is 250 V ac. The output current is 10 A and the output power is 0.5 kW.

What is the number of turns on the secondary coil?

8 An oxide of iron has the formula Fe3O4 and contains both Fe2+ and Fe3+ ions.

Which one of the following is the fraction of iron ions that are in the Fe2+ state?

1
A /4
1
B /3
1
C /2
2
D /3
3
E /4

9 Which row of the table shows the state of the heart valves when the heart pumps blood to
the lungs?

left side of heart right side of heart


atrio-ventricular atrio-ventricular
semilunar valve semilunar valve
valve (cuspid) valve (cuspid)
A closed closed open open
B closed open closed open
C closed open open closed
D open closed closed open
E open closed open closed
F open open closed closed

BMAT 2007: Section 2 - © Copyright UCLES 2007


7

10 The reaction between lead(II) nitrate solution and potassium iodide solution is shown below:

Pb(NO3)2 (aq) + 2Kl (aq) ! 2KNO3 (aq) + Pbl2 (s)

When the two solutions are mixed in a test tube the height of the lead(II) iodide precipitate
formed can be measured.

Which combination of solutions would produce the greatest height of the precipitate?

lead(II) nitrate solution potassium iodide solution


volume/cm3 concentration/ mol dm-3 volume/cm3 concentration/ mol dm-3
A 5.0 2.0 10.0 2.0
B 2.5 5.0 2.5 5.0
C 7.5 3.0 5.0 5.0
D 5.0 4.0 7.5 5.0

11 Below are four statements about thermal (heat) energy.

1 A substance can lose heat energy without its temperature falling.

2 Heat energy can pass through a vacuum.

3 Steam at 100 oC has more heat energy than the same mass of boiling water
at 100 oC
4 When a container of water is cooled near the top, a convection current is set
up in the water.

Which statements are true?

A 1, 2 and 3

B 2, 3 and 4

C 1, 2 and 4

D 1, 3 and 4

E all of the statements

BMAT 2007: Section 2 - © Copyright UCLES 2007


8

* 12 The box and whisker plot shows some information about members of a health club using a
walking machine. It shows the number of steps that a group of 30 members did in a period
of 10 minutes.

500 800 1100 1400


Use this information to find the probability that when 3 of these 30 members are chosen at
random, they will all have taken between 850 and 1000 steps.

3
A
4
150
B
500
1
C
8
3
D
30
1
E
64

BMAT 2007: Section 2 - © Copyright UCLES 2007


9

* 13 The diagram shows the bones and leg muscles of a person standing on their toes. The
bones and muscles are acting as a lever system.

Which of the following is the correct description of this lever system?

A The fulcrum is between the effort and the load.

B The load is being exerted by the calf muscle.

C The load is between the fulcrum and the effort.

D The effort is exerted by the muscles in the foot.

E The effort is between the load and the fulcrum.

14 Which of the following reactions between a Group 1 element and a Group 7 element would
be the most exothermic?

A caesium and fluorine

B lithium and iodine

C potassium and bromine

D sodium and chlorine

BMAT 2007: Section 2 - © Copyright UCLES 2007


10

* 15 Which diagram shows how a ray of white light is refracted by an air bubble in water, and
gives the correct relative positions of the red and violet light formed by dispersion?
red violet
violet red

A B
white white
AIR AIR

red violet

violet red

C white D white
AIR AIR

white AIR white AIR


E F

red violet
violet red

* 16 The two inequalities given below define an area on a graph.

y ≥ x2 +3
x ≥ 1/ y

Which one of the following points lies within the area defined?

A (-1, -6)

B (2, -1)

C (1, 6)

D (2, 2)

BMAT 2007: Section 2 - © Copyright UCLES 2007


11

17 The below diagram shows a family tree of a condition known as nail patella syndrome
(NPS).

1 2

3 4 5 6

7 8 9

Key

female without NPS male without NPS

female with NPS male with NPS

Which of the following pairs of individuals must be heterozygous for NPS?

A 1 and 5

B 2 and 6

C 3 and 7

D 4 and 8

E 5 and 9

F 6 and 7

BMAT 2007: Section 2 - © Copyright UCLES 2007


12

18 What volume of steam would be produced from an ice cube of mass 6.00 g if it were heated
until it all evaporated?

(Ar values: H = 1; O = 16)

(Take molar volume at room temperature and pressure = 24 dm3)

A 240 cm3

B 1800 cm3

C 4800 cm3

D 8000 cm3

* 19 A heavy uniform bar is in equilibrium in the arrangement shown.

1.5 m 4.5 m

pivot bar
1000 N 100 N

What is the weight of the bar? (Give your answer to the nearest Newton.)

20 The longest side of a right angled triangle is 6 + 5 units.


One of the shorter sides is 3 +2 5 units.

What is the length of the third side?

A 2 3

B 70 + 24 5

C 12

D 3− 5

E 14 + 7.5 5

BMAT 2007: Section 2 - © Copyright UCLES 2007


13

21 A patient with leukaemia is being treated with chemotherapy and radiotherapy. The patient
is found to have a low blood platelet count and a high percentage of the white blood cells
are abnormal.

Which row of the table shows this patient’s symptoms?

disease
blood clotting oxygen transport
resistance
A low low low
B high low high
C low normal low
D high normal normal
E low low normal
F high normal high

22 Some statements about ammonia are given below.

1 Its formula is NH3.

2 The pH of its solution in water is less than 7.

3 It has a molecular structure.

4 It turns damp blue litmus paper red.

5 It is a liquid at room temperature.

6 It is covalently bonded.

Which are correct statements about ammonia?

A 1, 2 and 3

B 2, 4 and 5

C 4, 5 and 6

D 1, 3 and 6

E 1, 3 and 5

F 2, 3 and 4

BMAT 2007: Section 2 - © Copyright UCLES 2007


14
23 An artery has a length x mm and blood flows steadily through it at V ml per second.

If, on average, a given red blood cell takes T seconds to travel the length of the artery, what
is the cross-sectional area of the artery in mm2?

VT
A × 10 −6
x
Vx
B × 10 −6
T
VT
C
x
Vx
D
T
VT
E × 10 3
x
Vx
F × 10 3
T

BMAT 2007: Section 2 - © Copyright UCLES 2007


15

24 The solid shown below consists of a cylinder topped by a hemisphere of the same radius.

Which one of the following correctly gives the volume of this solid?

πr 2
A (2r + 3l )
3
πr 2
B ( 4r + 3l )
3
πr 3
C (2 + 3l )
3
πr 2
D ( 2r + l )
3

25 A mixture of equal parts of hexane (bp 68 oC) and heptane (bp 98 oC) is distilled using a
fractionating column.

The temperature of the liquid in the flask and the temperature at the top of the fractionating
column are measured.

Which one of the following shows the likely temperatures when the first drops of distillate
are collected?

temperature in temperature at top


flask/oC of column/oC
A 83 68
B 98 68
C 83 83
D 98 83

BMAT 2007: Section 2 - © Copyright UCLES 2007


16

26 Which row of the table shows the roles of the female sex hormones in controlling the
menstrual cycle?

causes thickening maintains lining of causes break down


of lining of uterus uterus of lining of uterus
fall in oestrogen low oestrogen rise in progesterone
A
concentration concentration concentration
fall in progesterone low progesterone rise in oestrogen
B
concentration concentration concentration
rise in oestrogen high progesterone fall in progesterone
C
concentration concentration concentration
rise in progesterone high oestrogen fall in progesterone
D
concentration concentration concentration
rise in oestrogen low progesterone rise in progesterone
E
concentration concentration concentration
fall in progesterone high progesterone fall in oestrogen
F
concentration concentration concentration

27 The position of cobalt in the periodic table is as follows:

56 59 59 63.5

25
Mn 26
Fe 27
Co 28
Ni 29
Cu
Cobalt 60 is a radioactive isotope that decays with the emission of beta and gamma
radiation. A beam of radiation from a cobalt 60 source is directed towards a malignant
tumour in a patient’s abdomen.

Which of the following is/are correct when cobalt 60 is used in this way?
1 The cobalt (27Co) is being converted into nickel (28Ni) as it decays.

2 The tumour is attacked by the gamma radiation.

3 The radiation attacks only the malignant cells.

A 1 only

B 2 only

C 3 only

D 1 and 2

E 2 and 3

F 1 and 3

END OF SECTION 2

BMAT 2007: Section 2 - © Copyright UCLES 2007


17

BLANK PAGE

BMAT 2007: Section 2 - © Copyright UCLES 2007


18

BLANK PAGE

BMAT 2007: Section 2 - © Copyright UCLES 2007


19

BLANK PAGE

BMAT 2007: Section 2 - © Copyright UCLES 2007


20

BLANK PAGE

BMAT 2007: Section 2 - © Copyright UCLES 2007


BMAT 2007

Section 2 answer key

Question Answer
1 B
2 B
3 α = 220 β = 40
4 A
5 E
6 A
7 300
8 B
9 B
10 D
11 E
12 E
13 C
14 A
15 B
16 C
17 D
18 D
19 700N
20 A
21 E
22 D
23 E
24 A
25 A
26 C
27 D
BioMedical Admissions Test 4500/02

Wednesday 1 November 2006 Morning 30 minutes

SECTION 2 Scientific Knowledge and Applications

Instructions to Candidates

Please read this page carefully, but do not open the question paper until
you are told that you may do so.

A separate answer sheet is provided for this section. Please check you have one.
You also require a soft pencil and an eraser.

Please first write your name, date of birth, BioMedical Admissions Test candidate
number and centre number in the spaces provided on the answer sheet.
Please write very clearly.

Speed as well as accuracy is important in this section. Work quickly, or you


may not finish the paper. There are no penalties for incorrect responses, only
points for correct answers, so you should attempt all 27 questions.
Unless otherwise stated, all questions are worth one mark.

Answer on the sheet provided. Many questions ask you to show your choice
between options by shading a circle (or circles, if specified in the question).
If questions ask you to write in words or numbers, be sure to write clearly in the
spaces provided. If you make a mistake, erase thoroughly and try again.

Any rough work should be done on this question paper.

Calculators are NOT permitted.

Please wait to be told you may begin before turning this page.

This paper consists of 15 printed pages and 5 blank pages.

The questions in this paper that are marked with an asterisk (* Qs: 6, 19, 25)
assume knowledge that is not currently on the BMAT specification.
2

BLANK PAGE

BMAT 2006: Section 2 - © Copyright UCLES 2006


3

1 The graph shows changes in the mass of glycogen stored in the liver and skeletal muscles.

mass of glycogen

2
3 4 5
1 6

time
During which periods will the concentration of insulin in the blood be below normal?

A 1 and 6
B 2 and 3
C 2 and 4
D 3 and 4
E 3 and 5
F 4 and 5

2 Compound T has a melting point of 78 °C and a boiling point of 134 °C. T is soluble in
water and its solution does not conduct electricity. T has covalent bonding and has a
simple molecular structure.

Which property of T is not usually associated with its bonding and structure?

A the melting point


B the boiling point
C the solubility in water
D the lack of conductivity of the solution

BMAT 2006: Section 2 - © Copyright UCLES 2006


4

3 A particular radioisotope X with a half-life of 4 years decays into the stable isotope Y.

At a particular time, a sample contains 32 x 10²º atoms of nuclide X and 4 x 10²º atoms of
nuclide Y.

How many atoms of nuclide Y will be present in the sample 8 years later?

A 1 x 10²º
B 4 x 10²º
C 8 x 10²º
D 16 x 10²º
E 24 x 10²º
F 28 x 10²º

4 The amount of water in urine is controlled by the hormone ADH.

Which sequence of events will occur when you need to keep water in your body?

blood urine concentration


ADH production nephron action
concentration is: becomes:
A dilute falls less water reabsorbed dilute
B concentrated rises more water reabsorbed concentrated
C dilute rises less water reabsorbed dilute
D concentrated falls more water reabsorbed concentrated
E dilute falls more water reabsorbed concentrated
F concentrated rises less water reabsorbed dilute

5 Listed are the electronic configurations for the atoms of different elements.

Which one represents the most reactive non-metal?

A 2, 8, 7
B 2, 4
C 2, 7
D 2, 8, 1
E 2, 6
F 2, 8, 6

BMAT 2006: Section 2 - © Copyright UCLES 2006


5

* 6 The sides of triangle ABC are as follows:

AB = 3, AC = 2, BC = 4

Use the cosine rule, a 2 = b 2 + c 2 − 2bc cos A , to find the cosine of ∠BAC

1
A −
4

1
B
16

1
C
4

5
D
6

1
E
2

7 The following statements can be applied to certain types of wave:

1 Their oscillations are longitudinal.


2 They travel at the speed of light in air.
3 They are used in pre-natal scanning.
4 They are used in thermal imaging.
5 They will not travel through a vacuum.

Which of these statements can be applied to microwaves?

A 2 only
B 4 only
C 1 and 5 only
D 2 and 4 only
E 1, 3 and 5 only
F 2, 3 and 4 only

BMAT 2006: Section 2 - © Copyright UCLES 2006


6

8 A health club monitors the average heart rate of its members when using a cardiovascular
workout machine. The lower quartile of the heart rate is 115, the upper quartile is 165.

What is the probability that when a group of 4 members is chosen at random at least 3 of
them had an average heart rate of more than 165 when doing this workout?

1
A
256

3
B
64

13
C
256

5
D
16

11
E
16

9 Which changes to the conditions will increase the yield of the salt in the exothermic reaction
shown?

NH₃(g) + HCl(g) ⇋ NH₄Cl(s)

1 Adding a catalyst
2 Adding more ammonia
3 Increasing the pressure
4 Increasing the temperature

A 1 and 2 only
B 2 and 3 only
C 3 and 4 only
D 1 and 4 only
E 2, 3 and 4 only
F 1, 2 and 3 only

BMAT 2006: Section 2 - © Copyright UCLES 2006


7

10 A gene controls the production of chlorophyll in lettuce plants. The recessive allele of this
gene blocks chlorophyll formation which prevents seedling development.

Plants heterozygous for this gene were cross bred and 1000 of their seeds were planted.
Only about 750 developed into mature plants.

What percentage of these mature plants would be heterozygous for this gene?

A 25%
B 33%
C 50%
D 67%
E 75%

11 A force F is applied to the piston X of a brake pedal. This force is transmitted, hydraulically,
to piston Y as shown in the diagram.

F
X Y

Diameter of piston X = 2 cm.


Diameter of piston Y = 20 cm.

The pressure applied at X is 3 N/cm 2 .

What is the pressure transmitted to Y?

A 0.03 N/cm 2
B 0.3 N/cm 2
C 3 N/cm 2
D 30 N/cm 2
E 300 N/cm 2

BMAT 2006: Section 2 - © Copyright UCLES 2006


8

12 In statistics Spearman’s rank correlation coefficient is given by the formula:

6∑ d
2

r = 1−
n(n 2 − 1)

Rearrange the formula to make ∑d 2


the subject.

r ( n 3 − n)
A ∑d = 1− 6
2

2
⎛ (1 − r )(n 3 − n) ⎞
B ∑ d = ⎜⎜
2
⎟⎟
⎝ 6 ⎠

(1 − r )(n 3 − 1)
C ∑d 2 = 6

(1 + r )(n 3 − n)
D ∑d = 2

(1 − r )(n 3 − n)
E ∑d 2 = 6

13 At 200 °C, potassium hydrogencarbonate decomposes according to the following equation.

2KHCO 3 → K 2 CO 3 + H2 O + CO 2

What is the loss in mass when 50.0 g of potassium hydrogencarbonate are heated at
200 °C to constant mass?

( Ar : H = 1; C = 12; O = 16; K = 39 )

A 11.0 g
B 15.5 g
C 22.0 g
D 31.0 g

BMAT 2006: Section 2 - © Copyright UCLES 2006


9

14 Find the positive solution of the following simultaneous equations:

4 x 2 + y 2 + 10 y = 47
2x − y = 5

15 The diagram shows part of the circulatory system.

3
liver

2 1
stomach and
intestines

kidney
5 4

Which row of the table identifies the blood vessels that best fit the descriptions after eating
a carbohydrate rich meal?

concentration of substances in the blood


highest concentration of highest concentration of lowest concentration of
glucose urea oxygen
A 1 5 4
B 5 3 4
C 2 2 3
D 1 5 2
E 2 3 3
F 5 2 2

BMAT 2006: Section 2 - © Copyright UCLES 2006


10

16 An object with a mass of 20 kg is lifted by the arrangement shown in the diagram. Air
resistance (drag) can be ignored, and the gravitational field strength (acceleration due to
gravity) can be taken as 10 N/kg.

pulley pulley
120N 120N

object
(mass = 20 kg)

What is the acceleration of the object, in m/s²?

17 The element has a mass number of 40 and an atomic number of 20.

Consider the statements and decide which of them are correct.

1 The nucleus has a relative mass of 20.


2 It is a noble gas.
3 It would form a negative ion.
4 It is in group 2 of the periodic table.
5 It is a non-metallic element.

A 1, 2 and 3 only
B 5 only
C 2, 3 and 5 only
D 1, 4 and 5 only
E 4 only
F 1, 3 and 4 only

BMAT 2006: Section 2 - © Copyright UCLES 2006


11

18 The table shows some details of the composition of inhaled and exhaled air of a student.

Gas Inhaled air Exhaled air


Carbon dioxide 0.03% 4%
Nitrogen 78% 78%
Oxygen 21% 16%

The student is breathing 500 cm³ of air 14 times per minute.

How many dm³ of oxygen will be absorbed into the lung capillaries in 4 minutes?

A 0.14 dm³
B 0.35 dm³
C 1.4 dm³
D 3.5 dm³
E 35.0 dm³
F 140.0 dm³

* 19 Which row of the table shows the response of the iris to reducing levels of light?

radial muscles circular muscles pupil


A contract relax is contracted
B relax contract is dilated
C contract relax is dilated
D relax contract is contracted

BMAT 2006: Section 2 - © Copyright UCLES 2006


12

20 Five identical resistors are connected to a cell as in the diagram.

R1

R3

R2

The potential differences across R 1 , R 2 and R 3 are V1 , V2 and V3 respectively.

What is the order of increasing potential differences (smallest first)?

A V1 , V2 , V3
B V1 , V3 , V2
C V2 , V1 , V3
D V2 , V3 , V1
E V3 , V1 , V2
F V3 , V2 , V1

21 Evaluate

−1
⎛ 32 15 + 9 0 ⎞
⎜ ⎟
⎜⎜ 3 ⎟⎟
⎝ 81 ⎠
4

BMAT 2006: Section 2 - © Copyright UCLES 2006


13

22 The energy profile for a reaction is shown below.

W
Y
energy Z

progress of reaction

Which of the following is a fully correct statement about this energy level profile?

A The reaction is endothermic, V is the heat of reaction (positive sign) and X is the
activation energy.

B Products (Z) have more energy than the reactants (U), X shows the route taken when
a catalyst is present and W is the activation energy.

C Y is the route taken when a catalyst is present, V is the heat of reaction and the
reaction is exothermic overall.

D X shows the route taken without a catalyst, V + W is the activation energy and V (the
heat of reaction) has a positive sign.

BMAT 2006: Section 2 - © Copyright UCLES 2006


14

23 The diagram shows the dispersion of white light as it passes from air into glass.

normal

air

glass

red
blue
Red light travels at speed c in air, but only 2c/3 in glass. Red light has a wavelength of λ in
air.

What is the frequency of red light, and the speed of blue light in glass?

frequency of red speed of blue


light in glass light in glass
A 2c/3λ < 2c/3
B 2c/3λ > 2c/3
C c/λ < 2c/3
D c/λ > 2c/3
E 3c/2λ < 2c/3
F 3c/2λ > 2c/3

BMAT 2006: Section 2 - © Copyright UCLES 2006


15

24 The statements are about respiration.

1 Aerobic respiration releases more energy per unit mass of glucose than anaerobic
respiration.
2 Aerobic respiration in muscles causes an oxygen debt to occur.
3 Aerobic respiration forms carbon dioxide as the only waste product.
4 Reduced aerobic respiration will limit the active uptake of mineral ions in the
kidney tubules.
5 When there is little oxygen available to muscles only anaerobic respiration occurs.
6 Anaerobic respiration forms lactic acid as the only waste product.

Which statements are correct?

A 1, 2 and 3 only
B 4, 5 and 6 only
C 1, 3 and 5 only
D 2, 4 and 6 only
E 1, 4 and 6 only
F 2, 3 and 5 only

* 25 In stars four protons, p, are fused together to form helium, 4 He , with a mass of four units,
by the following processes.

2p → 2 H + e +
2 2 H + p → 3 He
2 3 He → 4 He + 2p

Which one of the following shows the overall change that has taken place when 4 He is
formed?

( e+ is a positively charged electron, n is a neutron, 2 H is a hydrogen of mass two and


3
He is helium of mass three.)

A 1p → 1n + e+
B 2p → 2n + 2e+
C 3p → 3n + 3e+
D 4p → 4n + 4e+

BMAT 2006: Section 2 - © Copyright UCLES 2006


16

26 a is inversely proportional to the square of b.

When a = 9 , b = 4 .

What is the positive value of b when a = 4 ?

A 6

B 9

C 20.25

D 36

27 A small conducting sphere is held at a fixed distance from a larger sphere which is
continually charged by a generator.

sphere

generator

2
The surface area of the large sphere is 0.04 m , and it discharges by sparking to the small
sphere when the charge density on the large sphere reaches 0.25 C/m².

charge density = charge per unit surface area

If the large sphere is charged at a rate of 2 mA, calculate the time interval (in seconds)
between sparks.

END OF SECTION 2

BMAT 2006: Section 2 - © Copyright UCLES 2006


17

BLANK PAGE

BMAT 2006: Section 2 - © Copyright UCLES 2006


18

BLANK PAGE

BMAT 2006: Section 2 - © Copyright UCLES 2006


19

BLANK PAGE

BMAT 2006: Section 2 - © Copyright UCLES 2006


20

BLANK PAGE

BMAT 2006: Section 2 - © Copyright UCLES 2006


BMAT 2006 answer keys

Section 2

Question No. Answer


1 E
2 C
3 F
4 B
5 C
6 A
7 A
8 C
9 B
10 D
11 C
12 E
13 B
14 x=3 y=1
15 E
16 2 m/s2
17 E
18 C
19 C
20 E
21 9
22 D
23 C
24 E
25 B
26 A
27 5 seconds
BioMedical Admissions Test 4500/02

Wednesday 1 November 2006 Morning 30 minutes

SECTION 2 Scientific Knowledge and Applications

Instructions to Candidates

Please read this page carefully, but do not open the question paper until
you are told that you may do so.

A separate answer sheet is provided for this section. Please check you have one.
You also require a soft pencil and an eraser.

Please first write your name, date of birth, BioMedical Admissions Test candidate
number and centre number in the spaces provided on the answer sheet.
Please write very clearly.

Speed as well as accuracy is important in this section. Work quickly, or you


may not finish the paper. There are no penalties for incorrect responses, only
points for correct answers, so you should attempt all 27 questions.
Unless otherwise stated, all questions are worth one mark.

Answer on the sheet provided. Many questions ask you to show your choice
between options by shading a circle (or circles, if specified in the question).
If questions ask you to write in words or numbers, be sure to write clearly in the
spaces provided. If you make a mistake, erase thoroughly and try again.

Any rough work should be done on this question paper.

Calculators are NOT permitted.

Please wait to be told you may begin before turning this page.

This paper consists of 15 printed pages and 5 blank pages.

The questions in this paper that are marked with an asterisk (* Qs: 6, 19, 25)
assume knowledge that is not currently on the BMAT specification.
2

BLANK PAGE

BMAT 2006: Section 2 - © Copyright UCLES 2006


3

1 The graph shows changes in the mass of glycogen stored in the liver and skeletal muscles.

mass of glycogen

2
3 4 5
1 6

time
During which periods will the concentration of insulin in the blood be below normal?

A 1 and 6
B 2 and 3
C 2 and 4
D 3 and 4
E 3 and 5
F 4 and 5

2 Compound T has a melting point of 78 °C and a boiling point of 134 °C. T is soluble in
water and its solution does not conduct electricity. T has covalent bonding and has a
simple molecular structure.

Which property of T is not usually associated with its bonding and structure?

A the melting point


B the boiling point
C the solubility in water
D the lack of conductivity of the solution

BMAT 2006: Section 2 - © Copyright UCLES 2006


4

3 A particular radioisotope X with a half-life of 4 years decays into the stable isotope Y.

At a particular time, a sample contains 32 x 10²º atoms of nuclide X and 4 x 10²º atoms of
nuclide Y.

How many atoms of nuclide Y will be present in the sample 8 years later?

A 1 x 10²º
B 4 x 10²º
C 8 x 10²º
D 16 x 10²º
E 24 x 10²º
F 28 x 10²º

4 The amount of water in urine is controlled by the hormone ADH.

Which sequence of events will occur when you need to keep water in your body?

blood urine concentration


ADH production nephron action
concentration is: becomes:
A dilute falls less water reabsorbed dilute
B concentrated rises more water reabsorbed concentrated
C dilute rises less water reabsorbed dilute
D concentrated falls more water reabsorbed concentrated
E dilute falls more water reabsorbed concentrated
F concentrated rises less water reabsorbed dilute

5 Listed are the electronic configurations for the atoms of different elements.

Which one represents the most reactive non-metal?

A 2, 8, 7
B 2, 4
C 2, 7
D 2, 8, 1
E 2, 6
F 2, 8, 6

BMAT 2006: Section 2 - © Copyright UCLES 2006


5

* 6 The sides of triangle ABC are as follows:

AB = 3, AC = 2, BC = 4

Use the cosine rule, a 2 = b 2 + c 2 − 2bc cos A , to find the cosine of ∠BAC

1
A −
4

1
B
16

1
C
4

5
D
6

1
E
2

7 The following statements can be applied to certain types of wave:

1 Their oscillations are longitudinal.


2 They travel at the speed of light in air.
3 They are used in pre-natal scanning.
4 They are used in thermal imaging.
5 They will not travel through a vacuum.

Which of these statements can be applied to microwaves?

A 2 only
B 4 only
C 1 and 5 only
D 2 and 4 only
E 1, 3 and 5 only
F 2, 3 and 4 only

BMAT 2006: Section 2 - © Copyright UCLES 2006


6

8 A health club monitors the average heart rate of its members when using a cardiovascular
workout machine. The lower quartile of the heart rate is 115, the upper quartile is 165.

What is the probability that when a group of 4 members is chosen at random at least 3 of
them had an average heart rate of more than 165 when doing this workout?

1
A
256

3
B
64

13
C
256

5
D
16

11
E
16

9 Which changes to the conditions will increase the yield of the salt in the exothermic reaction
shown?

NH₃(g) + HCl(g) ⇋ NH₄Cl(s)

1 Adding a catalyst
2 Adding more ammonia
3 Increasing the pressure
4 Increasing the temperature

A 1 and 2 only
B 2 and 3 only
C 3 and 4 only
D 1 and 4 only
E 2, 3 and 4 only
F 1, 2 and 3 only

BMAT 2006: Section 2 - © Copyright UCLES 2006


7

10 A gene controls the production of chlorophyll in lettuce plants. The recessive allele of this
gene blocks chlorophyll formation which prevents seedling development.

Plants heterozygous for this gene were cross bred and 1000 of their seeds were planted.
Only about 750 developed into mature plants.

What percentage of these mature plants would be heterozygous for this gene?

A 25%
B 33%
C 50%
D 67%
E 75%

11 A force F is applied to the piston X of a brake pedal. This force is transmitted, hydraulically,
to piston Y as shown in the diagram.

F
X Y

Diameter of piston X = 2 cm.


Diameter of piston Y = 20 cm.

The pressure applied at X is 3 N/cm 2 .

What is the pressure transmitted to Y?

A 0.03 N/cm 2
B 0.3 N/cm 2
C 3 N/cm 2
D 30 N/cm 2
E 300 N/cm 2

BMAT 2006: Section 2 - © Copyright UCLES 2006


8

12 In statistics Spearman’s rank correlation coefficient is given by the formula:

6∑ d
2

r = 1−
n(n 2 − 1)

Rearrange the formula to make ∑d 2


the subject.

r ( n 3 − n)
A ∑d = 1− 6
2

2
⎛ (1 − r )(n 3 − n) ⎞
B ∑ d = ⎜⎜
2
⎟⎟
⎝ 6 ⎠

(1 − r )(n 3 − 1)
C ∑d 2 = 6

(1 + r )(n 3 − n)
D ∑d = 2

(1 − r )(n 3 − n)
E ∑d 2 = 6

13 At 200 °C, potassium hydrogencarbonate decomposes according to the following equation.

2KHCO 3 → K 2 CO 3 + H2 O + CO 2

What is the loss in mass when 50.0 g of potassium hydrogencarbonate are heated at
200 °C to constant mass?

( Ar : H = 1; C = 12; O = 16; K = 39 )

A 11.0 g
B 15.5 g
C 22.0 g
D 31.0 g

BMAT 2006: Section 2 - © Copyright UCLES 2006


9

14 Find the positive solution of the following simultaneous equations:

4 x 2 + y 2 + 10 y = 47
2x − y = 5

15 The diagram shows part of the circulatory system.

3
liver

2 1
stomach and
intestines

kidney
5 4

Which row of the table identifies the blood vessels that best fit the descriptions after eating
a carbohydrate rich meal?

concentration of substances in the blood


highest concentration of highest concentration of lowest concentration of
glucose urea oxygen
A 1 5 4
B 5 3 4
C 2 2 3
D 1 5 2
E 2 3 3
F 5 2 2

BMAT 2006: Section 2 - © Copyright UCLES 2006


10

16 An object with a mass of 20 kg is lifted by the arrangement shown in the diagram. Air
resistance (drag) can be ignored, and the gravitational field strength (acceleration due to
gravity) can be taken as 10 N/kg.

pulley pulley
120N 120N

object
(mass = 20 kg)

What is the acceleration of the object, in m/s²?

17 The element has a mass number of 40 and an atomic number of 20.

Consider the statements and decide which of them are correct.

1 The nucleus has a relative mass of 20.


2 It is a noble gas.
3 It would form a negative ion.
4 It is in group 2 of the periodic table.
5 It is a non-metallic element.

A 1, 2 and 3 only
B 5 only
C 2, 3 and 5 only
D 1, 4 and 5 only
E 4 only
F 1, 3 and 4 only

BMAT 2006: Section 2 - © Copyright UCLES 2006


11

18 The table shows some details of the composition of inhaled and exhaled air of a student.

Gas Inhaled air Exhaled air


Carbon dioxide 0.03% 4%
Nitrogen 78% 78%
Oxygen 21% 16%

The student is breathing 500 cm³ of air 14 times per minute.

How many dm³ of oxygen will be absorbed into the lung capillaries in 4 minutes?

A 0.14 dm³
B 0.35 dm³
C 1.4 dm³
D 3.5 dm³
E 35.0 dm³
F 140.0 dm³

* 19 Which row of the table shows the response of the iris to reducing levels of light?

radial muscles circular muscles pupil


A contract relax is contracted
B relax contract is dilated
C contract relax is dilated
D relax contract is contracted

BMAT 2006: Section 2 - © Copyright UCLES 2006


12

20 Five identical resistors are connected to a cell as in the diagram.

R1

R3

R2

The potential differences across R 1 , R 2 and R 3 are V1 , V2 and V3 respectively.

What is the order of increasing potential differences (smallest first)?

A V1 , V2 , V3
B V1 , V3 , V2
C V2 , V1 , V3
D V2 , V3 , V1
E V3 , V1 , V2
F V3 , V2 , V1

21 Evaluate

−1
⎛ 32 15 + 9 0 ⎞
⎜ ⎟
⎜⎜ 3 ⎟⎟
⎝ 81 ⎠
4

BMAT 2006: Section 2 - © Copyright UCLES 2006


13

22 The energy profile for a reaction is shown below.

W
Y
energy Z

progress of reaction

Which of the following is a fully correct statement about this energy level profile?

A The reaction is endothermic, V is the heat of reaction (positive sign) and X is the
activation energy.

B Products (Z) have more energy than the reactants (U), X shows the route taken when
a catalyst is present and W is the activation energy.

C Y is the route taken when a catalyst is present, V is the heat of reaction and the
reaction is exothermic overall.

D X shows the route taken without a catalyst, V + W is the activation energy and V (the
heat of reaction) has a positive sign.

BMAT 2006: Section 2 - © Copyright UCLES 2006


14

23 The diagram shows the dispersion of white light as it passes from air into glass.

normal

air

glass

red
blue
Red light travels at speed c in air, but only 2c/3 in glass. Red light has a wavelength of λ in
air.

What is the frequency of red light, and the speed of blue light in glass?

frequency of red speed of blue


light in glass light in glass
A 2c/3λ < 2c/3
B 2c/3λ > 2c/3
C c/λ < 2c/3
D c/λ > 2c/3
E 3c/2λ < 2c/3
F 3c/2λ > 2c/3

BMAT 2006: Section 2 - © Copyright UCLES 2006


15

24 The statements are about respiration.

1 Aerobic respiration releases more energy per unit mass of glucose than anaerobic
respiration.
2 Aerobic respiration in muscles causes an oxygen debt to occur.
3 Aerobic respiration forms carbon dioxide as the only waste product.
4 Reduced aerobic respiration will limit the active uptake of mineral ions in the
kidney tubules.
5 When there is little oxygen available to muscles only anaerobic respiration occurs.
6 Anaerobic respiration forms lactic acid as the only waste product.

Which statements are correct?

A 1, 2 and 3 only
B 4, 5 and 6 only
C 1, 3 and 5 only
D 2, 4 and 6 only
E 1, 4 and 6 only
F 2, 3 and 5 only

* 25 In stars four protons, p, are fused together to form helium, 4 He , with a mass of four units,
by the following processes.

2p → 2 H + e +
2 2 H + p → 3 He
2 3 He → 4 He + 2p

Which one of the following shows the overall change that has taken place when 4 He is
formed?

( e+ is a positively charged electron, n is a neutron, 2 H is a hydrogen of mass two and


3
He is helium of mass three.)

A 1p → 1n + e+
B 2p → 2n + 2e+
C 3p → 3n + 3e+
D 4p → 4n + 4e+

BMAT 2006: Section 2 - © Copyright UCLES 2006


16

26 a is inversely proportional to the square of b.

When a = 9 , b = 4 .

What is the positive value of b when a = 4 ?

A 6

B 9

C 20.25

D 36

27 A small conducting sphere is held at a fixed distance from a larger sphere which is
continually charged by a generator.

sphere

generator

2
The surface area of the large sphere is 0.04 m , and it discharges by sparking to the small
sphere when the charge density on the large sphere reaches 0.25 C/m².

charge density = charge per unit surface area

If the large sphere is charged at a rate of 2 mA, calculate the time interval (in seconds)
between sparks.

END OF SECTION 2

BMAT 2006: Section 2 - © Copyright UCLES 2006


17

BLANK PAGE

BMAT 2006: Section 2 - © Copyright UCLES 2006


18

BLANK PAGE

BMAT 2006: Section 2 - © Copyright UCLES 2006


19

BLANK PAGE

BMAT 2006: Section 2 - © Copyright UCLES 2006


20

BLANK PAGE

BMAT 2006: Section 2 - © Copyright UCLES 2006


Biomedical Admissions Test - 2005

ANSWER KEYS, SECTION 2

Question Correct response Comments Maximum


number points
available for
this question
1 A 1
2 B 1
3 D 1
4 C 1
5 D 1
6 C 1
7 A 1
8 A 1
9 C 1
10 39 cm 1
11 D 1
12 B 1
13 A 1
14 B 1
15 D 1
16 i D all three correct for 1 mark 1
ii C
iii G
17 A 1
18 B 1
19 B 1
20 C 1
21 A 1
22 D 1
23 B 1
24 A 1
25 B 1
26 D 1
27 0.8 1
University of Cambridge Local Examinations Syndicate

Biomedical Admissions Test

Wednesday 3 November 2004 Morning 30 minutes

SECTION 2 Scientific Knowledge and Applications

Instructions to Candidates

Please read this page carefully, but do not open the question paper until
you are told that you may do so.

A separate answer sheet is provided for this section. Please check you have one.
You also require a soft pencil and an eraser.

Please first write your name, date of birth, Biomedical Admissions Test candidate
number and centre number in the spaces provided on the answer sheet. Please
write very clearly.

Please also write your BMAT candidate number in the spaces below.
BMAT Candidate Number

Speed as well as accuracy is important in this section. Work quickly, or you


may not finish the paper. There are no penalties for incorrect responses, only
points for correct answers, so you should attempt all 27 questions.
Unless otherwise stated, all questions are worth one mark.

Answer on the sheet provided. Many questions ask you to show your choice
between options by shading a circle (or circles, if specified in the question).
If questions ask you to write in words or numbers, be sure to write clearly in the
spaces provided. If you make a mistake, erase thoroughly and try again.

Any rough work should be done on this question paper.

Calculators are NOT permitted.

Please wait to be told you may begin before turning this page.

This paper consists of 12 printed pages and 4 blank pages.

The questions in this paper that are marked with an asterisk (* Qs: 4, 5, 10, 11, 13, 21, 23, 24, 26)
assume knowledge that is not currently on the BMAT specification.
2

BLANK PAGE

BMAT 2004: Section 2 - © Copyright UCLES 2004


3

1 The diagram shows a view from the front of a section through the heart and associated blood
vessels.

Which sequence of numbers shows the course of blood flow from the point of entry to the
heart from the lungs, to its eventual exit from the heart to supply the lungs?

A 2→3→4→8→1→5→6→7
B 2→3→4→1→7→6→5→8
C 7→6→5→8→2→3→4→1
D 7→6→5→1→8→4→3→2
E 8→5→6→7→2→3→4→1
F 8→5→6→7→1→4→3→2

2 The right-angled triangle shown has horizontal and vertical sides measuring (4 + √2) cm and
(2 - √2) cm respectively.

(2 - √2) cm

(4 + √2) cm
(not to scale)
Calculate the area of the triangle.

A (5 + 3√2) cm2
B (3 − √2) cm2
C (3 + 3√2) cm2
D (5 − √2) cm2

BMAT 2004: Section 2 - © Copyright UCLES 2004


4
3 The equation summarises the reaction of copper and dilute nitric acid.

q Cu + r HNO3 → s Cu(NO3)2 + 6 H2O + t NO2

What values of q, r, s and t are needed to balance the equation?

* 4 The diagram shows a simplified version of the brake pedal of a car.

pivot
4 cm
piston 2 cm to brakes

16 cm

6 cm
60 N

A driver applies a force of 60 N to the pedal. What force is applied to the piston?

A 15 N
B 20 N
C 180 N
D 240 N
E 300 N
F 540 N

* 5 During the electrolysis of hydrochloric acid, positively charged hydrogen ions move towards
the cathode at a rate of 1018 ions per second. Negatively charged chloride ions move
towards the anode at the same rate. Both hydrogen and chloride ions carry charges of
magnitude 1.6 × 10-19 C.

What is the resulting current flow between the electrodes?

BMAT 2004: Section 2 - © Copyright UCLES 2004


5
6 The diagram shows a section through the thorax.

When breathing out (expiring), what happens to the muscles at X and Y


and the pressure at Z?

muscle X muscle Y pressure Z


A contracts contracts increases
B contracts relaxes increases
C relaxes contracts decreases
D relaxes relaxes decreases
E relaxes relaxes increases

7 The table shows some properties of five substances A - E.

melting point boiling point electrical conductivity


substance
°C °C when solid when molten
A 800 1470 none good
B 98 880 good good
C -20 58 none none
D 114 444 none none
E 1700 2200 none none

From the substances in the table, select the substances that could be:

i) an inorganic substance with a giant molecular structure;


ii) a metal;
iii) an ionic compound;
iv) a liquid at room temperature and atmospheric pressure.

BMAT 2004: Section 2 - © Copyright UCLES 2004


6
8 An oxide of tungsten contains 79.31% by mass of tungsten.

What is the formula of this oxide? (relative atomic masses: O = 16; W = 184)

A WO
B W2O3
C WO2
D W3O4
E WO3

9 A ball is thrown vertically upwards and leaves the thrower’s hand with a speed of 12 m/s.
Calculate the height to which it rises. You may assume that all of the initial kinetic energy
of the ball has been converted into gravitational potential energy when the ball reaches its
highest point. (Take the value of g to be 10 N/kg.)

* 10 The diagram shows two sets of parallel lines. Angles θ, α, β and γ are as marked.

θ
γ

β
α

Which of the following statements are true? (shade all that apply)

A Cos α = − Cos θ
B Sin β = − Sin θ
C Tan γ = − Tan θ
D Sin (θ + α) = 1

BMAT 2004: Section 2 - © Copyright UCLES 2004


7
* 11 One of the steps in the breakdown of glucose is the conversion of pyruvic acid,
CH3COCO2H, into lactic acid, CH3CH(OH)CO2H.

This is an example of:

A oxidation
B reduction
C hydration
D acidification

12 The diagram shows a family tree.

1 2

Key
3 4 5 6 7 female
male

8 9 10 11

The grandfather, individual 1, carries a recessive allele on his X chromosome.


Which other individuals could have inherited this allele?

A 3,4,5,6
B 3,4,5,7
C 4,5,6,8
D 4,5,8,9
E 5,8,9,10
F 6,8,9,11

* 13 Solve the inequality


x2 ≥ 8 – 2x
A x≥4
B x ≤ 2 and x ≥ -4
C x ≥ -2 and x ≤ 4
D x ≥ 2 or x ≤ -4

BMAT 2004: Section 2 - © Copyright UCLES 2004


8
14 Plugs of cotton wool containing concentrated aqueous ammonia and concentrated
hydrochloric acid are placed at opposite ends of a tube, at room temperature.
A white ring of solid ammonium chloride forms nearer the hydrochloric acid.

concentrated white concentrated


aqueous ammonia ring hydrochloric acid

Which of the following can be deduced from this experiment?


A The ammonia molecule has more atoms than the hydrogen chloride molecule.
B The boiling point of ammonia is less than that of hydrogen chloride.
C The concentration of the aqueous ammonia is less than that of the acid.
D The mass of an ammonia molecule is less than that of a hydrogen chloride molecule.
E The reactivity of ammonia is less than that of hydrogen chloride.

15 In these circuits the cells are identical and their internal resistances can be ignored.

S3

S1

V1
A2

A3

A1 S2

Indicate whether each of the statements below is true or false for the circuits shown.

i) When switch S1 is closed, the reading on voltmeter V1 will increase.


ii) When switch S1 is closed, the reading on ammeter A1 will increase.
iii) When switch S2 is closed, the reading on ammeter A2 will increase.
iv) When switch S2 is closed, the reading on ammeter A3 will decrease.
v) When switch S3 is closed, the reading on ammeter A2 will increase.

BMAT 2004: Section 2 - © Copyright UCLES 2004


9
16 In 2001 a quarter of the population of Wales reported having a limiting long-term illness or
disability which restricted their daily activities.

What is the probability that out of a random group of three people who live in Wales,
exactly one of them will have reported such an illness or disability?

9
A
64
27
B
64
37
C
64
3
D
4

17 A parachutist of total mass 90 kg (including the parachute) is falling at a terminal velocity of


6 m/s. Take the force due to gravity on a mass of 1 kg to be 10 N.
What is the magnitude of the air resistance force acting on her?

A zero
B 150 N
C 540 N
D 900 N
E 5400 N

18 The general formula for the alkane series of hydrocarbons is CnH2n+2

The primary amine series has the same saturated structure as the alkanes, but also contains
an –NH2 group.

What is the general formula for the primary amine series?

A CnH2nN
B CnH2n+1N
C CnH2n+2N
D CnH2n+3N

BMAT 2004: Section 2 - © Copyright UCLES 2004


10
19 The diagram shows three sperm and three ova containing sex chromosomes X and Y (not
drawn to scale).
ova sperm

A B C D E F

X Y XY
X Y XY

Indicate the letters (A to F) of all the sex cells which are normal.

20 Which one of the following is not a measure of electrical power?


(V = potential difference, I = current, Q = charge, t = time, R = resistance)

QV
A
t
B I 2R
Q2R
C
t
D VI
V2
E
R

* 21 Which blood vessel will have capillaries at both ends?

A Anterior vena cava


B Aorta
C Hepatic artery
D Hepatic portal vein
E Renal vein

BMAT 2004: Section 2 - © Copyright UCLES 2004


11
22 Nuclide P decays by emission of ionising radiation to produce nuclide Q. This new nuclide
then decays by further emission into nuclide R. The process is shown below, with the
appropriate mass numbers (nucleon numbers) and atomic numbers (proton numbers).

A
Z P→ Z +A1 Q→ Z −X1 R
Which line in the table shows the type of particle emitted at each stage, and the value of X?

first decay second decay value X


A alpha alpha A
B alpha alpha A–2
C alpha beta A–4
D beta beta A
E beta alpha A–2
F beta alpha A–4

* 23 In water, the Fe2+ ion bonds with water to form Fe(H2O)62+. In haemoglobin in the blood,
five of the water molecules have been displaced. In the presence of air, the last water
molecule is displaced by oxygen to give oxyhaemoglobin. In the presence of carbon
monoxide, the oxygen is displaced to give carboxyhaemoglobin.

Which of the following shows the correct order of the strength of the bonding to the
Fe2+ ion (weakest first)?

weakest strongest
A carbon monoxide, water, oxygen
B carbon monoxide, oxygen, water
C oxygen, water, carbon monoxide
D oxygen, carbon monoxide, water
E water, oxygen, carbon monoxide
F water, carbon monoxide, oxygen

BMAT 2004: Section 2 - © Copyright UCLES 2004


12
* 24 The diagram shows the leg muscles of an athlete leaving the starting blocks for a race.

position 1 position 2
Q

P R

Which muscles contract and which relax to bring about the change from position 1 to
position 2?

muscles that contract muscles that relax


A P, Q, R S, T
B T, P Q, R, S
C Q, R, T P, S
D R, S P, Q, T
E Q, R, S P, T

25 For a rigid body performing small oscillations about a fixed horizontal axis, the period of
oscillations is given by

T = 2π
(k
+ h2 2
)
gh
where h is the distance of the centre of mass from the axis, g is the acceleration due to
gravity, and k is the radius of gyration of the body about a parallel axis through the centre of
mass.

Rearrange the formula to make k the subject.

2
 T 
A k =  gh − h
 2π 

 T 2 g 2h2 2
B k =  − h 
 4 π 2

C k= (T − 2π )2 gh − h 2
 T 2 gh 
D k =  − h 2 
 4π
2

BMAT 2004: Section 2 - © Copyright UCLES 2004


13
* 26 The list below describes some events that occur during nuclear division.

Indicate, by shading the appropriate circles on the answer sheet, which events occur during
mitosis and which occur during meiosis.

i daughter nuclei are identical to parent nucleus


ii all chromosomes are replicated
iii leads to the formation of gametes
iv two haploid nuclei are formed
v genetic material appears as distinct chromosomes

27 The sum of the roots of a quadratic equation is 7, the product of the roots is 9. What is the
equation?

A x2 + 7x + 9 = 0
B x2 + 7x − 9 = 0
C x2 − 7x + 9 = 0
D x2 − 7x − 9 = 0

END OF SECTION 2

BMAT 2004: Section 2 - © Copyright UCLES 2004


14

BLANK PAGE

BMAT 2004: Section 2 - © Copyright UCLES 2004


15

BLANK PAGE

BMAT 2004: Section 2 - © Copyright UCLES 2004


16

BLANK PAGE

BMAT 2004: Section 2 - © Copyright UCLES 2004


Biomedical Admissions Test - 2004

ANSWER KEYS, SECTION 2

Question Correct response Comments Maximum


number points
available for
this question
1 B 1
2 B 1
3 q = 3; r = 12; s = 3; t = 6 all correct for 1 mark 1
4 E 1
5 0.32 A 1
6 E 1
7 i) E all correct for 1 mark 1
ii) B
iii) A
iv) C
8 E 1
9 7.2 m 1
10 A&B both correct and no others for 1 mark 1
11 B 1
12 D 1
13 D 1
14 D 1
15 i) false all correct for 1 mark 1
ii) false
iii) true
iv) false
v) false
16 B 1
17 D 1
18 D 1
19 A, D & E all correct and no others for 1 mark 1
20 C 1
21 D 1
22 F 1
23 E 1
24 C 1
25 D 1
26 mitosis meiosis 4 or 5 correct for 1 mark 1
i 9 8
ii 9 9
iii 8 9
iv 8 
v 9 9
27 C 1
University of Cambridge Local Examinations Syndicate

Biomedical Admissions Test


Wednesday 5 November 2003 Morning 30 minutes

SECTION 2 Scientific Knowledge and Applications

Instructions to Candidates

Please read this page carefully, but do not open the question paper until
you are told that you may do so.

A separate answer sheet is provided for this section. Please check you have one.
You also require a soft pencil and an eraser.

Please first write your name, date of birth, Biomedical Admissions Test candidate
number and UCAS number (if known) in the spaces provided on the answer
sheet. Please write very clearly.

Please also write your BMAT candidate number in the space below.
BMAT Candidate Number

Speed as well as accuracy is important in this section. Work quickly, or you


may not finish the paper. There are no penalties for incorrect responses, only
points for correct answers, so you should attempt all 29 questions.
Unless otherwise stated, all questions are worth one mark.

Answer on the sheet provided. Many questions ask you to show your choice
between options by shading a circle (or circles, if specified in the question).
If questions ask you to write in words or numbers, be sure to write clearly in the
spaces provided. If you make a mistake, erase thoroughly and try again.

Any rough work should be done on this question paper.

Calculators are NOT permitted.

Please wait to be told you may begin before turning this page.

This paper consists of 13 printed pages and 3 blank pages.

The questions in this paper that are marked with an asterisk (* Qs: 4, 8, 13, 19, 21, 22)
assume knowledge that is not currently on the BMAT specification.
© UCLES 2003
2

BLANK PAGE

BMAT 2003: Section 2

© UCLES 2003
3

1 The table below shows the proportions of undigested and digested carbohydrates, fats and
proteins in three regions of the digestive system.

type of
nutrient mouth stomach small intestine

1 Key

2 undigested food
3 digested food

Which row of the table below correctly identifies the types of nutrient 1, 2 and 3?

1 2 3
A protein fat starch
B starch protein fat
C protein starch fat
D fat protein starch
E starch fat protein
F fat starch protein

2 The mass of an atom of uranium is 4 ´ 10-25 kg.

What is the mass, in milligrams, of 8 million atoms of uranium?

A 3.2 ´ 10-18
B 3.2 ´ 10-17
C 3.2 ´ 10-16
D 3.2 ´ 10-15
E 3.2 ´ 10-12

3 What values of a, b and c are needed to balance the equation?

aC3H6 + bO2 ® 6CO2 + cH2O

BMAT 2003: Section 2

© UCLES 2003
4
* 4 The diagram shows a uniform beam pivoted at its centre. The bar weighs 800N, and three
other forces act on it as shown. Distance x can be varied.

40cm
20cm
x

200N 200N
500N
What value of distance x would cause the beam to be balanced?

A 5 cm
B 10 cm
C 20 cm
D 30 cm
E 40 cm

5 The colours of three indicators are shown.

indicator colour at pH at which colour


low pH high pH change takes place
methyl orange red yellow 4.0
bromothymol blue yellow blue 6.5
phenolphthalein colourless pink 9.0

Equal volumes of these three indicators were mixed and the mixture was added to a solution
of pH 5.0. What colour would be seen?

A blue
B green
C orange
D yellow

6 A horse of weight 6000 N gallops at a speed of 16 m/s. Taking the acceleration of free fall
as 10 m/s2, calculate the kinetic energy of the horse. (Give your answer in kJ.)

BMAT 2003: Section 2

© UCLES 2003
5
7 The diagram shows the menstrual (oestrus) cycle for a human female who is not pregnant.

menstruation

D A

C B

ovulation

During which period, A, B, C or D, will the concentration of oestrogen reach its highest
level?

* 8 Three resistors of equal value are connected up as shown.

A B C D

Arrange the letters corresponding to the four combinations in increasing order of resistance
(least resistance first).

9 Ammonia is manufactured from the reaction between nitrogen and hydrogen.

N2 + 3 H2 ⇌ 2 NH3

What is the maximum mass of ammonia that can be obtained from a mixture of 56 g of
nitrogen with 9 g of hydrogen? (Relative atomic masses: H = 1; N = 14.)

A 34 g
B 51 g
C 65 g
D 68 g

BMAT 2003: Section 2

© UCLES 2003
6
10 Which one of these graphs could be a sketch of the graph of the function y = 2x ?

y y

A B

x x

y y

C D

x x

11 Three of the characteristic phenomena demonstrated by waves are reflection, refraction and
diffraction.
How many of these phenomena cause the frequency of the wave to change?

A 0
B 1
C 2
D 3

12 In the citric acid cycle, succinic acid, C4H6O4, is converted into fumaric acid,
C4H4O4. This is an example of which one of the following chemical changes?

A oxidation
B reduction
C neutralisation
D dehydration

BMAT 2003: Section 2

© UCLES 2003
7
* 13 Solve the inequality
x2 ³ 8 – 2x
A x³4
B x £ 2 and x ³ -4
C x ³ -2 and x £ 4
D x ³ 2 or x £ -4

14 The diagram shows the inheritance of a condition in a family.

1 2

Key
female unaffected
3 4 5 6 female with condition
male unaffected
male with condition
7 8

(i) Identify the three individuals who must be heterozygous for this condition.

(ii) What are the chances that a second child of individuals 3 and 4 will be a female who
has this condition?
A 1 in 1 (100%)
B 1 in 2 (50%)
C 1 in 3 (33.3%)
D 1 in 4 (25%)
E 1 in 8 (12.5%)
[2 marks]

15 In an experiment concerning radioactive decay, the count rate of radiation 5 cm from a


source X was measured as 140 counts per minute. 12 minutes later, with the detector in the
same position, the count rate was measured as 35 counts per minute.

Background radiation was recorded as 20 counts per minute.

Calculate the half-life of source X. (Give your answer in minutes.)

BMAT 2003: Section 2

© UCLES 2003
8
16 The graph shows the solubility of potassium chlorate(V).

80

solubility (g per 100g water) 70

60

50

40

30

20
0 10 20 30 40 50 60 70 80
temperature ( C)

A sample of potassium chlorate(V) weighing 80 g was added to 200 g of water at 70 °C.


The solution was allowed to cool to 20 °C.
What mass of crystals was formed?

A 80 g
B 40 g
C 26 g
D 17 g
E 13 g

17 Which one of the following is not equal to volts?

joules
A
coulombs
B amperes ´ ohms
watts
C
amperes

D watts ´ ohms

watts
E
coulombs

BMAT 2003: Section 2

© UCLES 2003
9
18 The antidiuretic hormone (ADH) is produced by the pituitary gland and its target organ is
the kidney. ADH leaves the head in the jugular vein.
In what order will the hormone pass through the following blood vessels to reach its target?

A aorta
B pulmonary artery
C pulmonary vein
D renal artery
E vena cava

* 19 The energy in joules (E) associated with a photon of radiation is related to its frequency in
hertz (f) by the equation: f = E/h where h is a constant (6.63 ´ 10-34). What is the energy of a
photon of radiation if 5 waves of this radiation are produced in 1 ´ 10-13 sec?

A 1.33 ´ 10-21 J
B 6.63 ´ 10-21 J
C 3.32 ´ 10-20 J
D 1.51 ´ 1046 J
E 7.54 ´ 1046 J

20 The electronic configuration of an isolated, non-ionised, atom is shown below:

In which group and in which period of the periodic table is this atom situated?

group period
A 2 4
B 2 6
C 4 2
D 4 6
E 6 2
F 6 4

BMAT 2003: Section 2

© UCLES 2003
10
* 21 Diagrams 1 to 5 show five types of levers.

1 2 3
effort load effort
fulcrum

fulcrum fulcrum
load effort load

4 5
load effort
fulcrum

fulcrum
effort load

Diagram 6 shows the bones of the arm with arrows showing its movement up and down.

up

down

Which two levers would be involved in the forcible up and down movement?

up down
A 1 2
B 1 3
C 2 3
D 3 4
E 3 5
F 5 2

BMAT 2003: Section 2

© UCLES 2003
11
* 22 In an a.c. generator driving a resistive load a coil is rotated slowly between the poles of a
magnet. Which of the following will increase if the coil is rotated more quickly?

1 the frequency of the a.c.


2 the amplitude of the a.c.
3 the output e.m.f. of the generator

A 1
B 1 and 2
C 1 and 3
D 2 and 3
E 1, 2 and 3

23 The reactivity of the alkali metals (group 1) increases as the atomic number increases.
Which one of these statements is the correct explanation of this fact?

A The atoms have only one electron in the outer orbit / energy level.
B They are the most reactive of all the groups of metals in the periodic table.
C The number of neutrons in the nuclei increases.
D The atomic mass increases.
E The outer orbit / energy level is further from the nucleus.

24 Which set of events occurs during exhalation?

volume of diaphragm pressure in ribs pivot


thorax becomes thorax
A decreases more convex increases down and outwards
B increases less convex decreases up and outwards
C decreases more convex increases down and inwards
D increases more convex decreases up and inwards
E decreases less convex increases down and outwards
F increases less convex decreases up and inwards

BMAT 2003: Section 2

© UCLES 2003
12
25 Simplify
2
æ 2x 2 y 3 ö
3

ç ÷
ç z ÷ø
è

A B C D
5
6 5 3 6 3 5
4x y 4x y 4x y 4x 2 y 5
z2 z z z

26 The bar charts X and Y show the concentration of five substances in the blood before and
after passing through two organs.

Organ X

concentration
of substances
in the blood

carbon salt glucose oxygen urea


dioxide Key
before

Organ Y after

concentration
of substances
in the blood

carbon salt glucose oxygen urea


dioxide

What are the organs X and Y through which the blood has passed?

X Y
A heart lung
B kidney liver
C kidney lung
D liver kidney
E lung heart
F lung liver

BMAT 2003: Section 2

© UCLES 2003
13
27 A parachutist falls from an aircraft and reaches a terminal velocity. After a while he opens
his parachute and reaches a new (lower) terminal velocity.

Which graph shows how the air resistance (drag) force acting on him varies with time during
the fall?

A B
drag drag

time time

C D
drag drag

time time

28 The enamel on teeth treated with fluorine may include fluoroapatite, which contains Ca2+,
PO43- and F- ions.

Which one of the following is a possible formula for fluoroapatite?

A Ca(PO4)F
B Ca3(PO4)2F
C Ca5(PO4)3F
D Ca7(PO4)5F

PLEASE TURN OVER

BMAT 2003: Section 2

© UCLES 2003
14
29 In triangle PQR Q
Ð QPR = 60°
Ð PQR = 45°
Ð QRP = 75° 45°
RQ = 6 units
6

60° 75°
P
R
Given that cos 60° is ½, find the length of side PR. (The diagram is not drawn to scale.)

A 2
B 3
C 2
D 6

END OF SECTION 2

BMAT 2003: Section 2

© UCLES 2003
15

BLANK PAGE

BMAT 2003: Section 2

© UCLES 2003
16

BLANK PAGE

BMAT 2003: Section 2

© UCLES 2003
Biomedical Admissions Test - 2003

ANSWER KEYS, SECTION 2

Question Correct response Comments Maximum


number points
available for
this question
1 B 1
2 E 1
3 a 2 All must be correct for 1 mark. 1
b 9
c 6
4 B 1
5 D 1
6 76.8 kJ Ek = ½ mv2 1
7 B 1
8 A, C, B, D All must be in correct order for 1 mark. 1
9 B 1
10 B 1
11 A 1
12 A 1
13 D 1
14 (i) 3, 4 & 5 All three and no others for 1 mark. 1
14 (ii) E 1
15 4 minutes 1
16 C 1
17 E 1
18 1st jugular vein All must be correct for 1 mark. 1
2nd E
3rd B
4th C
5th A
6th D
19 C 1
20 C 1
21 E 1
22 E 1
23 E 1
24 C 1
25 B 1
26 C 1
27 A 1
28 C 1
29 A 1

BMAT 2003: Section 2

You might also like